Solutions Manual

User Manual: Pdf

Open the PDF directly: View PDF PDF.
Page Count: 223

DownloadSolutions Manual
Open PDF In BrowserView PDF
An Introduction to Mathematical
Cryptography
Solution Manual
Jeffrey Hoffstein, Jill Pipher, Joseph H. Silverman
c
°2008
by J. Hoffstein, J. Pipher, J.H. Silverman
July 31, 2008

Chapter 1

An Introduction to
Cryptography
Exercises for Chapter 1
Section. Simple substitution ciphers
1.1. Build a cipher wheel as illustrated in Figure 1.1, but with an inner wheel
that rotates, and use it to complete the following tasks. (For your convenience,
there is a cipher wheel that you can print and cut out at www.math.brown.
edu/~jhs/MathCrypto/CipherWheel.pdf.)
(a) Encrypt the following plaintext using a rotation of 11 clockwise.
“A page of history is worth a volume of logic.”
(b) Decrypt the following message, which was encrypted with a rotation of 7
clockwise.
AOLYLHYLUVZLJYLAZILAALYAOHUAOLZLJYLALZAOHALCLYFIVKFNBLZZLZ
(c) Decrypt the following message, which was encrypted by rotating 1 clockwise for the first letter, then 2 clockwise for the second letter, etc.
XJHRFTNZHMZGAHIUETXZJNBWNUTRHEPOMDNBJMAUGORFAOIZOCC
Solution to Exercise 1.1.
apageofhistoryisworthavolumeoflogic
(a)
LALRPZQSTDEZCJTDHZCESLGZWFXPZQWZRTN
This quote is in a court decision of Oliver Wendell Holmes, Jr. (1921).
therearenosecretsbetterthanthesecretsthateverybodyguesses
(b)
AOLYLHYLUVZLJYLAZILAALYAOHUAOLZLJYLAZAOHALCLYFIVKFNBLZZLZ
There are no secrets better than the secrets that everybody
guesses.
This quote is due to George Bernard Shaw, Mrs. Warren’s Profession (1893)
1

2
(c)

Exercises for Chapter 1
whenangrycounttenbeforeyouspeakifveryangryanhundred
XJHRFTNZHMZGAHIUETXZJNBWNUTRHEPOMDNBJMAUGORFAOIZOCC
When angry, count ten before you speak; if very angry, an hundred.

This quote is due to Thomas Jefferson, A Decalogue of Canons. . . (1825).
1.2. Decrypt each of the following Caesar encryptions by trying the various
possible shifts until you obtain readable text.
(a) LWKLQNWKDWLVKDOOQHYHUVHHDELOOERDUGORYHOBDVDWUHH
(b) UXENRBWXCUXENFQRLQJUCNABFQNWRCJUCNAJCRXWORWMB
(c) BGUTBMBGZTFHNLXMKTIPBMAVAXXLXTEPTRLEXTOXKHHFYHKMAXFHNLX
Solution to Exercise 1.2.
ithinkthatishallneverseeabillboardlovelyasatree
(a)
LWKLQNWKDWLVKDOOQHYHUVHHDELOOERDUGORYHOBDVDWUHH
I think that I shall never see, a billboard lovely as a tree.
This quote is due to Ogden Nash, Many Long Years Ago (1945), Song of the
Open Road.
loveisnotlovewhichalterswhenitalterationfinds
(b)
UXENRBWXCUXENFQRLQJUCNABFQNWRCJUCNAJCRXWORWMB
Love is not love which alters when it alteration finds.
This quote is due to William Shakespeare, Sonnet 116.
inbaitingamousetrapwithcheesealwaysleaveroomforthemouse
(c)
BGUTBMBGZTFHNLXMKTIPBMAVAXXLXTEPTRLEXTOXKHHFYHKMAXFHNLX
In baiting a mousetrap with cheese, always leave room for the
mouse.
This quote is due to H.H. Munro (Saki), The Square Egg (1924).
1.3. For this exercise, use the simple substitution table given in Table 1.11.
(a) Encrypt the plaintext message
The gold is hidden in the garden.
(b) Make a decryption table, that is, make a table in which the ciphertext
alphabet is in order from A to Z and the plaintext alphabet is mixed up.
(c) Use your decryption table from (b) to decrypt the following message.
IBXLX JVXIZ SLLDE VAQLL DEVAU QLB
Solution to Exercise 1.3.
(a)

Exercises for Chapter 1

3

a b c d e f g h i j k l m n o p q r s t u v w x y z
S C J A X U F B Q K T P R W E Z H V L I G Y D N M O

Table 1.1: Simple substitution encryption table for exercise 1.3

t h e g o l d i s h i d d e n i n t h e g a r d e n
I B X F E P A Q L B Q A A X W Q W I B X F S V A X W
Breaking it into five letter blocks gives the ciphertext
IBXFE PAQLB QAAXW QWIBX FSVAX W
(b)
d h b w o g u q t c j s y x z l i m a k f r n e v p
A B C D E F G H I J K L M N O P Q R S T U V W X Y Z
(c)
t h e s e c r e t p a s s w o r d i s s w o r d f i s h
I B X L X J V X I Z S L L D E V A Q L L D E V A U Q L B
Putting in word breaks gives the plaintext
The secret password is swordfish.

1.4. Each of the following messages has been encrypted using a simple substitution cipher. Decrypt them. For your convenience, we have given you a
frequency table and a list of the most common bigrams that appear in the
ciphertext. (If you do not want to recopy the ciphertexts by hand, they can
be downloaded or printed from the web site listed in the preface.)
(a) “A Piratical Treasure”
JNRZR BNIGI BJRGZ IZLQR OTDNJ GRIHT USDKR ZZWLG OIBTM NRGJN
IJTZJ LZISJ NRSBL QVRSI ORIQT QDEKJ JNRQW GLOFN IJTZX QLFQL
WBIMJ ITQXT HHTBL KUHQL JZKMM LZRNT OBIMI EURLW BLQZJ GKBJT
QDIQS LWJNR OLGRI EZJGK ZRBGS MJLDG IMNZT OIHRK MOSOT QHIJL
QBRJN IJJNT ZFIZL WIZTO MURZM RBTRZ ZKBNN LFRVR GIZFL KUHIM
MRIGJ LJNRB GKHRT QJRUU RBJLW JNRZI TULGI EZLUK JRUST QZLUK
EURFT JNLKJ JNRXR S
The ciphertext contains 316 letters. Here is a frequency table:
R J I L Z T N Q B G K U M O S H W F E D X V
Freq 33 30 27 25 24 20 19 16 15 15 13 12 12 10 9 8 7 6 5 5 3 2

4

Exercises for Chapter 1

The most frequent bigrams are: JN (11 times), NR (8 times), TQ (6 times),
and LW, RB, RZ, and JL (5 times each).
(b) “A Botanical Code”
KZRNK GJKIP ZBOOB XLCRG BXFAU GJBNG RIXRU XAFGJ BXRME MNKNG
BURIX KJRXR SBUER ISATB UIBNN RTBUM NBIGK EBIGR OCUBR GLUBN
JBGRL SJGLN GJBOR ISLRS BAFFO AZBUN RFAUS AGGBI NGLXM IAZRX
RMNVL GEANG CJRUE KISRM BOOAZ GLOKW FAUKI NGRIC BEBRI NJAWB
OBNNO ATBZJ KOBRC JKIRR NGBUE BRINK XKBAF QBROA LNMRG MALUF
BBG
The ciphertext contains 253 letters. Here is a frequency table:
B R G N A I U K O J L X M F S E Z C T W P V Q
Freq 32 28 22 20 16 16 14 13 12 11 10 10 8 8 7 7 6 5 3 2 1 1 1
The most frequent bigrams are: NG and RI (7 times each), BU (6 times),
and BR (5 times).
(c) In order to make this one a bit more challenging, we have removed all
occurrences of the word “the” from the plaintext.
“A Brilliant Detective”
GSZES GNUBE SZGUG SNKGX CSUUE QNZOQ EOVJN VXKNG XGAHS AWSZZ
BOVUE SIXCQ NQESX NGEUG AHZQA QHNSP CIPQA OIDLV JXGAK CGJCG
SASUB FVQAV CIAWN VWOVP SNSXV JGPCV NODIX GJQAE VOOXC SXXCG
OGOVA XGNVU BAVKX QZVQD LVJXQ EXCQO VKCQG AMVAX VWXCG OOBOX
VZCSO SPPSN VAXUB DVVAX QJQAJ VSUXC SXXCV OVJCS NSJXV NOJQA
MVBSZ VOOSH VSAWX QHGMV GWVSX CSXXC VBSNV ZVNVN SAWQZ ORVXJ
CVOQE JCGUW NVA
The ciphertext contains 313 letters. Here is a frequency table:
V S X G A O Q C N J U Z E W B P I H K D M L R F
Freq 39 29 29 22 21 21 20 20 19 13 11 11 10 8 8 6 5 5 5 4 3 2 1 1
The most frequent bigrams are: XC (10 times), NV (7 times), and CS, OV,
QA, and SX (6 times each).
Solution to Exercise 1.4.
(a) The message was encrypted using the table:
a b c d e f g h i j k l m n o p q r s t u v w x y z
I E B H R W D N T P X U O Q L M A G Z J K V F C S Y
The plaintext reads:
“These characters, as one might readily guess, form a cipher—that is to
say, they convey a meaning; but then, from what is known of Captain Kidd,
I could not suppose him capable of constructing any of the more abstruse
cryptographs. I made up my mind, at once, that this was of a simple species—
such, however, as would appear, to the crude intellect of the sailor, absolutely
insoluble without the key.” (The Gold-Bug, 1843, Edgar Allan Poe)
(b) The message was encrypted using the table:

Exercises for Chapter 1

5

a b c d e f g h i j k l m n o p q r s t u v w x y z
R V C X B F S J K Q P O E I A W D U N G L T Z Y M H
The plaintext reads:
“I was, I think, well educated for the standard of the day. My sister and
I had a German governess. A very sentimental creature. She taught us the
language of flowers—a forgotten study nowadays, but most charming. A yellow
tulip, for instance, means Hopeless Love, while a China Aster means I die of
Jealousy at your feet.” (The Four Suspects, 1933, Agatha Christie)
(c) The message was encrypted using the table:
a b c d e f g h i j k l m n o p q r s t u v w x y z
S D J W V E H C G L R U Z A Q P T N O X I M K Y B F
The plaintext reads (all occurrences of the word “the” were omitted from the
text before encryption):
I am fairly familiar with all forms of secret writing, and am myself (the)
author of a trifling monograph upon (the) subject, in which I analyze one
hundred separate ciphers, but I confess that this is entirely new to me. (The)
object of those who invented this system has apparently been to conceal that
these characters convey a message, and to give (the) idea that they are (the)
mere random sketches of children. (The Adventure of the Dancing Men, 1903,
Sir Arthur Conan Doyle)
1.5. Suppose that you have an alphabet of 26 letters.
(a) How many possible simple substitution ciphers are there?
(b) A letter in the alphabet is said to be fixed if the encryption of the letter
is the letter itself. How many simple substitution ciphers are there that
leave:
(i) no letters fixed?
(ii) at least one letter fixed?
(iii) exactly one letter fixed?
(iv) at least two letters fixed?
(Part (b) is quite challenging! You might try doing the problem first with an
alphabet of four or five letters to get an idea of what is going on.)
Solution to Exercise 1.5.
(a) We can assign A to any of 26 letters, then B to any of the remaining 25
letters, etc. So there are 26! = 403291461126605635584000000 different simple
substitution ciphers.
(b) Let S(n, k) denote the number of permutations of n¡ elements
that fix at
¢
least k elements. You might guess that since there are nk ways to choose k
elements to fix and (n − k)! permutations of the remaining n − k elements,
µ ¶
n
S(n, k) =
(n − k)! ←− Incorrect Formula.
(1.1)
k

6

Exercises for Chapter 1

But this overcounts because any permutation fixing more than n − k elements will be counted multiple times. We can, however, get a useful formula
out of this mistake by modifying it somewhat. If we let R(n, k) denote the
number of permutations of n elements that fix exactly k elements, and !(n−k)
(the subfactorial of (n − k)) denote the number of permutations of n − k elements that fix no elements (such permutations are called derangements), then
the following equation holds:
µ ¶
n
R(n, k) =
!(n − k).
(1.2)
k
How can we compute !n? One way would be to consider cycle decompositions of permutations of n elements, since any derangement of n elements
decomposes into a disjoint union of cycles, with the size of the cycles summing
to n. This, however, is only feasible for relatively small n. It would also be
possible to formulate a recurrence relation, but a method following that tack
would take several steps. We’ll instead use the following fact:
!n = n! − #{permutations that fix at least 1 element}.

(1.3)

Now if we notice that
#{permutations that fix at least 1 element} =
#{permutations that fix element 1}
∪{permutations that fix element 2}
∪ · · · ∪ {permutations that fix element n}

(1.4)

and use an analogue of the following formula in probability (often called the
inclusion–exclusion principle):
P (E1 ∪ E2 ∪ · · · ∪ En ) =

n
X

P (Ei ) +

i=1

+(−1)r+1

X

P (Ei1 ∩ Ei2 ) + . . .

i1  0. Modify your program so
that it returns a solution with u > 0 and u as small as possible. [Hint.
If (u, v) is a solution, then so is (u + b/g, v − a/g).] Redo (c) using your
modified program.
Solution to Exercise 1.12.
(a) A solution for this exercise is not currently available.
(b) A solution for this exercise will not be provided.
(c) and (e): (i) 527 · 43 − 1258 · 18 = 17
(ii) 228 · 51 − 1056 · 11 = 12
(iii) 163961 · 4517 − 167181 · 4430 = 7

Exercises for Chapter 1

13

(iv) 3892394 · 59789 − 239847 · 970295 = 1
(d) If b = 0, then there is a “division by zero” error in step 2. So the program
should check if b = 0, if in that case it should return (a, 1, 0).
1.13. Let a1 , a2 , . . . , ak be integers with gcd(a1 , a2 , . . . , ak ) = 1, i.e., the
largest positive integer dividing all of a1 , . . . , ak is 1. Prove that the equation
a1 u1 + a2 u2 + · · · + ak uk = 1
has a solution in integers u1 , u2 , . . . , uk . (Hint. Repeatedly apply the extended
Euclidean algorithm, Theorem 1.11. You may find it easier to prove a more
general statement in which gcd(a1 , . . . , ak ) is allowed to be larger than 1.)
Solution to Exercise 1.13.
We prove more generally that for any integers a1 , . . . , ak (not all zero),
there is a solution to
a1 u1 + a2 u2 + · · · + ak uk = gcd(a1 , . . . , ak ).
We give the proof using induction on k. If k = 1 there is nothing to prove,
since a1 · 1 = gcd(a1 ). For k = 2, this is already proven in the extended
Euclidean algorithm. So assume now that we know the result for fewer than k
integers, where k ≥ 3, and we want to prove it for k integers. By the induction
hypothesis, we can find a solution to
a1 u1 + a2 u2 + · · · + ak−1 uk−1 = gcd(a1 , . . . , ak−1 ).
To ease notation, we let b = gcd(a1 , . . . , ak−1 ). We apply the extended Euclidean algorithm to the two numbers b and ak , which gives us a solution
to
bv + ak w = gcd(b, ak ).
Multiplying the earlier equation by v and subtituting this equation gives
a1 u1 v + a2 u2 v + · · · + ak−1 uk−1 v = gcd(a1 , . . . , ak−1 )v
= bv
by definition of b,
= −ak w + gcd(b, ak ).
Hence
a1 u1 v + a2 u2 v + · · · + ak−1 uk−1 v + ak w = gcd(b, ak ).
This completes the proof, since from the definition of gcd as the largest integer
dividing all of the listed integers, it’s clear that
¡
¢
gcd(b, ak ) = gcd gcd(a1 , . . . , ak−1 ), ak = gcd(a1 , . . . , ak−1 , ak ).

Section. Modular arithmetic

14

Exercises for Chapter 1

1.14. Let m ≥ 1 be an integer and suppose that
a1 ≡ a2 (mod m)

and

b1 ≡ b2 (mod m).

Prove that
a1 ± b1 ≡ a2 ± b2 (mod m)

and

a1 · b1 ≡ a2 · b2 (mod m).

(This is Proposition 1.13(a).)
Solution to Exercise 1.14.
1.15. Write out the following tables for Z/mZ and (Z/mZ)∗ , as we did in
Figures 1.4 and 1.5.
(a) Make addition and multiplication tables for Z/3Z.
(b) Make addition and multiplication tables for Z/6Z.
(c) Make a multiplication table for the unit group (Z/9Z)∗ .
(d) Make a multiplication table for the unit group (Z/16Z)∗ .
Solution to Exercise 1.15.

(a)

(b)

+
0
1
2
3
4
5

0
0
1
2
3
4
5

1
1
2
3
4
5
0

+
0
1
2

0
0
1
2

1
1
2
0

2
2
0
1

·
0
1
2

0
0
0
0

1
0
1
2

2
0
2
1

2
2
3
4
5
0
1

3
3
4
5
0
1
2

4
4
5
0
1
2
3

5
5
0
1
2
3
4

·
0
1
2
3
4
5

0
0
0
0
0
0
0

1
0
1
2
3
4
5

2
0
2
4
0
2
4

·
1
2
4
5
7
8

1
1
2
4
5
7
8

2
2
4
8
1
5
7

(c)

4
4
8
7
2
1
5

5
5
1
2
7
8
4

7
7
5
1
8
4
2

8
8
7
5
4
2
1

3
0
3
0
3
0
3

4
0
4
2
0
4
2

5
0
5
4
3
2
1

Exercises for Chapter 1
·
1
3
5
7
9
11
13
15

(d)

1
1
3
5
7
9
11
13
15

15
3
3
9
15
5
11
1
7
13

5
5
15
9
3
13
7
1
11

7
7
5
3
1
15
13
11
9

9
9
11
13
15
1
3
5
7

11
11
1
7
13
3
9
15
5

13
13
7
1
11
5
15
9
3

15
15
13
11
9
7
5
3
1

1.16. Do the following modular computations. In each case, fill in the box
with an integer between 0 and m − 1, where m is the modulus.
(a) 347 + 513 ≡
(mod 763).
(b) 3274 + 1238 + 7231 + 6437 ≡
(c) 153 · 287 ≡

(mod 9254).

(mod 353).

(d) 357 · 862 · 193 ≡

(mod 943).

(e) 5327 · 6135 · 7139 · 2187 · 5219 · 1873 ≡
(mod 8157).
(Hint. After each multiplication, reduce modulo 8157 before doing the
next multiplication.)
(f) 1372 ≡
(mod 327).
(g) 3736 ≡
3

(mod 581).
5

4

(h) 23 · 19 · 11 ≡

(mod 97).

Solution to Exercise 1.16.
97
(mod 763).
(a) 347 + 513 ≡
(b) 3274 + 1238 + 7231 + 6437 ≡ 8926 (mod 9254).
(c) 153 · 287 ≡ 139
(mod 353).
(d) 357 · 862 · 193 ≡ 636
(mod 943).
(e) 5327 · 6135 · 7139 · 2187 · 5219 · 1873 ≡ 603
(mod 8157).
2
(f) 137 ≡ 130
(mod 327).
(g) 3736 ≡ 463
(mod 581).
3
5
4
(h) 23 · 19 · 11 ≡
93
(mod 97).
1.17. Find all values of x between 0 and m − 1 that are solutions of the
following congruences. (Hint. If you can’t figure out a clever way to find the
solution(s), you can just substitute each value x = 1, x = 2,. . . , x = m − 1
and see which ones work.)
(a) x + 17 ≡ 23 (mod 37).
(b) x + 42 ≡ 19 (mod 51).
(c) x2 ≡ 3 (mod 11).
(d) x2 ≡ 2 (mod 13).
(e) x2 ≡ 1 (mod 8).

16

Exercises for Chapter 1

(f) x3 − x2 + 2x − 2 ≡ 0 (mod 11).
(g) x ≡ 1 (mod 5) and also x ≡ 2 (mod 7). (Find all solutions modulo 35,
that is, find the solutions satisfying 0 ≤ x ≤ 34.)
Solution to Exercise 1.17.
(a) x ≡ 23 − 17 ≡ 6 (mod 37).
(b) x ≡ 19 − 42 ≡ −23 ≡ 28 (mod 51).
(c) The squares modulo 11 are 02 ≡ 0, 12 ≡ 1, 22 ≡ 4, 32 ≡ 9, 42 ≡ 16 ≡ 5,
etc. The full list is {0, 1, 4, 9, 5, 3, 3, 5, 9, 4, 1}. Thus 52 ≡ 2 (mod 11) and
62 ≡ 2 (mod 11), so there are two solutions, x = 5 and x = 6 .
(d) The squares modulo 13 are {0, 1, 4, 9, 3, 12, 10, 10, 12, 3, 9, 4, 1}. Thus x2 ≡
2 (mod 13) has no solutions .
(e) The solutions to x2 ≡ 1 (mod 8) are x = 1, x = 3, x = 5 and x = 7 .
(f) Plugging x = 0, 1, 2, . . . , 10 into x3 − x2 + 2x − 2 and reducing modulo 11,
we find the three solutions x = 1, x = 3, and x = 8 .
(g) One method is to try all values x = 0, 1, 2, . . . , 34. A faster method is
to list the solutions to x ≡ 1 (mod 5), namely 1, 6, 11, 16, 21, 26, 31, . . . and
reduce them modulo 7 to see which ones are congruent to 2 modulo 7. Thus
working modulo 7,
1 ≡ 1,

6 ≡ 6,

11 ≡ 4,

16 ≡ 2,

21 ≡ 0,

26 ≡ 5,

31 ≡ 3.

Thus the solution is x = 16 .
1.18. Suppose that g a ≡ 1 (mod m) and that g b ≡ 1 (mod m). Prove that
g gcd(a,b) ≡ 1

(mod m).

Solution to Exercise 1.18.
The extended Euclidean algorithm says that there are integers u and v
satisfying au + bv = gcd(a, b). Then
g gcd(a,b) ≡ g au+bv ≡ (g a )u · (g b )v ≡ 1u · 1v ≡ 1 (mod p).

1.19. Prove that if a1 and a2 are units modulo m, then a1 a2 is a unit modulo
m.
Solution to Exercise 1.19.
By definition of unit, there are numbers b1 and b2 so that
a1 b1 ≡ 1 (mod m)

and a2 b2 ≡ 1 (mod m).

Then
(a1 a2 )(b1 b2 ) ≡ (a1 b1 )(a2 b2 ) ≡ 1 · 1 ≡ 1 (mod m),
so a1 a2 is a unit. Its inverse is b1 b2 .

Exercises for Chapter 1

17

1.20. Prove that m is prime if and only if φ(m) = m − 1, where φ is Euler’s
phi function.
Solution to Exercise 1.20.
Suppose first that m is prime. Let k be any number between 1 and m − 1
and let d = gcd(k, m). Then d | m, so the fact that m is prime tells us that
either d = 1 or d = m. But also d | k and 1 ≤ k < m, so we have d <
m. Hence d = 1. This proves that every number k between 1 and m − 1
satisfies gcd(k, m) = 1. Hence
©
ª
φ(m) = # 1 ≤ k < m : gcd(k, m) = 1 = #{1, 2, 3, . . . , m − 1} = m − 1.
Next suppose that φ(m) = m − 1. This means that every number k between 1 and m − 1 satisfies gcd(k, m) = 1. Suppose that d divides m and
that d 6= m. Then 1 ≤ d ≤ m − 1, so gcd(d, m) = 1. But the fact that d
divides m implies that gcd(d, m) = d. Hence d = 1. This proves that the only
divisors of m are 1 and m, so m is prime.
1.21. Let m ∈ Z.
(a) Suppose that m is odd. What integer between 1 and m − 1 equals 2−1 mod m?
(b) More generally, suppose that m ≡ 1 (mod b). What integer between 1
and m − 1 is equal to b−1 mod m?
Solution to Exercise 1.21.
(a) The fact that m is odd means that
2·

m+1
2

is an integer, and clearly

m+1
= m + 1 ≡ 1 (mod m).
2

(b) The assumption that m ≡ 1 (mod b) means that m−1
is an integer, so
b
we have
m−1
b·
= m − 1 ≡ −1 (mod m).
b
This is almost what we want, so multiply by −1 to get
b·

1−m
= 1 − m ≡ 1 (mod m).
b

Unfortunately, 1−m
is negative, but we can add on multiples of m without
b
1+(b−1)m
changing its value modulo m. Thus 1−m
is an integer and
b +m=
b
b·

1 + (b − 1)m
= 1 + (b − 1)m ≡ 1
b

Hence b−1 mod m is equal to

1+(b−1)m
b

(mod m).

.

1.22. Let m be an odd integer and let a be any integer. Prove that 2m + a2
can never be a perfect square. (Hint. If a number is a perfect square, what
are its possible values modulo 4?)

18

Exercises for Chapter 1

Solution to Exercise 1.22.
Any number squared is either 0 or 1 modulo 4. But
(
2 + 0 ≡ 2 if a is even,
2
2
2m + a ≡ 2 + a ≡
2 + 1 ≡ 3 if a is odd.
Thus 2m + a2 is either 2 or 3 modulo 4, so it can never be a perfect square.
1.23. (a) Find a single value x that simultaneously solves the two congruences
x≡3

(mod 7)

and

x ≡ 4 (mod 9).

(Hint. Note that every solution of the first congruence looks like x = 3+7y
for some y. Substitute this into the second congruence and solve for y;
then use that to get x.)
(b) Find a single value x that simultaneously solves the two congruences
x ≡ 13

(mod 71)

and

x ≡ 41 (mod 97).

(c) Find a single value x that simultaneously solves the three congruences
x≡4

(mod 7),

x ≡ 5 (mod 8),

and

x ≡ 11 (mod 15).

(d) Prove that if gcd(m, n) = 1, then the pair of congruences
x≡a

(mod m)

and

x ≡ b (mod n)

has a solution for any choice of a and b. Also give an example to show
that the condition gcd(m, n) = 1 is necessary.
Solution to Exercise 1.23.
(a) x = 31 (b) x = 5764 (c) x = 221
(d) The solutions to the first congruence look like x = a + my for any
integer y. Substituting into the second congruence yields
a + my ≡ b (mod n),
so we want to find a value of z such that
a + my − b = nz.
In other words, we need integers y and z satisfying
my − nz = b − a.
We are given that gcd(m, n) = 1, so we can find integers u and v satisfying
mu + nv = 1. Multiplying this by b − a gives
mu(b − a) + nv(b − a) = b − a,

Exercises for Chapter 1

19

so we can take y = u(b − a) and z = v(b − a). Then we have x = a + my =
a + mu(b − a).
To summarize, we first solve mu + nv = 1 and then we take
x = a + mu(b − a) = a + (1 − nv)(b − a) = b + nv(b − a).
The two expressions for x show that x ≡ a (mod m) and x ≡ v (mod n).
This exercise is a special case of the Chinese remainder theorem, which is
covered in Chapter 2.
1.24. Let N , g, and A be positive integers (note that N need not be
prime). Prove that the following algorithm, which is a low-storage variant
of the square-and-multiply algorithm described in Section 1.3.2, returns the
value g A (mod N ). (In Step 4 we use the notation bxc to denote the greatest
integer function, i.e., round x down to the nearest integer.)
Input. Positive integers N , g, and A.
1. Set a = g and b = 1.
2. Loop while A > 0.
3. If A ≡ 1 (mod 2), set b = b · a (mod N ).
4. Set a = a2 (mod N ) and A = bA/2c.
5. If A > 0, continue with loop at Step 2.
6. Return the number b, which equals g A (mod N ).
Solution to Exercise 1.24.
*** fill in solution
1.25. Use the square-and-multiply algorithm described in Section 1.3.2, or the
more efficient version in Exercise 1.24, to compute the following powers.
(a) 17183 (mod 256).
(b) 2477 (mod 1000).
(c) 11507 (mod 1237).
Solution to Exercise 1.25.
(a)

183 = 1 + 2 + 22 + 24 + 25 + 27 ,

(b) 477 = 1 + 22 + 23 + 24 + 26 + 27 + 28 ,
(c) 507 = 1 + 2 + 23 + 24 + 25 + 26 + 27 + 28 ,

17183 (mod 256) = 113 .
2477 (mod 1000) = 272
11507 (mod 1237) = 322 .

Section. Prime numbers, unique factorization, and finite fields
1.26. Let {p1 , p2 , . . . , pr } be a set of prime numbers, and let
N = p1 p2 · · · pr + 1.

20

Exercises for Chapter 1

Prove that N is divisible by some prime not in the original set. Use this fact
to deduce that there must be infinitely many prime numbers. (This proof of
the infinitude of primes appears in Euclid’s Elements. Prime numbers have
been studied for thousands of years.)
Solution to Exercise 1.26.
Let q be any prime that divides N . (Since N ≥ 2, we know that it must
be divisible by some prime.) Suppose that q were equal to some pi . Then we
would have
1 = N − p1 p2 · · · pr ≡ 0 (mod q),
since q would divide both of the terms N and p1 · · · pr . But then q | 1, which
is impossible. Therefore q is not equal to any of the pi ’s.
Next suppose that there were only finitely many primes. That means we
can list them, say p1 , p2 , . . . , pr . But from the first part of the exercise, we can
create a new prime that’s not in our list. This contradicts the assumption that
there are finitely many primes, and hence proves that there must be infinitely
many primes.
1.27. Without using the fact that every integer has a unique factorization
into primes, prove that if gcd(a, b) = 1 and if a | bc, then a | c. (Hint. Use the
fact that it is possible to find a solution to au + bv = 1.)
Solution to Exercise 1.27.
From the extended Euclidean algorithm, we can solve au+bv = 1. Multiply
by c to get acu + bcv = c. We are given that a | bc, so there is an integer d
satisfying bc = ad. Substituting this gives acu + adv = c. Thus a(cu + dv) = c,
which shows that a | c.
1.28. Compute the following ordp values:
(a) ord2 (2816).
(b) ord7 (2222574487).
(c) ordp (46375) for each of p = 3, 5, 7, and 11.
Solution to Exercise 1.28.
(a) ord2 (2816) = 8.
(b) ord7 (2222574487) = 5.
(c) Let a = 46375. Then ord3 (a) = 0, ord5 (a) = 3, ord7 (a) = 1,
ord11 (a) = 0.
1.29. Let p be a prime number. Prove that ordp has the following properties.
(a) ordp (ab) = ordp (a) + ordp (b). (Thus ordp resembles the logarithm function, since it converts
© multiplication
ª into addition!)
(b) ordp (a + b) ≥ min ordp (a), ordp (b) .
©
ª
(c) If ordp (a) 6= ordp (b), then ordp (a + b) = min ordp (a), ordp (b) .
A function satisfying properties (a) and (b) is called a valuation.

Exercises for Chapter 1

21

Solution to Exercise 1.29.
(a) By definition of ordp , we have
a = pordp (a) A

and

b = pordp (b) B

with

p-A

and

p - B.

Then
ab = pordp (a) A · pordp (b) B = pordp (a)+ordp (b) AB

with

p - AB,

so by definition,
ordp (ab) = ordp (a) + ordp (b).
(b) We continue with the notation from (a) and, without loss of generality,
we switch a and b if necessary so that ordp (a) ≥ ordp (b). Then
³
´
a + b = pordp (a) A + pordp (b) B = pordp (b) pordp (a)−ordp (b) A + B .
Thus pordp (b) | a + b, so by definition of ordp we have
ordp (a + b) ≥ ordp (b).
(Note that we’ve set things up so that ordp (b) = min{ordp (a), ordp (b)}, so
this is the result that we want.)
(c) We continue with the notation from (a) and (b), but for this part we are
given that ordp (a) > ordp (b). We also know that p - B, so it follows that
³
´
p - pordp (a)−ordp (b) A + B ,
since the exponent of p on the first term is positive. Hence pordp (b) is the
largest power of p dividing a + b, which proves that
ordp (a + b) = ordp (b).

Section. Powers and primitive roots in finite fields
1.30. For each of the following primes p and numbers a, compute a−1 mod p
in two ways: (i) Use the extended Euclidean algorithm. (ii) Use the fast power
algorithm and Fermat’s little theorem. (See Example 1.28.)
(a) p = 47 and a = 11.
(b) p = 587 and a = 345.
(c) p = 104801 and a = 78467.
Solution to Exercise 1.30.
(a) (i) We use the extended Euclidean algorithm to solve
11u + 47v = 1.

22

Exercises for Chapter 1

The solution is (u, v) = (−17, 4), so 11−1 ≡ −17 ≡ 30 (mod 47). (ii) Fermat’s
little theorem gives
11−1 ≡ 1145 ≡ 30 (mod 47).
(b) (i) We use the extended Euclidean algorithm to solve
345u + 587v = 1.
The solution is (u, v) = (114, −67), so 345−1 ≡ 114 (mod 587). (ii) Fermat’s
little theorem gives
345−1 ≡ 345585 ≡ 114 (mod 587).
(c) (i) We use the extended Euclidean algorithm to solve
78467u + 104801v = 1.
The solution is (u, v) = (1763, −1320), so 78467−1 ≡ 1763 (mod 104801). (ii)
Fermat’s little theorem gives
78467−1 ≡ 78467104799 ≡ 1763 (mod 104801).

1.31. Let p be a prime and let q be a prime that divides p − 1.
(a) Let a ∈ F∗p and let b = a(p−1)/q . Prove that either b = 1 or else b has
order q. (Recall that the order of b is the smallest k ≥ 1 such that bk = 1
in F∗p . Hint. Use Proposition 1.30.)
(b) Suppose that we want to find an element of F∗p of order q. Using (a), we
can randomly choose a value of a ∈ F∗p and check whether b = a(p−1)/q
satisfies b 6= 1. How likely are we to succeed? In other words, compute
the value of the ratio
#{a ∈ F∗p : a(p−1)/q 6= 1}
.
#F∗p
(Hint. Use Theorem 1.31.)
Solution to Exercise 1.31.
(a) Let k be the order of b, i.e., the smallest exponent such that bk = 1. We
know that bq = ap−1 = 1 from Fermat’s little theorem. Then Proposition 1.30
tells us that k divides q, and since q is prime, it follows that either k = q
or k = 1. Thus either b has order q, or else it has order 1, in which case b =
b1 = 1.
(b) Let g ∈ F∗p be a primitive root. Then every a ∈ F∗p has the form g i for
some 0 ≤ i < p − 1. We’ll count the number of a with a(p−1)/q = 1. Thus

Exercises for Chapter 1

23

#{a ∈ F∗p : a(p−1)/q = 1} = #{0 ≤ i < p − 1 : (g i )(p−1)/q = 1}
= #{0 ≤ i < p − 1 : g i(p−1)/q = 1}.
Since g has order p − 1, we have g k = 1 if and only if p − 1 | k. Hence
g i(p−1)/q = 1

⇐⇒

p − 1 | i(p − 1)/q

⇐⇒

q | i.

Hence
#{a ∈ F∗p : a(p−1)/q = 1} = #{0 ≤ i < p − 1 : q | i} =

p−1
.
q

It follows that
#{a ∈ F∗p : a(p−1)/q 6= 1} = p − 1 − #{a ∈ F∗p : a(p−1)/q = 1}
µ
¶
1
p−1
=p−1−
= (p − 1) 1 −
.
q
q
Hence

#{a ∈ F∗p : a(p−1)/q 6= 1}
1
=1− ,
#F∗p
q

so if q is large, we have a very good chance of succeeding on our first try.
1.32. Recall that g is called a primitive root modulo p if the powers of g give
all nonzero elements of Fp .
(a) For which of the following primes is 2 a primitive root modulo p?
(i) p = 7
(ii) p = 13
(iii) p = 19
(iv) p = 23
(b) For which of the following primes is 3 a primitive root modulo p?
(i) p = 5
(ii) p = 7
(iii) p = 11
(iv) p = 17
(c) Find a primitive root for each of the following primes.
(i) p = 23
(ii) p = 29
(iii) p = 41
(iv) p = 43
(d) Find all primitive roots modulo 11. Verify that there are exactly φ(10) of
them, as asserted in Remark 1.33.
(e) Write a computer program to check for primitive roots and use it to find
all primitive roots modulo 229. Verify that there are exactly φ(229) of
them.
(f) Use your program from (e) to find all primes less than 100 for which 2 is
a primitive root.
(g) Repeat the previous exercise to find all primes less than 100 for which 3
is a primitive root. Ditto to find the primes for which 4 is a primitive
root.
Solution to Exercise 1.32.
(a) (i) No. (ii) Yes. (iii) Yes. (iv) No.
(b) (i) Yes. (ii) Yes. (iii) No. (iv) Yes.
(c) In each case, we list the smallest primitive root

24

Exercises for Chapter 1

(i) p = 23, g = 5. (ii) p = 29, g = 2. (iii) p = 41, g = 6. (iv) p = 43, g = 3.
(d) The primitive roots modulo 11 are {2, 6, 7, 8}. There are φ(10) = 4 of
them.
(e) The primitive roots modulo 229 are
{6, 7, 10, 23, 24, 28, 29, 31, 35, 38, 39, 40, 41, 47, 50, 59, 63, 65, 66,
67, 69, 72, 73, 74, 77, 79, 87, 90, 92, 96, 98, 102, 105, 110, 112, 113,
116, 117, 119, 124, 127, 131, 133, 137, 139, 142, 150, 152, 155, 156, 157,
160, 162, 163, 164, 166, 170, 179, 182, 188, 189, 190, 191, 194, 198, 200,
201, 205, 206, 219, 222, 223}.
There are exactly φ(228) = 72 of them.
(f) 2 is a primitive root modulo p for p ∈ {3, 5, 11, 1319, 29, 37, 53, 59, 61, 67, 83}
and for no other primes less than 100. It is conjectured that 2 is a primitive
root for infinitely many primes (Artin’s conjecture).
(g) 3 is a primitive root modulo p for p ∈ {5, 7, 17, 19, 29, 31, 43, 53, 79, 89}
and for no other primes less than 100. On the other hand, there are no primes
for which 4 is a primitive root. This is because 4 = 22 is a square, so the
powers of 4 can hit at most half of the possible nonzero values modulo p.
1.33. Let p be a prime such that q = 21 (p − 1) is also prime. Suppose that g
is an integer satisfying
g 6≡ ±1

(mod p)

and

g q 6≡ 1

(mod p).

Prove that g is a primitive root modulo p.
Solution to Exercise 1.33.
Let n be the order of g, i.e., the smallest power of g that is congruent to 1.
Then n divides p − 1 from Proposition 1.30. Since p − 1 = 2q with q prime,
this means that
n=1

orn = 2

orn = q

orn = 2q.

We are given that g 6≡ ±1 (mod p), so n 6= 1 and n 6= 2, and we are also given
that g q 6≡ ±1 (mod p), so n 6= q.The only value left is n = 2q. This proves
that n = p − 1, so g is a primitive root modulo p.
1.34. This exercise begins the study of squares and square roots modulo p.
(a) Let p be an odd prime number and let b be an integer with p - b. Prove
that either b has two square roots modulo p or else b has no square roots
modulo p. In other words, prove that the congruence
X 2 ≡ b (mod p)
has either two solutions or no solutions in Z/pZ. (What happens for p =
2? What happens if p | b?)

Exercises for Chapter 1

25

(b) For each of the following values of p and b, find all of the square roots
of b modulo p.
(i) (p, b) = (7, 2)
(ii) (p, b) = (11, 5)
(iii) (p, b) = (11, 7)
(iv) (p, b) = (37, 3)
(c) How many square roots does 29 have modulo 35? Why doesn’t this contradict the assertion in (a)?
(d) Let p be an odd prime and let g be a primitive root modulo p. Then
any number a is equal to some power of g modulo p, say a ≡ g k (mod p).
Prove that a has a square root modulo p if and only if k is even.
Solution to Exercise 1.34.
(a) If X = a1 and X = a2 are square roots of b modulo p, then p divides
a21 − b and p divides a22 − b, so p divides their difference
(a21 − b) − (a22 − b) = a21 − a22 = (a1 − a2 )(a1 + a2 ).
It follows that p divides either a1 − a2 or a1 + a2 . If the former, then a1 ≡ a2
(mod p), and if the latter, then a1 ≡ −a2 (mod p). Thus there are at most
two possibilities.
Further, if there is one solution a and if p ≥ 3, then p − a is a second
solution different from a, so if there are any solutions, then there are exactly
two solutions. On the other hand, if p = 2, then X 2 ≡ b (mod p) always has
exactly one solution, namely X = b.
(b) (i) 3 and 4.
(ii) 4 and 7.
(iii) None.
(iv) 15 and 22.
(c) 8, 13, 22, and 27 are all solutions to X 2 ≡ 29 (mod 35), so 29 has four
square roots modulo 35. This does not contradict (a), since the modulus 35
is not prime.
(d) Suppose first that k is even, say k = 2j. Then
a ≡ g k ≡ g 2j ≡ (g j )2

(mod p),

so a is a square modulo p.
Next suppose a is a square, say a ≡ b2 (mod p). Since g is a primitive root,
we can write b ≡ g i (mod p) for some exponent i. Then
g k ≡ a ≡ b2 ≡ (g i )2 ≡ g 2i

(mod p).

Thus g k−2i ≡ 1 (mod p), and the fact that g is a primitive root implies
that p − 1 divides k − 2i. But p − 1 is even, hence 2 divides k − 2i, so 2
divides k.
1.35. Let p ≥ 3 be a prime and suppose that the congruence
X 2 ≡ b (mod p)
has a solution.

26

Exercises for Chapter 1

(a) Prove that for every exponent e ≥ 1 the congruence
X 2 ≡ b (mod pe )

(1.11)

has a solution. (Hint. Use induction on e. Build a solution modulo pe+1
by suitably modifying a solution modulo pe .)
(b) Let X = α be a solution to X 2 ≡ b (mod p). Prove that in (a), we can find
a solution X = β to X 2 ≡ b (mod pe ) that also satisfies β ≡ α (mod p).
(c) Let β and β 0 be two solutions as in (b). Prove that β ≡ β 0 (mod pe ).
(d) Use Exercise 1.34 to deduce that the congruence (1.14) has either two
solutions or no solutions modulo pe .
Solution to Exercise 1.35.
We do (a), (b), and (c) simultaneously. We are given that X = α is a
solution to X 2 ≡ b (mod p). We are going to prove by induction that for
every e ≥ 1 there is a unique value β mod pe satisfying both
β2 ≡ b

(mod pe )

and

β≡α

(mod p).

The case e = 1 is given to us, we must take β = α. Now suppose that we
have a value of β that works for e, and we ask for all solutions that work for
e + 1. Note that if γ is a solution for e + 1, then γ mod pe is a solution for e.
So by the uniqueness part of the induction hypothesis, we would need to have
γ ≡ β (mod pe ). In other words, if there are any solutions γ for e + 1, then γ
is forced to have the form
γ = β + ype

for some integer y.

What we want to do is show that there is a unique value of y modulo p that
makes γ into a solution of X 2 ≡ b (mod pe+1 ).
We also want to use the fact that β is a solution to X 2 ≡ b (mod pe ). This
means that
β 2 = b + pe B
for some integer B.
Now we substitute γ = β + ype into the congruence X 2 ≡ b (mod pe+1 ) and
try to solve for y. Thus
(β + ype )2 ≡ b

(mod pe+1 )

β 2 + 2ype + y 2 p2e ≡ b

(mod pe+1 )

β 2 + 2ype ≡ b

(mod pe+1 )

e

e

b + p B + 2yp ≡ b
pe (B + 2y) ≡ 0

e+1

(mod p

)

(mod pe+1 ).

Thus we need to solve
B + 2y ≡ 0 (mod p).

since 2e ≥ e + 1,
since β 2 = b + pe B,

Exercises for Chapter 1

27

This has a unique solution for y. (Note that p is assumed to be an odd prime.
If p = 2, the argument does not work.) We can even solve explicitly,
y≡

p−1
B
2

(mod p).

This completes the proof that for every e ≥ 1 there exists a unique value of β
(mod pe ) satisfying
β2 ≡ b

(mod pe )

and

β≡α

(mod p),

which gives all of the statements in (a), (b), and (b).
(d) From the earlier exercise we know that X 2 ≡ b (mod p) has either 0
or 2 solutions. If it has no solutions, there there certainly aren’t any solutions
to X 2 ≡ b (mod pe ) for e ≥ 2, since any such solution could always be
reduced modulo p. On the other hand, if X 2 ≡ b (mod p) has two solutions,
then (a), (b), and (c) together imply that there are also two solutions to
X 2 ≡ b (mod pe ) for each e ≥ 1, since the solutions to X 2 ≡ b (mod p) are
matched up one-to-one with the solutions to X 2 ≡ b (mod pe ).
This exercise is a very special case of Hensel’s lemma.
1.36. Compute the value of
2(p−1)/2 (mod p)
for every prime 3 ≤ p < 20. Make a conjecture as to the possible values of
2(p−1)/2 (mod p) when p is prime and prove that your conjecture is correct.
Solution to Exercise 1.36.
p=3

21 = 2 ≡ 2

p=5

22 = 4 ≡ 4

p=7

23 = 8 ≡ 1

p = 11

25 = 32 ≡ 10

p = 13

26 = 64 ≡ 12

p = 17

28 = 256 ≡ 1

p = 19

29 = 512 ≡ 18

Conjecture: 2(p−1)/2 is congruent to either 1 or p − 1 modulo p.
Proof : Let a = 2(p−1)/2 . Then a2 ≡ 2p−1 ≡ 1 (mod p) by Fermat’s little
theorem. Therefore a ≡ ±1 (mod p). To see this last fact, note that p | (a2 −1),
so p | (a − 1)(a + 1), so since p is prime, it divides one of a − 1 or a + 1, which
is just another way of saying that a ≡ ±1 (mod p).
Section. Cryptography by hand

28

Exercises for Chapter 1

1.37. Write a 2 to 5 page paper on one of the following topics, including both
cryptographic information and placing events in their historical context:
(a) Cryptography in the Arab world to the 15th century.
(b) European cryptography in the 15th and early 16th centuries.
(c) Cryptography and cryptanalysis in Elizabethan England.
(d) Cryptography and cryptanalysis in the 19th century.
(e) Cryptography and cryptanalysis during World War I.
(f) Cryptography and cryptanalysis during World War II.
(Most of these topics are too broad for a short term paper, so you should
choose a particular aspect on which to concentrate.)
Solution to Exercise 1.37.
A solution for this exercise will not be provided.
1.38. A homophonic cipher is a substitution cipher in which there may be
more than one ciphertext symbol for each plaintext letter. Here is an example
of a homophonic cipher, where the more common letters have several possible
replacements.
a bc d e f g h i j k l m n o p q r s t
! 4# $ 1 % & * ( ) 3 2 = + [ 9 ] { } :
♥ ◦ ? ℵ 6 % . ♦∧
& ∆∇ 8 ♣ Ω ∨ ⊗
Θ
∞
⇑
\
• ¯
/ ⊕
.
⇓
⇒-

u vwxyz
; 7<>5?
♠
[
⇐

Decrypt the following message.
( % ∆ ♠ ⇒ \ # 4 ∞ : ♦ 6 % ¯ [ ℵ 8 % 2 [ 7 ⇓ ♣ & ♥ 5 ¯ ∇
Solution to Exercise 1.38.
(
I
%
f

%
f
2
l

∆
m
[
o

♠
u
7
v

⇒
s
⇓
e

\
i
♣
p

#
c
&
l

4
b
♥
a

∞
e
5
y

:
t
¯
o

♦
h
∇
n

6
e

%
f

¯
o

[
o

ℵ
d

8
o

From Shakespeare’s Twelfth Night: If music be the food of love, play
on...
1.39. A transposition cipher is a cipher in which the letters of the plaintext
remain the same, but their order is rearranged. Here is a simple example in
which the message is encrypted in blocks of 25 letters at a time.1 Take the
given 25 letters and arrange them in a 5-by-5 block by writing the message
horizontally on the lines. For example, the first 25 letters of the message
Now is the time for all good men to come to the aid...
is written as
1 If the number of letters in the message is not an even multiple of 25, then extra random
letters are appended to the end of the message.

Exercises for Chapter 1

29
N
T
M
A
O

O
H
E
L
D

W
E
F
L
M

I
T
O
G
E

S
I
R
O
N

Now the cipehrtext is formed by reading the letters down the columns, which
gives the ciphertext
NTMAO OHELD WEFLM ITOGE SIRON.
(a) Use this transposition cipher to encrypt the first 25 letters of the message
Four score and seven years ago our fathers...
(b) The following message was encrypted using this transposition cipher. Decrypt it.
WNOOA HTUFN EHRHE NESUV ICEME
(c) There are many variations on this type of cipher. We can form the letters
into a rectangle instead of a square, and we can use various patterns to
place the letters into the rectangle and to read them back out. Try to
decrypt the following ciphertext, in which the letters were placed horizontally into a rectangle of some size and then read off vertically by
columns.
WHNCE STRHT TEOOH ALBAT DETET SADHE
LEELL QSFMU EEEAT VNLRI ATUDR HTEEA
(For convenience, we’ve written the ciphertext in 5 letter blocks, but that
doesn’t necessarily mean that the rectangle has a side of length 5.)
Solution to Exercise 1.39.
(a) Ciphertext: FCNER OODNS URSYA REEEG SAVAO
F
C
N
E
R

O
O
D
N
S

U
R
S
Y
A

R
E
E
E
G

S
A
V
A
O

(b) Plaintext: When in the course of human events it becomes necessary...
Hopefully everyone recognizes the first few words of the American Declaration of Independence.
W
N
O
O
A

H
T
U
F
N

E
H
R
H
E

N
E
S
U
V

I
C
E
M
E

30

Exercises for Chapter 1

(c) Plaintext: We hold these truths to be self-evident, that all men
are created equal, that they are endowed by their Creator...
Another excerpt from the Declaration of Independence. It was encrypted
using a 15-by-4 rectangle.
W
H
N
C

E
S
T
R

H
T
T
E

O
O
H
A

L
B
A
T

D
E
T
E

T
S
A
D

H
E
L
E

E
L
L
Q

S
F
M
U

E
E
E
A

T
V
N
L

R
I
A
T

U
D
R
H

T
E
E
A

Section. Symmetric ciphers and asymmetric ciphers
1.40. Encode the following phrase (including capitalization, spacing and
punctuation) into a string of bits using the ASCII encoding scheme given
in Table 1.10.
Bad day, Dad.
Solution to Exercise 1.40.
B
66

a
97

d
100

32

d
100

a
97

y
121

,
44

01000010

01100001

01100100

00100000

01100100

01100001

01111001

00101100

32

D
68

a
97

d
100

.
46

00100000

01000100

01100001

01100100

00101110

Thus the phrase “Bad day, Dad.” becomes the ASCII list of bits
0100001001100001011001000010000001100100011000010111
1001001011000010000001000100011000010110010000101110

1.41. Consider the affine cipher with key k = (k1 , k2 ) whose encryption and
decryption functions are given by (1.11) on page 43.
(a) Let p = 541 and let the key be k = (34, 71). Encrypt the message m =
204. Decrypt the ciphertext c = 431.
(b) Assuming that p is public knowledge, explain why the affine cipher is
vulnerable to a chosen plaintext attack. (See Property 4 on page 38.)
How many plaintext/ciphertext pairs are likely to be needed in order to
recover the private key?
(c) Alice and Bob decide to use the prime p = 601 for their affine cipher. The
value of p is public knowledge, and Eve intercepts the ciphertexts c1 =
324 and c2 = 381 and also manages to find out that the corresponding
plaintexts are m1 = 387 and m2 = 491. Determine the private key and
then use it to encrypt the message m3 = 173.
(d) Suppose now that p is not public knowledge. Is the affine cipher still
vulnerable to a chosen plaintext attack? If so, how many plaintext/ciphertext pairs are likely to be needed in order to recover the private key?

Exercises for Chapter 1

31

Solution to Exercise 1.41.
(a) The encryption of m = 204 is c ≡ 34·204+71 ≡ 7007 ≡ 515 (mod 541).
The inverse of k1 is 34−1 ≡ 366 (mod 541). The decryption of c = 431 is
m ≡ 366(431 − 71) ≡ 297 (mod 541).
(b) Given two plaintext/ciphertext pairs, one can solve the two linear congruences
c1 ≡ k1 · m1 + k2

(mod p)

and

c2 ≡ k1 · m2 + k2

and

381 ≡ k1 · 491 + k2

(mod p)

for the two unknowns k1 and k2 .
(c) Eve knows that
324 ≡ k1 · 387 + k2

(mod 601)

(mod 601)

She subtracts the first equation from the second to get
57 ≡ k1 · 104 (mod 601).
She computes 104−1 ≡ 549 (mod 601), and hence
k1 ≡ 57 · 104−1 ≡ 41

(mod 601).

Then she uses either of the above congruences to recover k2 ,
k2 ≡ 324 − k1 · 387 ≡ 83 (mod 601).
Eve now knows Alice and Bob’s private key, so she can encrypt a message,
c3 ≡ k1 · m3 + k2 ≡ 41 · 173 + 83 ≡ 565 (mod 601).
(d) Yes. Suppose that we have three plaintext/ciphertext pairs,
(m1 , c1 ), (m2 , c2 ), (m3 , c3 ).
This gives us a system of three congruences
c1 ≡ k1 m1 + k2
c2 ≡ k1 m2 + k2
c3 ≡ k1 m3 + k2

(mod p)
(mod p)
(mod p)

We can write this in suggestive matrix and vector notation at


c1 m1 1 ¡
¢ ¡
¢
c2 m2 1 1 −k1 −k2 ≡ 0 0 0
(mod p).
c3 m3 1
Using linear algebra modulo p, this implies that the determinant of the matrix
satisfies

32

Exercises for Chapter 1



c1 m1 1
det c2 m2 1 ≡ 0
c3 m3 1

(mod p).

Thus three plaintext/ciphertext pairs allows Eve to compute a number,
namely


c1 m1 1
D = det c2 m2 1
c3 m3 1
that is divisible by the secret prime p. If Eve can factor D, then at worst she
has a few possible values of p to check. So three pairs may be enough to break
the cipher.
More generally, if Eve has n different pairs, she can compute determinant
values D1 , . . . , Dn−2 by using different pairs in the last row of the matrix
(keeping the first two rows the same). This gives her a bunch of numbers that
are divisible by p, and within a short time she will almost certain find that
gcd(D1 , . . . , Dn−2 ) is equal to p.
1.42. Consider the Hill cipher defined by (1.11),
ek (m) ≡ k1 · m + k2

(mod p)

and

dk (c) ≡ k1−1 · (c − k2 ) (mod p),

where m, c, and k2 are column vectors of dimension n, and k1 is an n-by-n
matrix.
(a) We use the vector Hill cipher with p = 7 and the key k1 = ( 12 32 )
and k2 = ( 54 ).
(i) Encrypt the message m = ( 21 ).
(ii) What is the matrix k1−1 used for decryption?
(iii) Decrypt the message c = ( 35 ).
(b) Explain why the Hill cipher is vulnerable to a chosen plaintext attack.
(c) The following plaintext/ciphertext pairs were generated using a Hill cipher with the prime p = 11. Find the keys k1 and k2 .
m1 = ( 54 ) ,

c1 = ( 18 ) ,

8 ),
m2 = ( 10

c2 = ( 85 ) ,

m3 = ( 71 ) ,

c3 = ( 87 ) .

(d) Explain how any simple substitution cipher that involves a permutation
of the alphabet can be thought of as a special case of a Hill cipher.
Solution to Exercise 1.42.
(a-i) ek (m) = 5 3 .
−1
36
(a − kii)
1 = 4 5.
(a − diii)
k (c) = 0 4 .
(b) Each known plaintext/ciphertext pair gives a congruence of the form
c ≡ k1 ·m+k2 (mod p). Writing this out gives n linear equations for the n2 +n
unknown entries of k1 and k2 . Hence n+1 plaintext/ciphertext pairs probably
gives enough equations to solve for the keys k1 and k2 .

Exercises for Chapter 1

33

(c) We let k1 = ( xz wy ) and k2 = ( uv ). Then the congruence c1 = k1 m1 +
k2 (mod 11) becomes the matrix equation
µ ¶ µ
¶µ ¶ µ ¶ µ
¶
1
x y
5
u
5x + 4y + u
=
+
=
(mod 11).
8
zw
4
v
5z + 4w + v
So this gives the two congruences
5x + 4y + u ≡ 1

(mod 11)

and

5z + 4w + v ≡ 8

(mod 11).

Similarly, the congruence c2 = k1 m2 + k2 (mod 11) gives
8x + 10y + u ≡ 8

(mod 11)

and

8z + 10w + v ≡ 5

and

7z + w + v ≡ 7

(mod 11).

and c3 = k1 m3 + k2 (mod 11) gives
7x + y + u ≡ 8

(mod 11)

(mod 11).

This gives us 6 equations for the 6 unknowns x, y, z, w, u, v. Further, three of
the equations only involve x, y, u and the other three only involve z, w, v, so
it’s really two sets of 3-by-3 equations to solve:
5x + 4y + u = 1
8x + 10y + u = 8
7x + y + u = 8

5z + 4w + v = 8
8z + 10w + v = 5
7z + w + v = 7.

(All equations are modulo 11.) These are easily solved using basic linear algebra methods, and we find that
(x, y, u) = (3, 7, 2)
Hence
k1 =

µ ¶
37
43

and

and

(z, w, v) = (4, 3, 9).

k2 =

µ ¶
2
.
9

(d) We work with vectors of dimension 26. Let e1 , . . . , e26 be the usual basis
vectors for R26 , i.e., ei has a 1 in the ith place and 0’s elsewhere. For the
plaintext, we use e1 to represent (a), we use e2 to represent (b), and so
on. We view the the simple substitution cipher as a function that takes each
plaintext letter and assigns it to a ciphertext letter. Equivalently, it takes
each ei and assigns it to some eπ(i) , where π is a one-to-one function
π : {1, 2, . . . , 26} −→ {1, 2, . . . , 26}.
In the Hill cipher, we now take k1 to be the matrix whose ij th entry is 1 if
e(i) = j, and otherwise it is 0. We also take k2 = 0. Then k1 · ei = eπ (i), so
the encryption of the plaintext ei is equal to eπ(i) , as desired.

34

Exercises for Chapter 1

1.43. Let N be a large integer and let K = M = C = Z/N Z. For each of the
functions
e : K × M −→ C
listed in (a), (b), and (c), answer the following questions:
• Is e an encryption function?
• If e is an encryption function, what is its associated decryption function d?
• If e is not an encryption function, can you make it into an encryption
function by using some smaller, yet reasonably large, set of keys?
(a) ek (m) ≡ k − m (mod N ).
(b) ek (m) ≡ k · m (mod N ).
(c) ek (m) ≡ (k + m)2 (mod N ).
Solution to Exercise 1.43.
(a) Yes, e is an encryption function. The decryption function dk (c) = k − c
is the same as e!
(b) No, e is not an encryption function, it is not one-to-one. However, if we
restrict the keys to K = (Z/N Z)∗ (i.e., gcd(k, N ) = 1), then e is an encryption
function, with decryption function dk (c) ≡ k −1 c (mod N ).
(c) No, e is not an encryption function, it is not one-to-one, and no subset of keys will make it√one-to-one. However, one might define a decryption
“function” by dk (c) ≡ c − k (mod N ). Assuming that one knows how to
compute square
√ roots modulo N , this gives two possibly decryptions, since
it’s really ± c. In practice, one might be able to use some property of valid
messages to figure out which one is correct.
1.44. (a) Convert the 12 bit binary number 110101100101 into a decimal
integer between 0 and 212 − 1.
(b) Convert the decimal integer m = 37853 into a binary number.
(c) Convert the decimal integer m = 9487428 into a binary number.
(d) Use exclusive or (XOR) to “add” the bit strings 11001010 ⊕ 10011010.
(e) Convert the decimal numbers 8734 and 5177 into binary numbers, combine them using XOR, and convert the result back into a decimal number.
Solution to Exercise 1.44.
(a) 211 + 210 + 28 + 26 + 25 + 22 + 20 = 3429
(b) 37853 = 215 + 212 + 29 + 28 + 27 + 26 + 24 + 23 + 22 + 20 , so the binary
form of 37853 is 1001001111011101 .
(c) 9487428 = 223 + 220 + 215 + 214 + 210 + 26 + 22 , so the binary form
of 9487428 is 100100001100010001000100 .
(d) 11001010 ⊕ 10011010 = 01010000 .
(e)

Exercises for Chapter 1

35

8734 = ‘10001000011110’,
5177 = ‘01010000111001’,
8734 ⊕ 5177 = 10001000011110 ⊕ 01010000111001 = 11011000100111,
‘11011000100111’ = 13863 .

1.45. Alice and Bob choose a key space K containing 256 keys. Eve builds a
special-purpose computer that can check 10,000,000,000 keys per second.
(a) How many days does it take Eve to check half of the keys in K?
(b) Alice and Bob replace their key space with a larger set containing 2B different keys. How large should Alice and Bob choose B in order to force
Eve’s computer to spend 100 years checking half the keys? (Use the approximation that there are 365.25 days in a year.)
For many years the United States government recommended a symmetric
cipher called DES that used 56 bit keys. During the 1990s, people built special
purpose computers demonstrating that 56 bits provided insufficient security.
A new symmetric cipher called AES, with 128 bit keys, was developed to
replace DES. See Section 8.10 for further information about DES and AES.
Solution to Exercise 1.45.
(a)
µ
(256 keys) ·

¶ µ
¶
1 second
1 minute
·
10,000,000,000 keys
60 seconds
µ
¶ µ
¶
1 hour
1 day
·
·
≈ 83.4 days.
60 minutes
24 hours

It thus takes about 83.4 days to check all the keys, so about 41.7 days to
check half the keys.
(b)
µ

¶ µ
¶ µ
¶
10,000,000,000 keys
60 seconds
60 minutes
·
·
1 second
1 minute
1 hour
µ
¶ µ
¶
24 hours
365.25 days
·
·
· (100 years)
1 day
1 year
= 31557600000000000000 keys ≈ 264.775 keys.

Thus it takes Eve’s computer 100 years to check 264.775 keys. The problem says
that this should be half the keys, so Alice and Bob should have at least 265.775
different keys. In practice, it is easiest to choose an integral power of 2, so Alice
and Bob’s key space should contain (at least) 266 keys.
Comparing (a) and (b), notice that by increasing the keylength from 56 bits
to 66 bits, Alice and Bob’s security goes from 42 days to 100 years. Thus even a

36

Exercises for Chapter 1

small increase in the keylength results in an enormous increase in the breaking
time by exhaustive search. This reflects the fact that exponential functions
grow extremely rapidly.
1.46. Explain why the cipher
ek (m) = k ⊕ m

and

dk (c) = k ⊕ c

defined by XOR of bit strings is not secure against a chosen plaintext attack.
Demonstrate your attack by finding the private key used to encrypt the 16-bit
ciphertext c = 1001010001010111 if you know that the corresponding plaintext
is m = 0010010000101100.
Solution to Exercise 1.46.
If you know m and c, since they are related by c = k ⊕ m, it follows that
c ⊕ m = k ⊕ m ⊕ m = k. For the example,
k = c ⊕ m = 1001010001010111 ⊕ 0010010000101100 = 1011000001111011 .

1.47. Alice and Bob create a symmetric cipher as follows. Their private key k
is a large integer and their messages (plaintexts) are d-digit integers
M = {m ∈ Z : 0 ≤ m < 10d }.
√
To encrypt a message, Alice computes k to d decimal places, throws away
the part to the left of the decimal point, and keeps the remaining d digits.
Let
√ α be this d-digit number. (For example, if k = 23 and d = 6, then
87 = 9.32737905 . . . and α = 327379.)
Alice encrypts a message m as
c≡m+α

(mod 10d ).

Since Bob knows k, he can also find α, and then he decrypts c by computing m ≡ c − α (mod 10d ).
(a) Alice and Bob choose the secret key k = 11 and use it to encrypt 6-digit
integers (i.e., d = 6). Bob wants to send Alice the message m = 328973.
What is the ciphertext that he sends?
(b) Alice and Bob use the secret key k = 23 and use it to encrypt 8-digit
integers. Alice receives the ciphertext c = 78183903. What is the plaintext m?
(c) Show that the number α used for encryption and decryption is given by
the formula
³√
j
√ ´k
α = 10d
k−b kc ,
where btc denotes the greatest integer that is less than or equal to t.

Exercises for Chapter 1

37

(d) (Challenge Problem) If Eve steals a plaintext/ciphertext pair (m, c), then
it is clear that she can recover the number α, since α ≡ c − m (mod 10d ).
If 10d is large compared to k, can she also recover the number k? This
might
√ be useful, for example, if Alice and Bob use some of the other digits
of k to encrypt subsequent messages.
Solution√to Exercise 1.47.
(a) 11 = 3.3166247903 . . . , so α = 316624 and the ciphertext is c =
328973 + 316624 = 645597 .
√
(b) 23 = 4.7958315233127195 . . . , so α = 79583152 and the plaintext is
c = 78183903 − 79583152 = −1399249 ≡ 98600751

(mod 108 ).

(c) The quantity x − bxc gives the fractional part of x, i.e., the part to the
right of the decimal point. The remaining part of the formula simply shifts
the digits d places to the left and then discards everything after the decimal
point.
(d) The answer is yes, Eve should be able to recover k, but probably not
using the tools that we’ve developed so far. Let β = α/10d . Then
√
k =L+β
for some L ∈ Z.
There are two unknowns here, k and L, and all that Eve knows is that they
are both integers. Squaring both sides gives
k = L2 + 2Lβ + β 2 .
Thus there are integers A and B satisfying
β 2 + Aβ + B = 0,
namely A = 2L and B = L2 − k. Of course, Eve doesn’t know A or B, either.
However, there are algorithms based on lattice reduction that are very good at
finding the smallest (quadratic) polynomial with integer coefficients satisfied
by a given decimal number. Using these algorithms, Eve should be able to
find A and B, from which it is easy to recover k as k = 14 A2 − B.
1.48. Bob and Alice use a cryptosystem in which their private key is a (large)
prime k and their plaintexts and ciphertexts are integers. Bob encrypts a
message m by computing the product c = km. Eve intercepts the following
two ciphertexts:
c1 = 12849217045006222,

c2 = 6485880443666222.

Use the gcd method described in Section 1.7.4 to find Bob and Alice’s private
key.
Solution to Exercise 1.48.
We compute
gcd(c1 , c2 ) = 174385766.
This factors as 174385766 = 2 · 87192883 and 87192883 is prime, so it is Bob
and Alice’s key.

Chapter 2

Discrete Logarithms and
Diffie–Hellman
Exercises for Chapter 2
Section. Diffie–Hellman and RSA
2.1. Write a one page essay giving arguments, both pro and con, for the
following assertion:
If the government is able to convince a court that there is a valid
reason for their request, then they should have access to an individual’s private keys (even without the individual’s knowledge),
in the same way that the government is allowed to conduct court
authorized secret wiretaps in cases of suspected criminal activity
or threats to national security.
Based on your arguments, would you support or oppose the government being given this power? How about without court oversight? The idea that all
private keys should be stored at a secure central location and be accessible to
government agencies (with or without suitably stringent legal conditions) is
called key escrow.
Solution to Exercise 2.1.
A solution for this exercise will not be provided.
2.2. Research and write a one to two page essay on the classification of cryptographic algorithms as munitions under ITAR (International Traffic in Arms
Regulations). How does that act define “export”? What are the potential
fines and jail terms for those convicted of violating the Arms Export Control
Act? Would teaching non-classified cryptographic algorithms to a college class
that includes non-US citizens be considered a form of export? How has US
government policy changed from the early 1990s to the present?
39

40

Exercises for Chapter 2

Solution to Exercise 2.2.
Some historical material:
Press Release
Law Professor Sues Federal Government Over Computer Privacy Issues
Federal Civil Rights Action Seeks Injunction Against State Department
And National Security Agency
Cleveland Scholar Attacks Prohibition On Discussing Cryptographic Software With Foreign Students And Colleagues
For Immediate Release
Cleveland, Wednesday, August 7, 1996
A Case Western Reserve University law professor filed suit today in federal
court, challenging government regulations which restrict his ability to teach
a course in computer law. Peter Junger, a twenty-five year veteran of the
law school faculty, will file a federal civil rights action this afternoon in the
United States District Court in Cleveland. The suit names the Department
of State and the secretive National Security Agency, which administer federal
regulations limiting Professor Junger’s ability to teach.
The case involves the International Traffic in Arms Regulations, or ITAR,
federal regulations which restrict the export of military technology. Under the
ITAR, cryptographic computer software, which encodes text to preserve the
privacy of messages on the Internet, is considered a “munition” and subject to
strict export control. The regulations raise significant First Amendment questions by defining “export” to include discussing technical information about
non-classified software with foreign nationals, such as students registered for
Professor Junger’s course.
In recent months, the State Department has sent a series of letters threatening possible criminal action to a Florida man who posted a simple cryptographic algorithm to the ”sci.crypt” Usenet Newsgroup, an Internet site popular with cryptography enthusiasts. These and similar incidents have caused
Professor Junger to limit his discussions of cryptographic material with foreign
colleagues, for fear of violating the ITAR. Penalties for unlicensed disclosure
of cryptographic information are severe: federal law provides ten year prison
terms and One Million Dollar fines for those convicted of violating the Arms
Export Control Act, the legislation under which the ITAR was promulgated.
————————————
Statement by Ambassador David Aaron
US Envoy for Cryptography
RSA Data Security Conference, January 28, 1997
International Views of Key Recovery
These concerns are being heard in Washington. The Administration has
taken the following steps - many based on the direct recommendations of
industry representatives:
First, at the end of last year, jurisdiction for licenses of encryption exports
was transferred from the Department of State to the Department of Commerce. Commercial encryption is no longer treated as a munition and thereby

Exercises for Chapter 2

41

subject to various foreign policy embargoes. We hope this will both speed up
and simplify the tasks of obtaining licenses.
Second, and very important, the Administration will license the export of
encryption products, of any algorithm and any key length, if they incorporate
key recovery.
Third, the Administration will also permit the export, over the next two
years, of 56-bit DES and equivalent encryption products without key recovery
provided exporters make commitments to develop key recovery products. I
am pleased to report that already at least 4 vendors have formally filed key
recovery commitments and several more companies are in the initial stages of
dialogue with the Department of Commerce.
And last, a point which is often lost in the debate, domestic use of key
recovery will be voluntary as announced by the Vice President last October.
All Americans will remain free to use any encryption system in the United
States.
———————————
In 1992, the Software Publishers Association and the State Department
reached an agreement which allows the export of programs containing RSA
Data Security’s RC2 and RC4 algorithms, but only when the key size is set
to 40 bits or less. 40 bits is not very secure, and application of a distributed
attack using standard workstations in a good-size lab can break these in at
most a few days. This theory was demonstrated quite visibly in mid-1995
when two independent groups broke 40-bit keys used in the export version of
the Netscape browser.
Section. The discrete logarithm problem
2.3. Let g be a primitive root for Fp .
(a) Suppose that x = a and x = b are both integer solutions to the congruence
g x ≡ h (mod p). Prove that a ≡ b (mod p − 1). Explain why this implies
that the map (2.1) on page 63 is well-defined.
(b) Prove that logg (h1 h2 ) = logg (h1 ) + logg (h2 )
for all h1 , h2 ∈ F∗p .
n
∗
(c) Prove that logg (h ) = n logg (h)
for all h ∈ Fp and n ∈ Z.
Solution to Exercise 2.3.
(a) We are given that g a ≡ g b (mod p), since they are both congruent
to h. Hence g a−b ≡ 1 (mod p). But g is a primitive root, so its order is p − 1,
which implies that p − 1 divides a − b. Hence a ≡ b (mod p − 1). This means
the logg (h) is well-defined up to adding or subtracting multiples of p − 1, so
the map (2.1) on page 63 is well-defined.
(b) We have
g logg (h1 )+logg (h2 ) = g logg (h1 ) · g logg (h2 )
≡ h1 · h2 (mod p)
≡ g logg (h1 h2 )

(mod p).

42

Exercises for Chapter 2

Hence logg (h1 ) + logg (h2 ) = logg (h1 h2 ), or more precisely, the are congruent
modulo p − 1.
(c) We have
³
´n
n
g n logg (h) = g logg (h) ≡ hn ≡ g logg (h ) (mod p).
Hence n logg (h) = logg (hn ).
2.4. Compute the following discrete logarithms.
(a) log2 (13) for the prime 23, i.e., p = 23, g = 2, and you must solve the
congruence 2x ≡ 13 (mod 23).
(b) log10 (22) for the prime p = 47.
(c) log627 (608) for the prime p = 941. (Hint. Look in the second column of
Table 2.1 on page 64.)
Solution to Exercise 2.4.
(a) log2 (13) = 7 in F23 , since 213 = 128 ≡ 13 (mod 23).
(b) log10 (22) = 11 in F47 .
(c) The table shows that 62718 ≡ 608 (mod 941), so log627 (608) = 18
in F941 .
2.5. Let p be an odd prime and let g be a primitive root modulo p. Prove
that a has a square root modulo p if and only if its discrete logarithm logg (a)
modulo p is even.
Solution to Exercise 2.5.
This solution is taken from the proof of Proposition 3.60.
Let m = logg (a), so a = g m . If m = 2k is even, then g m = g 2k = (g k )2 is
a square.
On the other hand, let m be odd, say m = 2k + 1, and suppose that g m
is a square modulo p, say g m ≡ c2 (mod p). Fermat’s little theorem (Theorem 1.25) tells us that
cp−1 ≡ 1 (mod p).
However, cp−1 (mod p) is also equal to
cp−1 ≡ (c2 )

p−1
2

≡ (g m )

p−1
2

≡ (g 2k+1 )

p−1
2

≡ g k(p−1) · g

p−1
2

(mod p).

Another application of Fermat’s little theorem tells us that
g k(p−1) ≡ (g p−1 )k ≡ 1k ≡ 1 (mod p),
so we find that
g

p−1
2

≡ 1 (mod p).

This contradicts the fact that g is a primitive root, which proves that every
odd power of g is not a square modulo p.
Section. Diffie–Hellman key exchange

Exercises for Chapter 2

43

2.6. Alice and Bob agree to use the prime p = 1373 and the base g = 2 for
a Diffie–Hellman key exchange. Alice sends Bob the value A = 974. Bob asks
your assistance, so you tell him to use the secret exponent b = 871. What
value B should Bob send to Alice, and what is their secret shared value? Can
you figure out Alice’s secret exponent?
Solution to Exercise 2.6.
Bob sends B = g b = 2871 ≡ 805 (mod 1373) to Alice. Their shared value
is Ab = 974871 ≡ 397 (mod 1373). There is no really easy way to determine
Alice’s secret exponent, but with a computer or even a progammable calculator, it does not take long to compute all of the powers of 2 modulo 1373.
(Using the babystep–giantstep method
is even faster, you only need to make
√
two lists of length approximately 1373 = 37.04 . . . . If you do this, you will
find that 25 87 ≡ 974 (mod 1373), so Alice’s secret exponent is 587.
2.7. Let p be a prime and let g be an integer. The Diffie–Hellman Decision
Problem is as follows. Supoose that you are given three numbers A, B, and C,
and suppose that A and B are equal to
A ≡ g a (mod p)

and

B ≡ g b (mod p),

but that you do not necessarily know the values of the exponents a and b.
Determine whether C is equal to g ab (mod p). Notice that this is different
from the Diffie–Hellman problem described on page 67. The Diffie–Hellman
problem asks you to actually compute the value of g ab .
(a) Prove that an algorithm that solves the Diffie–Hellman problem can be
used to solve the Diffie–Hellman decision problem.
(b) Do you think that the Diffie–Hellman decision problem is hard or easy?
Why?
See Exercise 5.35 for a related example in which the decision problem is easy,
but it is believed that the associated computational problem is hard.
Solution to Exercise 2.7.
(a) This is obvious. If you can compute g ab from g, g a , and g b , then you
can simply compare the value of g ab with the value of C and check if they are
equal.
(b) No one currently knows how to solve the Diffie–Hellman decision problem
without solving the Diffie–Hellman computational problem.
Section. The ElGamal public key cryptosystem
2.8. Alice and Bob agree to use the prime p = 1373 and the base g = 2 for
communications using the ElGamal public key cryptosystem.
(a) Alice chooses a = 947 as her private key. What is the value of her public
key A?
(b) Bob chooses b = 716 as his private key, so his public key is
B ≡ 2716 ≡ 469 (mod 1373).

44

Exercises for Chapter 2

Alice encrypts the message m = 583 using the ephemeral key k = 877.
What is the ciphertext (c1 , c2 ) that Alice sends to Bob?
(c) Alice decides to choose a new private key a = 299 with associated public
key A ≡ 2299 ≡ 34 (mod 1373). Bob encrypts a message using Alice’s
public key and sends her the ciphertext (c1 , c2 ) = (661, 1325). Decrypt
the message.
(d) Now Bob chooses a new private key and publishes the associated public
key B = 893. Alice encrypts a message using this public key and sends the
ciphertext (c1 , c2 ) = (693, 793) to Bob. Eve intercepts the transmission.
Help Eve by solving the discrete logarithm problem 2b ≡ 893 (mod 1373)
and using the value of b to decrypt the message.
Solution to Exercise 2.8.
(a) A ≡ 2947 ≡ 177 (mod 1373), so Alice’s public key is A = 177 .
(b) c1 ≡ 2877 ≡ 719 (mod 1373) and c2 ≡ 583 · 469877 ≡ 623 (mod 1373).
Alice sends the ciphertext (c1 , c2 ) = (719, 623) to Bob.
(c) (ca1 )−1 · c2 ≡ (661299 )−1 · 1325 ≡ 645−1 · 1325 ≡ 794 · 1325 ≡ 332
(mod 1373). Thus the plaintext is m=332 . It turns out that the ephemeral
key is k = 566, but Alice does not know this value.
(d) The solution to 2b ≡ 893 (mod 1373) is b = 219 , which is Bob’s private
key. It is now easy to decrypt,
(ca1 )−1 · c2 ≡ (693219 )−1 · 793 ≡ 431−1 · 793 ≡ 532 · 793 ≡ 365

(mod 1373).

Thus Alice’s message to Bob is m = 365 . (The ephemeral key was k = 932.)
2.9. Suppose that an oracle offers to solve the Diffie–Hellman problem for
you. (See page 67 for a description of the Diffie–Hellman problem.) Explain
how you can use the oracle to decrypt messages that have been encrypted
using the ElGamal public key cryptosystem.
Solution to Exercise 2.9.
In the ElGamal PKC, you know Alice’s public key A ≡ g a (mod p) and
you know the ciphertext consisting of the two quantities c1 ≡ g k (mod p) and
c2 ≡ m · Ak (mod p), where k is Bob’s secret ephemeral key. You thus know
the values of g a and g k , so the Diffie–Hellman problem oracle will take those
values and tell you the value of g ak (mod p). But g ak ≡ Ak (mod p), so you
can recover Bob’s plaintext message by computing (g ak )−1 · c2 ≡ m (mod p).
2.10. The exercise describes a public key cryptosystem that requires Bob and
Alice to exchange several messages. We illustrate the system with an example.
Bob and Alice fix a publicly known prime p = 32611, and all of the other
numbers used are private. Alice takes her message m = 11111, chooses a random exponent a = 3589, and sends the number u = ma (mod p) = 15950 to
Bob. Bob chooses a random exponent b = 4037 and sends v = ub (mod p) = 15422
back to Alice. Alice then computes w = v 15619 ≡ 27257 (mod 32611) and

Exercises for Chapter 2

45

sends w = 27257 to Bob. Finally, Bob computes w31883 (mod 32611) and
recovers the value 11111 of Alice’s message.
(a) Explain why this algorithm works. In particular, Alice uses the numbers
a = 3589 and 15619 as exponents. How are they related? Similarly, how
are Bob’s exponents b = 4037 and 31883 related?
(b) Formulate a general version of this cryptosystem, i.e., using variables, and
show that it works in general.
(c) What is the disadvantage of this cryptosystem over ElGamal? (Hint. How
many times must Alice and Bob exchange data?)
(d) Are there any advantages of this cryptosystem over ElGamal? In particular, can Eve break it if she can solve the discrete logarithm problem?
Can Eve break it if she can solve the Diffie–Hellman problem?
Solution to Exercise 2.10.
(a) Alice’s and Bob’s exponents satisfy
3589 · 15619 ≡ 1

(mod 32610)

and

4037 · 31883 ≡ 1 (mod 32610)

The reason why the algorithm works is discussed in the answer to (b).
(b) In the general formulation, a public prime p is fixed. Alice choose a plaintext m mod p and a random exponent a satisfying gcd(a, p − 1) = 1. She send
u ≡ ma

(mod p)

to Bob. Bob chooses a random exponent b satisfying gcd(b, p − 1) = 1, computes
v ≡ ub (mod p),
and send v to Alice. Alice now computes the inverse of a modulo p − 1, i.e.,
she solves ax ≡ 1 (mod p − 1) for x. Let a0 = a−1 mod p − 1. Alice computes
w ≡ va

0

(mod p)

and sends it to Bob. Finally, Bob computes the inverse b0 = b−1 (mod p − 1)
0
and then wb mod p is equal to m.
To see that this last assertion is true, we compute
0

0 0

0 0

0 0

wb ≡ v a b ≡ uba b ≡ maba b

(mod p).

We know that
aa0 ≡ 1

(mod 1) (mod p − 1)

and

bb0 ≡ 1 (mod 1) (mod p − 1),

so the exponent aba0 b0 is congruent to 1 modulo p − 1. Then Fermat’s little
0 0
theorem tells us that maba b ≡ m (mod p).
(c) ElGamal only require Alice to send Bob a single message. This new cryptosystem requires Alice to send Bob two messages and for Bob to send a

46

Exercises for Chapter 2

message back to Alice. So this new system is much more interactive and requires a lot more communication than does ElGamal.
(d) The advantage of this new system is that Alice and Bob reveal somewhat
less information than in ElGamal. Of course, if Eve can solve the DLP, then
since she knows u, v and w, she can solve
w ≡ vx

(mod p)

to recover x = a0 , and then she can recover m, because
0

0

ua ≡ maa ≡ m (mod p).
However, there does not appear to be an easy way for Eve to break the
system if she knows how to solve the Diffie–Hellman Problem. Thus this new
cryptosystem is potentially more secure than ElGamal, if it turns out that
the DHP is easier to solve than the DLP.
Section. An overview of the theory of groups
2.11. The group S3 consists of the following six distinct elements
e, σ, σ 2 , τ, στ, σ 2 τ,
where e is the identity element and multiplication is performed using the rules
σ 3 = e,

τ 2 = 1,

τ σ = σ 2 τ.

Compute the following values in the group S3 :
(a) τ σ 2
(b) τ (στ )
(c) (στ )(στ )
(d) (στ )(σ 2 τ ).
Is S3 a commutative group?
Solution to Exercise 2.11.
(a) τ σ 2 = (τ σ)σ = (σ 2 τ )σ = σ 2 (τ σ) = σ 2 (σ 2 τ ) = σ 4 τ = (σ 3 )στ = στ .
(b) τ (στ ) = (τ σ)τ = (σ 2 τ )τ = σ 2 τ 2 = σ 2 .
(c) (στ )(στ ) = σ(τ σ)τ = σ(σ 2 τ )τ = σ 3 τ 2 = e.
(d) (στ )(σ 2 τ ) = σ(τ σ)στ = σ(σ 2 τ )στ = σ 3 (τ σ)τ = e(σ 2 τ )τ = σ 2 τ 2 = σ 2 .
No, S3 is not a commutative group. For example τ σ = σ 2 τ , which is different
from στ .
2.12. Let G be a group, let d ≥ 1 be an integer, and define a subset of G by
G[d] = {g ∈ G : g d = e}.
(a) Prove that if g is in G[d], then g −1 is in G[d].
(b) Suppose that G is commutative. Prove that if g1 and g2 are in G[d], then
their product g1 ? g2 is in G[d].
(c) Deduce that if G is commutative, then G[d] is a group.
(d) Show by an example that if G is not a commutative group, then G[d]
need not be a group. (Hint. Use Exercise 2.11.)

Exercises for Chapter 2

47

Solution to Exercise 2.12.
(a) For any element h of G and any positive integer n, we have
(h−1 )n ? hn = (h−1 ? h−1 ? · · · ? h−1 ) ? (h ? h ? · · · ? h) = e,
since there are n copies of h−1 to cancel the n copies of h. Thus (h−1 )n is the
inverse of hn , which we can write succintly as (h−1 )n = (hn )−1 . We apply this
with h = g and n = d and use the assumption that g d = e to conclude that
(g −1 )d = (g d )−1 = e−1 = e.
Hence g −1 is in G[d].
(b) We are given that g1d = e and g2d = e. We use the commutativity to
compute
(g1 g2 )d = g1 g2 g1 g2 · · · g1 g2 = g1d g2d = ee = e.
Therefore g1 g2 ∈ G[d].
(c) From (a) and (b), if we start with two elements in G[d], their product
and their inverses are in G[d]. Also clearly e is in G[d]. This gives the first
two axioms, and the third (associativity) is automatic, since it’s true for all
elements in G.
(d) Using the group S3 in Exercise 2.11, we have τ 2 = e and (στ )2 = e. (The
first is true from the description of the group, and the second is true form
part (c) of the exercise.) However, (στ )τ = στ 2 = σ does not satisfy σ 2 = e.
To see why, note that σ 3 = e, so if also σ 2 = e, then we would have e = σ 3 =
(σ 2 )σ = eσ = σ, which is not true.
An alternative solution is to use the group of 2-by-2 matrices with integer
¡
¢
coefficients. The matrix A = ( 01 10 ) satisfies A2 = I and the matrix B = 10 −1
−1
¡
¢
3
satisfies B 2 = I, but AB = 10 −1
−1 actually has order 3, i.e., (AB) = I.
2.13. Let G and H be groups. A function φ : G → H is called a (group)
homomorphism if it satisfies
φ(g1 ? g2 ) = φ(g1 ) ? φ(g2 )

for all g1 , g2 ∈ G.

(Note that the product g1 ? g2 uses the group law in the group G, while the
product φ(g1 ) ? φ(g2 ) uses the group law in the group H.)
(a) Let eG be the identity element of G, let eH be the identity element of H,
and let g ∈ G. Prove that
φ(eG ) = eH

and

φ(g −1 ) = φ(g)−1 .

(b) Let G be a commutative group. Prove that the map φ : G → G defined
by φ(g) = g 2 is a homomorphism. Give an example of a noncommutative
group for which this map is not a homomorphism.
(c) Same question as (b) for the map φ(g) = g −1 .
Solution to Exercise 2.13.
A solution for this exercise is not currently available.

48

Exercises for Chapter 2

2.14. Prove that each of the following maps is a group homomorphism.
(a) The map φ : Z → Z/N Z that sends a ∈ Z to a¡mod N
¢ in Z/N Z.
0
(b) The map φ : R∗ → GL2 (R) defined by φ(a) = a0 a−1
.
(c) The discrete logarithm map logg : F∗p → Z/(p−1)Z, where g is a primitive
root modulo p.
Solution to Exercise 2.14.
A solution for this exercise is not currently available.
2.15. (a) Prove that GL2 (Fp ) is a group.
(b) Show that GL2 (Fp ) is a noncommutative group for every prime p.
(c) Describe GL2 (F2 ) completely. That is, list its elements and describe the
multiplication table.
(d) How many elements are there in the group GL2 (Fp )?
(e) How many elements are there in the group GLn (Fp )?
Solution to Exercise 2.15.
(a) The identity element is the usual matrix ( 10 01 ). The definition of GL2 (Fp )
ensures that every element has an inverse. Finally, the associative law is true
because it’s true in general for matrix multiplication. (But feel free to write
it out explicitly for the product of three 2-by-2 matrices.)
(b) Here’s an example of noncommuting matrices:
µ ¶µ ¶ µ ¶
µ ¶µ ¶ µ ¶
11
11
10
21
10
11
.
=
and
=
12
01
11
11
11
01
(If p = 2, then 2 = 0, but they are still different matrices.)
(c) The group GL2 (F2 ) has 6 elements:
µ ¶
µ ¶
µ ¶
10
11
01
e=
α=
β=
01
10
11
µ ¶
µ ¶
µ ¶
10
11
01
²=
δ=
γ=
11
01
10
They satisfy many relations, for example β = α2 and ² = α2 γ. In fact, we can
get all 6 elements as
e, α, α2 , γ, αγ, α2 γ,
and the group operation is determined by the rules
α3 = e,

γ 2 = e,

γα = α2 γ.

Comparing with Exercise 2.11 we see that GL2 (F2 ) is the same as the group S3
described in that exercise, we’ve just named the generating elements α and γ
instead of σ and τ .
(d) Let α be a matrix in GL2 (Fp ). The first row can be any vector except for
the 0 vector, so there are p2 − 1 possibilities for the first row. The second row

Exercises for Chapter 2

49

can be any vector that is not a scalar multiple of the first row. There are p
possible scalar multiples of the first row, so there are p2 − p possibilities for
the second row. Hence
# GL2 (Fp ) = (p1 − 1)(p2 − p) = (p − 1)2 p(p + 1).
(e) Using the same reasoning as in (d), there are pn − 1 allowable first rows,
then pn − p allowable second rows, then pn − p2 allowable third rows (since
we have to disallow all linear combinations of the first two rows), etc. Hence
# GLn (Fp ) =

n−1
Y

(pn − pi ).

i=0

Section. How hard is the discrete logarithm problem?
2.16. Verify the following assertions from Example 2.17.
¡
¢
¡ ¢
√
(a) x2 + x = O x2 .
(d) (ln k)375 = O k 0.001 .
¡
¢
¡
¢
(b) 5 + 6x2 − 37x5 = O x5 .
(e) k 2 2k = O e2k .
¡ k¢
¡ ¢
300
(c) k
=O 2 .
(f) N 10 2N = O eN .
Solution to Exercise 2.16.
A solution for this exercise is not currently available.
Section. A Collision Algorithm for the DLP
2.17. Use Shanks’s babystep–giantstep method to solve the following discrete
logarithm problems. (For (b) and (c), you may want to write a computer
program implementing Shanks’s algorithm.)
(a) 11x = 21 in F71 .
(b) 156x = 116 in F593 .
(c) 650x = 2213 in F3571 .
Solution to Exercise 2.17.
√
(a) The number 11 has order 70 in F71 . Set N = d 70 e = 9 and H =
h−N = 11−9 = 7. From Table ?? we see that
111 = 21 · 74 = 11

in F71 .

Hence
21 = 111 · 7−4 = 111 · (119 )4 = 1137

in F71 ,

so the solution is x=37 .
√
(b) The number 156 has order 148 in F593 . Set N = d 148 e = 13 and
H = h−N = 156−13 = 297. From Table ?? we see that
1567 = 116 · 2974 = 452

in F593 .

50

Exercises for Chapter 2
hk
11
50
53
15

k
1
2
3
4

a · Hk
5
35
32
11

Table 2.1: Solve 11x ≡ 21 (mod 71) with babystep–giantstep
k
1
2
3
4
5
6
7

hk
156
23
30
529
97
307
452

a · Hk
58
29
311
452
226
113
353

Table 2.2: Solve 156x ≡ 116 (mod 593) via babystep–giantstep

Hence

116 = 1567 · 297−4 = 1567 · (15613 )4 = 15659

in F593 ,

so the solution is x=59 .
√
(c) The number h = 650 has order 510 in F3571 . Set N = d 510 e = 23 and
H = h−N = 650−23 = 1925. Table ?? lists the values of hk and a · H k for
k = 1, 2, . . .. From the table we see that
65020 = 2213 · 192513 = 3011

in F3571 .

Using the fact that 1925 = 650−23 , we compute
2213 = 65020 · 1925−13 = 65020 · (65023 )13 = 650319

in F3571 ,

so the solution is x=319 .
Section. The Chinese remainder theorem
2.18. Solve each of the following simultaneous systems of congruences (or
explain why no solution exists).
k

hk

a · Hk

k

hk

a · Hk

k

hk

a · Hk

k

hk

a · Hk

1
2
3
4
5

650
1122
816
1892
1376

3393
166
1731
432
3128

6
7
8
9
10

1650
1200
1522
133
746

694
396
1677
41
363

11
12
13
14
15

2815
1398
1666
887
1619

2430
3311
3011
442
952

16
17
18
19
20

2476
2450
3405
2801
3011

677
3381
2063
323
421

Table 2.3: Solve 650x ≡ 2213 (mod 3571) using babystep–giantstep

Exercises for Chapter 2
(a)
(b)
(c)
(d)
(e)

51

x ≡ 3 (mod 7) and x ≡ 4 (mod 9).
x ≡ 137 (mod 423) and x ≡ 87 (mod 191).
x ≡ 133 (mod 451) and x ≡ 237 (mod 697).
x ≡ 5 (mod 9), x ≡ 6 (mod 10), and x ≡ 7 (mod 11).
x ≡ 37 (mod 43), x ≡ 22 (mod 49), and x ≡ 18 (mod 71).

Solution to Exercise 2.18.
(a) x ≡ 31 (mod 63).
(b) x ≡ 27209 (mod 80793).
(c) No solution, since gcd(451, 697) = 41 and 133 and 237 are not congruent to one another modulo 41.
(d) x ≡ 986 (mod 990).
(e) x ≡ 11733 (mod 149597).
2.19. Solve the 1700-year-old Chinese remainder problem from the Sun Tzu
Suan Ching stated on page 82.
Solution to Exercise 2.19.
In the modern notation, the solution in the Sun Tzu Suan Ching uses the
fact that:
70 ≡ 1 (mod 3)
21 ≡ 0 (mod 3)
15 ≡ 0 (mod 3)

≡ 0 (mod 5)
≡ 1 (mod 5)
≡ 0 (mod 5)

≡ 0 (mod 7),
≡ 0 (mod 7),
≡ 1 (mod 7).

Hence (2 ∗ 70) + (3 ∗ 21) + (2 ∗ 15) = 233 satisfies the desired congruences.
Since any multiple of 105 is divisible by 3, 5 and 7, we can subtract 2 ∗ 105
from 233 to get 23 as the smallest positive solution.
Problem 26 is the only problem in the Sun Tzu Suan Ching that illustrates the Chinese remainder theorem. Thus it is not known if the author had
developed a general method to solve such problems.
2.20. Let a, b, m, n be integers with gcd(m, n) = 1. Let
c ≡ (b − a) · m−1

(mod n).

Prove that x = a + cn is a solution to
x ≡ a (mod m)

and

x≡b

(mod n),

(2.1)

and that every solution to (2.24) has the form x = a+cn+ymn for some y ∈ Z.
Solution to Exercise 2.20.
A solution for this exercise is not currently available.
2.21. Let x = c and x = c0 be two solutions of the system of simultaneous
congruences (2.7) in the Chinese remainder theorem (Theorem 2.25). Prove
that
c ≡ c0 (mod m1 m2 · · · mk ).

52

Exercises for Chapter 2

Solution to Exercise 2.21.
A solution for this exercise is not currently available.
2.22. For those who have studied ring theory, this exercise sketches a short, albeit nonconstructive, proof of the Chinese remainder theorem. Let m1 , . . . , mk
be integers and let m = m1 m2 · · · mk be their product.
(a) Prove that the map
Z
Z
Z
Z
−−−−→
×
×
mZ
m1 Z m2 Z mk Z
a mod m −−−−→ (a mod m1 , a mod m2 , . . . , a mod mk )

(2.2)

is a well-defined homomorphism of rings. (Hint. First define a homomorphism from Z to the right-hand side of (2.25), and then show that mZ is
in the kernel.)
(b) Assume that m1 , . . . , mk are pairwise relatively prime. Prove that the
map given by (2.25) is one-to-one. (Hint. What is the kernel?)
(c) Continuing with the assumption that the numbers m1 , . . . , mk are pairwise relatively prime, prove that the map (2.25) is onto. (Hint. Use (b)
and count the size of both sides.)
(d) Explain why the Chinese remainder theorem (Theorem 2.25) is equivalent
to the assertion that (b) and (c) are true.
Solution to Exercise 2.22.
A solution for this exercise is not currently available.
2.23. Use the method described in Section 2.8.1 to find square roots modulo
the following composite moduli.
(a) Find a square root of 340 modulo 437. (Note that 437 = 19 · 23.)
(b) Find a square root of 253 modulo 3143.
(c) Find four square roots of 2833 modulo 4189. (The modulus factors as
4189 = 59 · 71. Note that your four square roots should be distinct modulo 4189.)
(d) Find eight square roots of 813 modulo 868.
Solution to Exercise 2.23.
(a) The square roots of 340 modulo 437 are 146, 215, 222, and 291.
(b) The square roots of 253 modulo 3143 are 489, 1387, 1756, 2654. (Note
3143 = 7 · 449 and 449 is prime.)
(c) The square roots of 2833 modulo 4189 are 1002, 1712, 2477, and 3187.
(d) We factor 868 = 4 · 7 · 31. The eight square roots of 813 modulo 868 are
41, 83, 351, 393, 475, 517, 785, and 827.
2.24. Let p be an odd prime and let b be a square root of a modulo p. This
exercise investigates the square root of a modulo powers of p.
(a) Prove that for some choice of k, the number b + kp is a square root of a
modulo p2 , i.e., (b + kp)2 ≡ a (mod p2 ).

Exercises for Chapter 2

53

(b) The number b = 537 is a square root of a = 476 modulo the prime
p = 1291. Use the idea in (a) to compute a square root of 476 modulo p2 .
(c) Suppose that b is a square root of a modulo pn . Prove that for some choice
of j, the number b + jpn is a square root of a modulo pn+1 .
(d) Explain why (c) implies the following statement: If p is an odd prime and
if a has a square root modulo p, then a has a square root modulo pn for
every power of p. Is this true if p = 2?
(e) Use the method in (c) to compute the square root of 3 modulo 133 , given
that 92 ≡ 3 (mod 13).
Solution to Exercise 2.24.
(a),(c),(d) A solution for this exercise is not currently available.
(b) (b + k · p)2 ≡ a (mod p2 ) gives 1074k + 223 ≡ 0 (mod p), and hence
k ≡ 239 (mod p). This gives 309086 as the square root of a modulo p2 .
(e) 9863 is the square root of 3 modulo 133 .
2.25. Suppose n = pq with p and q both primes.
(a) Suppose that gcd(a, pq) = 1. Prove that if the equation x2 ≡ a (mod n)
has any solutions, then it has four solutions.
(b) Suppose you had a machine that could find all four solutions for some
given a. How could you use this machine to factor n?
Solution to Exercise 2.25.
A solution for this exercise is not currently available.
Section. The Pohlig–Hellman algorithm
2.26. Let Fp be a finite field and let N | p − 1. Prove that F∗p has an element
of order N . This is true in particular for any prime power that divides p − 1.
(Hint. Use the fact that F∗p has a primitive root.)
Solution to Exercise 2.26.
Let g be a primitive root. Then g has order p − 1, so h = g (p−1)/N has
order N .
2.27. Write out your own proof that the Pohlig–Hellman algorithm works in
the particular case that p − 1 = q1 · q2 is a product of two distinct primes.
This provides a good opportunity for you to understand how the proof works
and to get a feel for how it was discovered.
Solution to Exercise 2.27.
A solution for this exercise will not be provided.
2.28. Use the Pohlig–Hellman algorithm (Theorem 2.32) to solve the discrete
logarithm problem
g x = a in Fp
in each of the following cases.
(a) p = 433, g = 7, a = 166.

54

Exercises for Chapter 2

(b) p = 746497, g = 10, a = 243278.
(c) p = 41022299, g = 2, a = 39183497. (Hint. p = 2 · 295 + 1.)
(d) p = 1291799, g = 17, a = 192988. (Hint. p − 1 has a factor of 709.)
Solution to Exercise 2.28.
(a) Step 1 is to solve
q
2
3

h = g (p−1)/q
265
374

e
4
3

e

b = a(p−1)/q
250
335

e

y with hy = b
15
20

Step 2 is to solve
x ≡ 15 (mod 24 ),

x ≡ 20 (mod 33 ).

The solution is x=47 .
(b) Step 1 is to solve
q
2
3

e
10
6

h = g (p−1)/q
4168
674719

e

b = a(p−1)/q
38277
322735

e

y with hy = b
523
681

Step 2 is to solve
x ≡ 523 (mod 210 ),

x ≡ 681 (mod 36 ).

The solution is x=223755 .
(c) Step 1 is to solve
q
2
29

e
1
5

h = g (p−1)/q
41022298
4

e

b = a(p−1)/q
1
11844727

e

y with hy = b
0
13192165

In order to solve the discrete logarithm problem modulo 295 , it is best to solve
4
it step by step. Note that 429 = 18794375 is an element of order 29 in F∗p . To
avoid notational confusion, we use the letter u for the exponents.
¡
¢294
First solve 18794375u0 = 11844727
= 987085. The solution is u0 = 7.
The value of u so far is u = 7.
¡
¢293
Solve 18794375u1 = 11844727·4−7
= 8303208. The solution is u1 = 8.
The value of u so far is u = 239 = 7 + 8 · 29.
¡
¢292
Solve 18794375u2 = 11844727 · 4−239
= 30789520. The solution is
u2 = 26. The value of u so far is u = 22105 = 7 + 8 · 29 + 26 · 292 .
¡
¢291
Solve 18794375u3 = 11844727 · 4−22105
= 585477. The solution is
u3 = 18. The value of u so far is u = 461107 = 7 + 8 · 29 + 26 · 292 + 18 · 293 .
¡
¢290
Solve 18794375u4 = 11844727 · 4−461107
= 585477. The solution is
u4 = 18. The final value of u is u = 13192165 = 7 + 8 · 29 + 26 · 292 + 18 · 293 +
18 · 294 , which is the number you see in the last column of the table.

Exercises for Chapter 2

55

Step 2 is to solve
x ≡ 13192165 (mod 295 ).

x ≡ 0 (mod 2),
The solution is x=33703314 .
(d) Step 1 is to solve
q
2
709
911

e
1
1
1

h = g (p−1)/q
1291798
679773
329472

e

b = a(p−1)/q
1
566657
898549

e

y with hy = b
0
322
534

There is no magical way to solve the DLP’s modulo 709 or 911, although
they are easily solved by an exhaustive search on a computer, and a collision
algorithm is even faster. Step 2 is to solve
x ≡ 0 (mod 2),

x ≡ 322 (mod 709),

x ≡ 534 (mod 911).

The solution is x=984414 .
Section. Rings, quotient rings, polynomial rings, and finite fields
2.29. Let R be a ring with the property that the only way that a product a · b
can be 0 is if a = 0 or b = 0. (In the terminology of Example 2.56, the ring R
has no zero divisors.) Suppose further that R has only finitely many elements.
Prove that R is a field. (Hint. Let a ∈ R with a 6= 0. What can you say about
the map R → R defined by b 7→ a · b?)
Solution to Exercise 2.29.
A solution for this exercise is not currently available.
2.30. Let R be a ring. Prove the following properties of R directly from the
ring axioms described in Section 2.10.1.
(a) Prove that the additive identity element 0 ∈ R is unique, i.e., prove that
there is only one element in R satisfying 0+a = a+0 = 0 for every a ∈ R.
(b) Prove that the multiplicative identity element 1 ∈ R is unique.
(c) Prove that every element of R has a unique additive inverse.
(d) Prove that 0 ? a = a ? 0 = 0 for all a ∈ R.
(e) We denote the additive inverse of a by −a. Prove that −(−a) = a.
(f) Let −1 be the additive inverse of the multiplicative identity element 1 ∈
R. Prove that (−1) ? (−1) = 1.
(g) Prove that b | 0 for every nonzero b ∈ R.
(h) Prove that an element of R has at most one multiplicative inverse.
Solution to Exercise 2.30.
(a) If 0 and 00 are both additive identities, then
00 = 00 + 0 = 0.

56

Exercises for Chapter 2

(b) If 1 and 10 are both multiplicative identities, then
10 = 10 ? 1 = 1.
(c) If b and c are both additive inverses of a, then
b = b + 0 = b + (c + b0 ) = (b + c) + b0 = 0 + b0 = b0 .
(d)
0 ? a = (0 + 0) ? a = (0 ? a) + (0 ? a).
Subtracting 0 ? a from both sides give 0 ? a = 0. (Note “subtraction” really
means to add the additive inverse.)
(e) Let b = −(−a). Then by definition, b + (−a) = 0. But we also know by
definition that a + (−a) = 0. Since additive inverses are unique from (c), it
follows that b = a.
(f) To ease notation, we let i = 1 and u = −1. Then
0 = 0 ? u = (i + u) ? u = (i ? u) + (u ? u) = u + (u ? u).
Thus u?u is the additive inverse of u. Using (e) gives (−1)?(−1) = −(−1) = 1.
(g) We have b ? 0 = 0 from (d), so b | 0 by definition of divisibility.
(h) Let a ∈ R and suppose that ab = 1 and ac = 1, so b and c are both
multiplicative inverses of a. Then
b = b · 1 = b · (a · c) = (a · b) · c = 1 · c = c.
Thus b = c, so a has at most one multiplicative inverse.
2.31. Prove Proposition 2.42.
Solution to Exercise 2.31.
A solution for this exercise is not currently available.
2.32. Prove Proposition 2.44. (Hint. First use Exercise 2.31 to prove that the
congruence classes a + b and a ? b depend only on the congruence classes of a
and b.)
Solution to Exercise 2.32.
A solution for this exercise is not currently available.
2.33. Let F be a field and let a and b be nonzero polynomials in F[x].
(a) Prove that deg(a · b) = deg(a) + deg(b).
(b) Prove that a has a multiplicative inverse in F[x] if and only if a is in F,
i.e., if and only if a is a constant polynomial.
(c) Prove that every nonzero element of F[x] can be factored into a product of
irreducible polynomials. (Hint. Use (a), (b), and induction on the degree
of the polynomial.)

Exercises for Chapter 2

57

(d) Let R be the ring Z/6Z. Give an example to show that (a) is false for
some polynomials a and b in R[x].
Solution to Exercise 2.33.
(a) A solution for this exercise is not currently available.
(b) If a · b = 1, then taking degrees and using (a) gives
0 = deg(1) = deg(a · b) = deg(a) + deg(b).
The degree of a nonzero polynomial is a nonnegative integer, so we conclude
that deg(a) = deg(b) = 0. Hence a and b are constant polynomials.
(c) Polynomials of degree 0 and 1 are already irreducible. Suppose we know
that every polynomial of degree smaller than n can be factored into a product
of irreducible polynomials, and let a ∈ F[x] have degree n. If a is itself irreducible, we’re done. Otherwise it factors as a = b · c, where neither b nor c
is a unit. It follows from (b) that b and c both have degree at least 1, so
using (a) we find that b and c have degrees that are strictly smaller than the
degree of a. Hence by induction, both b and c can be factored as a product
of irreducible polynomials. But then their product, which equals a, is also a
product of irreducible polynomials.
(d) Let a = 2x + 1 and bf b = 3x + 1, then a · b = 6x2 + 5x + 1 = 5x + 1,
since 6 = 0 in Z/6Z. Hence
deg(a) = deg(b) = deg(a · b) = 1,
so the degree formula in (a) is false.
2.34. Let a and b be the polynomials
a = x5 + 3x4 − 5x3 − 3x2 + 2x + 2,
b = x5 + x4 − 2x3 + 4x2 + x + 5.
Use the Euclidean algorithm to compute gcd(a, b) in each of the following
rings.
(a) F2 [x]
(b) F3 [x]
(c) F5 [x]
(d) F7 [x].
Solution to Exercise 2.34.
(a) gcdF2 [x] (a, b) = x3 + x2 + x + 1.
(b) gcdF3 [x] (a, b) = x2 + x + 2.
(c) gcdF5 [x] (a, b) = x + 4.
(d) gcdF7 [x] (a, b) = 1.
(Note for instructor: The resultant of a and b is −23 · 32 · 5 · 59 · 107, so
gcd(a, b) = 1 in Fp [x] unless p ∈ {2, 3, 5, 59, 107}.)
2.35. Continuing with the same polynomials a and b as in Exercise 2.34,
for each of the polynomial rings (a), (b), (c), and (d) in Exercise 2.34, find
polynomials u and v satisfying
a · u + b · v = gcd(a, b).

58

Exercises for Chapter 2

Solution to Exercise 2.35.
(a) u = 1 and v = 1.
(b) u = x + 1 and v = 2x.
(c) u = 3x3 + 4x2 + x + 2 and v = 2x3 + x.
(d) u = 3x4 + 3x3 + x2 + 5x + 4 and v = 4x4 + 5x3 + x2 + 2x.
2.36. Prove that the polynomial x3 + x + 1 is irreducible in F2 [x]. (Hint.
Think about what a factorization would have to look like.)
Solution to Exercise 2.36.
If x3 + x + 1 factors, then it can be written as the product of a linear
polynomial and a quadratic polynomial. Since the only possible coefficients
are 0 and 1, this means we would have
x3 + x + 1 = (x + a)(x2 + bx + c)

in F2 [x].

Putting x = 0 yields 1 = ac, so we must have a = c = 1. (Remember that a
and c are in F2 , so they are either 0 or 1.) Now we have
x3 + x + 1 = (x + 1)(x2 + bx + 1),
and putting x = 1 yields 1 = 2 · (2 + b) = 0. This contradiction shows
that x3 + x + 1 does not factor in F2 [x].
2.37. The multiplication table for the field F2 [x]/(x3 + x + 1) is given in
Table 2.5, but we have omitted fourteen entries. Fill in the missing entries.
(This is the field described in Example 2.58. You can download and print
a copy of Table 2.5 at www.math.brown.edu/~jhs/MathCrypto/Table2.5.
pdf.)
Solution to Exercise 2.37.
Note that it’s not necessary to compute both a · b and b · a. Half missing
entries in the table are
1 · x2 = x2
x · (x2 + x) = x2 + x + 1
x2 · x = x + 1
(x + 1) · 1 = x + 1
(x2 + 1) · (x + 1) = x2
(x2 + x) · (x2 + x + 1) = x2
(x2 + x + 1) · (x2 + 1) = x2 + x.

The other half are the same products in the opposite order.

Exercises for Chapter 2
x

x2

1+x

1 + x2

x + x2

0

0

0

0

1

x
x2

0
1 + x2

0
x + x2

0

1

0

0

1

0

x

x

0

x2

0

1+x
1 + x2

0

0

1 + x2
x + x2

1

x + x2
1 + x + x2

0

1 + x + x2

1 + x2

0

59

x + x2
x + x2

1 + x + x2

x + x2
1 + x + x2
1 + x2

x
1 + x2

1

1

x

1
x

1 + x + x2
0
1 + x + x2
1 + x2

1 + x2

1

1

x

1 + x + x2

1+x

1+x

x
1+x

Table 2.4: Multiplication table for the field F2 [x]/(x3 + x + 1)

2.38. The field F7 [x]/(x2 + 1) is a field with 49 elements, which for the moment we denote by F49 . (See Example 2.59 for a convenient way to work
with F49 .)
(a) Is 2 + 5x a primitive root in F49 ?
(b) Is 2 + x a primitive root in F49 ?
(c) Is 1 + x a primitive root in F49 ?
(Hint. Lagrange’s theorem says that the order of u ∈ F49 must divide 48. So
if uk 6= 1 for all proper divisors k of 48, then u is a primitive root.)
Solution to Exercise 2.38.
(a) No, (2 + x)8 = 1.
(b) Yes. It suffices to check that (2 + x)16 = 4 and (2 + x)24 = 6 are not equal
to 1.
(c) No, (1 + x)24 = 1.
2.39. Let p be a prime number and let e ≥ 2. The quotient ring Z/pe Z and
the finite field Fpe are both rings and both have the same number of elements.
Describe some ways in which they are intrinsically different.
Solution to Exercise 2.39.
Every nonzero element in the field Fpe has a multiplicative inverse,
while Z/(pe ) has lots of elements that do not have inverses, for example all elements of the form kp with 1 ≤ k < pe−1 . In the field Fpe , if a product ab = 0,
then either a = 0 or b = 0. (To see this, note that if a 6= 0, then a−1 exists, so
multiplying ab = 0 by a−1 shows that b = 0.) On the other hand, Z/(pe ) does
not have this property. For example, p · pe−1 = 0, but neither p nor pe−1 is 0
e
in Z/(pe ). A subtler property is that every element α of Fpe satisfies αp = α,
e
but this is not true in Z/(pe ). For example, if we take α = p, the αp = 0.
2.40. Let F be a finite field.
(a) Prove that there is an integer m ≥ 1 such that if we add 1 to itself m
times,
1 + 1 + · · · + 1,
|
{z
}
m ones

60

Exercises for Chapter 2

then we get 0. Note that here 1 and 0 are the multiplicative and additive
identity elements of the field F. If the notation is confusing, you can let u
and z be the multiplicative and additive identity elements of F, and then
you need to prove that u + u + · · · + u = z. (Hint. Since F is finite, the
numbers 1, 1 + 1, 1 + 1 + 1,. . . cannot all be different.)
(b) Let m be the smallest positive integer with the property described in (a).
Prove that m is prime. (Hint. If m factors, show that there are nonzero
elements in F whose product is zero, so F cannot be a field.) This prime
is called the characteristic of the field F.
(c) Let p be the characteristic of F. Prove that F is a finite-dimensional vector
space over the field Fp of p elements.
(d) Use (c) to deduce that F has pd elements for some d ≥ 1.
Solution to Exercise 2.40.
(a) The fact that F is finite means that when we look at
1,

1 + 1,

1 + 1 + 1,

1 + 1 + 1 + 1, . . .

eventually we get a repeated value. Subtracting the smaller number of terms
from the larger, it follows that some sum of 1’s is equal to 0 in F.
(b) Suppose that m factors as m = qr. Then we have
1 + 1 + · · · + 1 · 1 + 1 + · · · + 1 = 1 + 1 + · · · + 1 = 0.
|
{z
} |
{z
} |
{z
}
q ones

r ones

m ones

Since F is a field, the only way for a product to be 0 is for one of the factors
to be 0, so we have either
1 + 1 + ··· + 1 = 0
|
{z
}

or

q ones

1 + 1 + ··· + 1 = 0
|
{z
}

in F.

r ones

But we defined m to be the smallest number of 1’s that sums to 0, so either q ≥
m or r ≥ m. Since we also have m = qr, it follows that either q = m (and
r = 1) or r = m (and q = 1). This proves that m is prime.
(c) It follows that we have a copy of Fp inside F by sending 1 to 1 and
1 + 1 to 1 + 1, etc. The axioms for a field show that this makes F into a
vector space using Fp as scalars. By standard linear algebra, F has a basis as
a vector space over Fp , and the basis is finite since F itself is finite. Hence F
is a finite-dimensional vector space over Fp .
(d) Let v1 , . . . , vd be a basis for F as a vector space over Fp . Then every
element of F can be written uniquely as
a1 v1 + a2 v2 + · · · + ad vd

with a1 , . . . , ad ∈ Fp .

There are p choices of a1 , and p choices of a2 , and p choices of a3 , etc. So
there are pd distinct elements in F.

Chapter 3

Integer Factorization and
RSA
Exercises for Chapter 3
Section. Euler’s theorem and roots modulo pq
3.1. Solve the following congruences.
(a) x19 ≡ 36 (mod 97).
(b) x137 ≡ 428 (mod 541).
(c) x73 ≡ 614 (mod 1159).
(d) x751 ≡ 677 (mod 8023).
(e) x38993 ≡ 328047 (mod 401227). (Hint. 401227 = 607 · 661.)
Solution to Exercise 3.1.
(a) 97 is prime. The congruence 19d ≡ 1 (mod 96) has solution d ≡ 91
(mod 96). Then x ≡ 3691 ≡ 36 (mod 97).
(b) 541 is prime. The congruence 137d ≡ 1 (mod 540) has solution d ≡ 473
(mod 540). Then x ≡ 428473 ≡ 213 (mod 541).
(c) 1159 = 19 · 61 and 18 · 60 = 1080. The congruence 73d ≡ 1 (mod 1080)
has solution d ≡ 577 (mod 1080). Then x ≡ 614577 ≡ 158 (mod 1159).
More efficiently, g = gcd(18, 60) = 6 and (18)(60)/6 = 180. The congruence
73d ≡ 1 (mod 180) has solution d ≡ 37 (mod 180). Then x ≡ 61437 ≡ 158
(mod 1159).
(d) 8023 = 71 · 113 and 71 · 112 = 7840. The congruence 751d ≡ 1
(mod 7840) has solution d ≡ 7151 (mod 7840). Then x ≡ 6777151 ≡ 1355
(mod 8023). More efficiently, g = gcd(70, 112) = 14 and (70)(112)/14 = 560.
The congruence 751d ≡ 1 (mod 560) has solution d ≡ 431 (mod 560). Then
x ≡ 677431 ≡ 1355 (mod 8023).
(e) 401227 = 607 · 661 and 608 · 660 = 399960. The congruence 38993d ≡
1 (mod 399960) has the solution d ≡ 265457 (mod 399960). Then x ≡
61

62

Exercises for Chapter 3

328047265457 ≡ 36219 (mod 401227). More efficiently, g = gcd(606, 660) = 6
and (606)(660)/6 = 66660. The congruence 38993d ≡ 1 (mod 66660) has
the solution d ≡ 65477 (mod 66660). Then x ≡ 32804765477 ≡ 36219
(mod 401227).
3.2. Let p and q be distinct primes and let e and d be integers satisfying
de ≡ 1 (mod (p − 1)(q − 1)).
Suppose further that c is an integer with gcd(c, pq) > 1. Prove that
x ≡ cd (mod pq) is a solution to the congruence

xe ≡ c (mod pq),

thereby completing the proof of Proposition 3.4.
Solution to Exercise 3.2.
If pq | c, then the solution is x = 0. So the interesting case is when c is
divisible by exactly one of p and q, say p | c and q - c. Then x ≡ cd ≡ 0 (mod p)
is a solution to xe ≡ c ≡ 0 (mod p), so we only need to check that it is true
modulo q. We compute
(cd )e ≡ c1+k(p−1)(q−1) ≡ c · (cq−1 )k(p−1) ≡ c (mod q),
since cq−1 ≡ 1 (mod q) from Fermat’s little theorem.
3.3. Recall from Section 1.3 that Euler’s phi function φ(N ) is the function
defined by
φ(N ) = #{0 ≤ k < N : gcd(k, N ) = 1}.
In other words, φ(N ) is the number of integers between 0 and N − 1 that are
relatively prime to N , or equivalently, the number of elements in Z/N Z that
have inverses modulo N .
(a) Compute the values of φ(6), φ(9), φ(15), and φ(17).
(b) If p is prime, what is the value of φ(p)?
(c) Prove Euler’s formula
aφ(N ) ≡ 1

(mod N )

for all integers a satisfying gcd(a, N ) = 1.

(Hint. Mimic the proof of Fermat’s little theorem (Theorem 1.25), but
instead of looking at all of the multiples of a as was done in (1.8), just
take the multiples ka of a for values of k satisfying gcd(k, N ) = 1.)
Solution to Exercise 3.3.
A solution for this exercise is not currently available.
3.4. Euler’s phi function has many beautiful properties.
(a) If p and q are distinct primes, how is φ(pq) related to φ(p) and φ(q)?

Exercises for Chapter 3

63

(b) If p is prime, what is the value of φ(p2 )? How about φ(pj )? Prove that
your formula for φ(pj ) is correct. (Hint. Among the numbers between 0
and pj − 1, remove the ones that have a factor of p. The ones that are
left are relatively prime to p.)
(c) Let M and N be integers satisfying gcd(M, N ) = 1. Prove the multiplication formula
φ(M N ) = φ(M )φ(N ).
(d) Let p1 , p2 , . . . , pr be the distinct primes that divide N . Use your results
from (b) and (c) to prove the following formula:
φ(N ) = N

r µ
Y

1−

i=1

1
pi

¶
.

(e) Use the formula in (d) to compute the following values of φ(N ).
(i) φ(1728). (ii) φ(1575). (iii) φ(889056) (Hint. 889056 = 25 · 34 · 73 ).
Solution to Exercise 3.4.
(a)–(d) A solution for this exercise is not currently available.
(e) (i) φ(1728) = 576, (ii) φ(1575) = 720, (iii) φ(889056) = 254016.
3.5. Let¡N , c, and
¢ e be positive integers satisfying the conditions gcd(N, c) = 1
and gcd e, φ(N ) = 1.
(a) Explain how to solve the congruence
xe ≡ c (mod N ),
assuming that you know the value of φ(N ). (Hint. Use the formula in
Exercise 3.3(c).)
(b) Solve the following congruences. (The formula in Exercise 3.4(d) may be
helpful for computing the value of φ(N ).)
(i) x577 ≡ 60 (mod 1463).
(ii) x959 ≡ 1583 (mod 1625).
(iii) x133957 ≡ 224689 (mod 2134440).
Solution to Exercise 3.5.
(a) A solution for this exercise is not currently available.
(b) (i) N = 7 · 11 · 19, so
¶µ
¶µ
¶
µ
1
1
1
1−
1−
= 1080.
φ(1463) = 1463 1 −
7
11
19
We compute d ≡ 577−1 ≡ 73 (mod 1080), so
x ≡ 6073 ≡ 1390

(mod 1463).

64

Exercises for Chapter 3

Check: 1390577 ≡ 60 (mod 1463). X
(ii) N = 53 · 13, so
¶µ
¶
µ
1
1
φ(1625) = 1625 1 −
1−
= 1200.
5
13
We compute d ≡ 959−1 ≡ 239 (mod 1200), so
x ≡ 1583239 ≡ 147

(mod 1625).

Check: 147959 ≡ 1583 (mod 1625). X
(iii) N = 23 · 32 · 5 · 72 · 112 , so
µ
¶µ
¶µ
¶µ
¶µ
¶
1
1
1
1
1
φ(2134440) = 2134440 1 −
1−
1−
1−
1−
2
3
5
7
11
= 443520.
We compute d ≡ 133957−1 ≡ 326413 (mod 443520), so
x ≡ 224689326413 ≡ 1892929

(mod 2134440).

Check: 1892929133957 ≡ 224689 (mod 2134440). X
Section. The RSA public key cryptosystem
3.6. Alice publishes her RSA public key: modulus N = 2038667 and exponent
e = 103.
(a) Bob wants to send Alice the message m = 892383. What ciphertext does
Bob send to Alice?
(b) Alice knows that her modulus factors into a product of two primes, one
of which is p = 1301. Find a decryption exponent d for Alice.
(c) Alice receives the ciphertext c = 317730 from Bob. Decrypt the message.
Solution to Exercise 3.6.
(a) Bob sends c = me = 892383103 ≡ 45293 (mod 2038667).
(b) The modulus is N = 2038667 = 1301 · 1567, so φ(N ) = 1300 · 1568 =
2035800. A decryption exponent is given by a solution to
103d ≡ 1

(mod 2035800).

The solution is d ≡ 810367 (mod 2035800).
(c) Alice needs to solve
m103 ≡ 317730 (mod 2038667).
Raising both sides to the dth power, where d = 810367 is her decryption
exponent, yields
m ≡ 317730810367 ≡ 514407

(mod 2038667).

Exercises for Chapter 3

65

3.7. Bob’s RSA public key has modulus N = 12191 and exponent e = 37.
Alice sends Bob the ciphertext c = 587. Unfortunately, Bob has chosen too
small a modulus. Help Eve by factoring N and decrypting Alice’s message.
(Hint. N has a factor smaller than 100.)
Solution to Exercise 3.7.
The modulus factors as N = 12191 = 73 · 167, so φ(N ) = 72 · 168 = 11952.
The congruence
37d ≡ 1 (mod 11952)
has solution d ≡ 11629 (mod 11952). Then
m ≡ 58711629 ≡ 4894

(mod 12191)

is a solution to m37 ≡ 587 (mod 12191).
It is possible to be a bit more efficient, using the fact that g = gcd(72, 166) =
2 and (72)(166)/2 = 5976. Thus a solution to the congruence
37d ≡ 1 (mod 5976)
is a decryption exponent, giving the smaller decryption exponent d ≡ 5653
(mod 5976). Of course, this gives the same plaintext
m ≡ 5875653 ≡ 4894

(mod 12191).

3.8. For each of the given values of N = pq and (p − 1)(q − 1), use the method
described in Remark 3.10 to determine p and q.
(a) N = pq = 352717
and (p − 1)(q − 1) = 351520.
(b) N = pq = 77083921
and (p − 1)(q − 1) = 77066212.
(c) N = pq = 109404161
and (p − 1)(q − 1) = 109380612.
(d) N = pq = 172205490419 and (p − 1)(q − 1) = 172204660344.
Solution to Exercise 3.8.
(a) Suppose that N = pq = 352717 and (p − 1)(q − 1) = 351520. Then
p + q = N + 1 − (p − 1)(q − 1) = 1198, so
X 2 − (p + q)X + N = X 2 − 1198X + 352717 = (X − 677)(X − 521).
Hence N = 352717 = 677 · 521.
(b) Suppose that N = pq = 77083921 and (p − 1)(q − 1) = 77066212. Then
p + q = N + 1 − (p − 1)(q − 1) = 17710, so
X 2 − (p + q)X + N = X 2 − 17710X + 77083921 = (X − 10007)(X − 7703).
Hence N = 77083921 = 10007 · 7703.
(c) Suppose that N = pq = 109404161 and (p − 1)(q − 1) = 109380612. Then
p + q = N + 1 − (p − 1)(q − 1) = 23550, so

66

Exercises for Chapter 3

X 2 − (p + q)X + N = X 2 − 23550X + 109404161 = (X − 6367)(X − 17183).
Hence N = 109404161 = 6367 · 17183.
(d) Suppose that N = pq = 172205490419 and (p − 1)(q − 1) = 172204660344.
Then p + q = N + 1 − (p − 1)(q − 1) = 830076, so
X 2 −(p+q)X+N = X 2 −830076X+172205490419 = (X−407893)(X−422183).
Hence N = 172205490419 = 407893 · 422183.
3.9. A decryption exponent for an RSA public key (N, e) is an integer d with
the property that ade ≡ a (mod N ) for all integers a that are relatively prime
to N .
(a) Suppose that Eve has a magic box that creates decryption exponents
for (N, e) for a fixed modulus N and for a large number of different
encryption exponents e. Explain how Eve can use her magic box to try
to factor N .
(b) Let N = 38749709. Eve’s magic box tells her that the encryption exponent e = 10988423 has decryption exponent d = 16784693 and
that the encryption exponent e = 25910155 has decryption exponent
d = 11514115. Use this information to factor N .
(c) Let N = 225022969. Eve’s magic box tells her the following three encryption/decryption pairs for N :
(70583995, 4911157),

(173111957, 7346999),

(180311381, 29597249).

Use this information to factor N .
(d) Let N = 1291233941. Eve’s magic box tells her the following three encryption/decryption pairs for N :
(1103927639, 76923209),

(1022313977, 106791263),

(387632407, 7764043).

Use this information to factor N .
Solution to Exercise 3.9.
Let e1 , e2 , . . . , en be a bunch of random encryption exponents, and suppose
that Eve uses her magic box to create decryption exponents d1 , d2 , . . . , dn .
The numbers K with the property that aK ≡ a (mod N ) for all a satisfying
gcd(a, N ) = 1 are numbers satisfying
µ
¶
(p − 1)(q − 1)
K ≡ 1 mod
.
gcd(p − 1, q − 1)
Thus di ei − 1 is a multiple of (p − 1)(q − 1)/ gcd(p − 1, q − 1) for all 1 ≤ i ≤ n.
Assuming that the ei ’s are reasonably random, Eve will find that
T = gcd(d1 e1 − 1, d2 e2 − 1, d3 e3 − 1, . . . , dn en − 1)

(3.1)

Exercises for Chapter 3

67

is equal to a small multiple of
(p − 1)(q − 1)
.
gcd(p − 1, q − 1)
Next Eve uses the fact that gcd(p − 1, q − 1) is even and tends to be fairly
small. So she first assumes that T = (p − 1)(q − 1)/2 and uses this to compute
R = N + 1 − (p − 1)(q − 1) = N + 1 − 2T . If she is right about the value of T ,
then R will equal p+q, and she can recover p and q by factoring x2 −T x+N . If
this doesn’t work, she repeats the process with R = N +1−3T , R = N +1−4T ,
etc. Continuing in this fashion, she should recover p and q fairly quickly.
Eve can save a bit of time in finding the right multiple of T . The idea is
that N + 1 − kT should equal p + q, and in practice p and q will have
√ more or
less the same order of magnitude. So Eve wants
N + 1 − kT ≈ 2 N , which
√
means that she should take k ≈ (N + 1 − 2 N )/T .
(b)
gcd(16784693 · 10988423 − 1,11514115 · 25910155 − 1)
= gcd(184437306609138, 298332504337824)
= 19368558
First Eve tries N + 1 − 1 · gcd = 19381152, but x2 − 19381152x + 38749709
is irreducible. Next she tries N + 1 − 2 · gcd = 12594, and this time she finds
that x2 − 12594x + 38749709 = (x − 7247)(x − 5347). Hence N = 38749709 =
7247 · 5347.
(c)
gcd(4911157 · 70583995 − 1, 7346999 · 173111957 − 1,
29597249 · 180311381 − 1)
= gcd(346649081132214, 1271853374967042, 5336720840990868)
= 37498566
√
Eve computes ( 225022969 − 1)2 /37498566 ≈ 6.00004193, which suggests
that she should try N + 1 − 6 · gcd = 31574. This given
x2 − 31574x + 225022969 = (x − 20707)(x − 10867).
Hence N = 225022969 = 20707 · 10867.
(d)
gcd(76923209 · 1103927639 − 1, 106791263 · 1022313977 − 1,
7764043 · 387632407 − 1)
= gcd(84917656495673550, 109174200786382950, 3009594676141500)
= 129112350

68

Exercises for Chapter 3

√
Eve computes ( 1291233941 − 1)2 /129112350 ≈ 10.0002987, which suggests
that she should use N + 1 − 10 · gcd = 110442. This yields
x2 − 110442x + 1291233941 = (x − 97151)(x − 13291).
Hence N = 1291233941 = 97151 · 13291.
3.10. Here is an example of a public key system that was proposed at a
cryptography conference. It is supposed to be faster and more efficient than
RSA.
Alice chooses two large primes p and q and she publishes N = pq. It is assumed that N is hard to factor. Alice also chooses three random numbers g, r1 ,
and r2 modulo N and computes
g1 ≡ g r1 (p−1)

(mod N )

and

g2 ≡ g r2 (q−1)

(mod N ).

Her public key is the triple (N, g1 , g2 ) and her private key is the pair of
primes (p, q).
Now Bob wants to send the message m to Alice, where m is a number
modulo N . He chooses two random integers s1 and s2 modulo N and computes
c1 =≡ mg1s1

(mod N )

and

c2 ≡ mg2s2

(mod N ).

Bob sends the ciphertext (c1 , c2 ) to Alice.
Decryption is extremely fast and easy. Alice use the Chinese remainder
theorem to solve the pair of congruences
x ≡ c1

(mod p)

and

x ≡ c2

(mod q).

(a) Prove that Alice’s solution x is equal to Bob’s plaintext m.
(b) Explain why this cryptosystem is not secure.
Solution to Exercise 3.10.
(a) Notice that
c1 ≡ mg1s1 ≡ mg s1 r1 (p−1) ≡ m (mod p)
by Fermat’s little theorem, and similarly c2 ≡ m (mod q). Hence Alice’s
solutions satisfies x ≡ m (mod pq).
(b) As in (a), we observe that g1 ≡ 1 (mod p) from Fermat’s little theorem.
On the other hand, most likely g1 6≡ 1 (mod q). So Eve can recover p from
the trivial gcd computation
gcd(g1 − 1, N ) = p.
(If, by some rare coincidence, g1 ≡ 1 (mod q), then c1 ≡ m (mod N ), so
although Eve cannot factor N , she can read Bob’s message.)
Section. Implementation and security issues

Exercises for Chapter 3

69

3.11. Formulate a man-in-the-middle attack, similar to the attack described
in Example 3.12 on page 122, for the following public key cryptosystems.
(a) The ElGamal public key cryptosystem (Table 2.3 on page 70).
(b) The RSA public key cryptosystem (Table 3.1 on page 119).
Solution to Exercise 3.11.
A solution for this exercise is not currently available.
3.12. Alice decides to use RSA with the public key N = 1889570071. In
order to guard against transmission errors, Alice has Bob encrypt his message
twice, once using the encryption exponent e1 = 1021763679 and once using
the encryption exponent e2 = 519424709. Eve intercepts the two encrypted
messages
c1 = 1244183534 and c2 = 732959706.
Assuming that Eve also knows N and the two encryption exponents e1 and e2 ,
use the method described in Example 3.14 to help Eve recover Bob’s plaintext
without finding a factorization of N .
Solution to Exercise 3.12.
With notation as in Example 3.14, we find that
u · c1 + v · c2 = 1
with
u = 252426389

and

v = −496549570.

Then the plaintext is
m ≡ cu1 · cv2 ≡ 1054592380 (mod N ).
Section. Primality testing
3.13. We stated that the number 561 is a Carmichael number, but we never
checked that a561 ≡ a (mod 561) for every value of a.
(a) The number 561 factors as 3 · 11 · 17. First use Fermat’s little theorem to
prove that
a561 ≡ a (mod 3),

a561 ≡ a (mod 11),

and

a561 ≡ a (mod 17)

for every value of a. Then explain why these three congruences imply that
a561 ≡ a (mod 561) for every value of a.
(b) Mimic the idea used in (a) to prove that each of the following numbers is
a Carmichael number. (To assist you, we have factored each number into
primes.)
(i) 1729 = 7 · 13 · 19
(ii) 10585 = 5 · 29 · 73

70

Exercises for Chapter 3
(iii) 75361 = 11 · 13 · 17 · 31
(iv) 1024651 = 19 · 199 · 271

(c) Prove that a Carmichael number must be odd.
(d) Prove that a Carmichael number must be a product of distinct primes.
(e) Look up Korselt’s criterion in a book or online, write a brief description of
how it works, and use it to show that 29341 = 13·37·61 and 172947529 =
307 · 613 · 919 are Carmichael numbers.
Solution to Exercise 3.13.
A solution for this exercise is not currently available.
Here is a list of all Carmichael up to 100000, plus a few others.
• 561 = 3 · 11 · 17
• 1105 = 5 · 13 · 17
• 1729 = 7 · 13 · 19
• 2465 = 5 · 17 · 29
• 2821 = 7 · 13 · 31
• 6601 = 7 · 23 · 41
• 8911 = 7 · 19 · 67
• 10585 = 5 · 29 · 73
• 15841 = 7 · 31 · 73
• 29341 = 13 · 37 · 61
• 41041 = 7 · 11 · 13 · 41
• 46657 = 13 · 37 · 97
• 52633 = 7 · 73 · 103
• 62745 = 3 · 5 · 47 · 89
• 63973 = 7 · 13 · 19 · 37
• 75361 = 11 · 13 · 17 · 31
• 294409 = 37 · 73 · 109
• 56052361 = 211 · 421 · 631
• 118901521 = 271 · 541 · 811
• 172947529 = 307 · 613 · 919

Exercises for Chapter 3

71

• 1024651 = 19 · 199 · 271
3.14. Use the Miller–Rabin test on each of the following numbers. In each
case, either provide a Miller–Rabin witness for the compositeness of n, or
conclude that n is probably prime by providing 10 numbers that are not
Miller–Rabin witnesses for n.
(a)
(b)
(d)
(f)

n = 1105. (Yes, 5 divides n, but this is just a warm-up exercise!)
n = 294409
(c) n = 294409
n = 118901509
(e) n = 118901521
n = 118901527
(g) n = 118915387

Solution to Exercise 3.14.
(a) n − 1 = 1104 = 24 · 69.
269 ≡ −138 (mod 1105)
22·69 ≡ 259

(mod 1105)

4·69

≡ −324 (mod 1105)

8·69

≡1

2
2

(mod 1105)

Thus 1105 is composite. It factors as n = 5 · 13 · 17.
(b) n − 1 = 294408 = 23 · 36801.
236801 ≡ 512

(mod 294409)

2·36801

≡ −32265

(mod 294409)

4·36801

≡1

(mod 294409)

2
2

Thus 294409 is composite. It factors as n = 37 · 73 · 109.
(c) n − 1 = 294438 = 21 · 147219.
2147219 ≡ 1

(mod 294439)

147219

≡ −1 (mod 294439)

147219

≡1

3
5

(mod 294439)

Thus 2, 3, 5 are not Miller–Rabin witnesses for 294439. It turns out that
294439 is prime.
(d) n − 1 = 118901508 = 22 · 29725377.

72

Exercises for Chapter 3
229725377 ≡ 7906806

(mod 118901509)

2·29725377

≡ −1

(mod 118901509)

29725377

3

≡ −1

(mod 118901509)

2·29725377

≡1

(mod 118901509)

5

≡ −1

(mod 118901509)

2·29725377

≡1

(mod 118901509)

2
3

29725377

5

29725377

7

≡ 7906806

(mod 118901509)

2·29725377

≡ −1

(mod 118901509)

29725377

≡ −1

(mod 118901509)

≡1

(mod 118901509)

7

11
11

2·29725377

Thus 2, 3, 5, 7, and 11 are not Miller–Rabin witnesses for 118901509. It turns
out that 118901509 is prime.
(e) n − 1 = 118901520 = 24 · 7431345
27431345 ≡ 45274074 (mod 118901521)
22·7431345 ≡ 1758249 (mod 118901521)
24·7431345 ≡ 1 (mod 118901521)
28·7431345 ≡ 1 (mod 118901521)
Thus 118901521 is composite. It factors as 118901521 = 271 · 541 · 811.
(f) n − 1 = 118901526 = 21 · 59450763.
259450763 ≡ 1

≡ −1 (mod 118901527)

59450763

≡ −1 (mod 118901527)

59450763

≡1

(mod 118901527)

59450763

≡1

(mod 118901527)

3
5
7
11

(mod 118901527)

59450763

Thus 2, 3, 5, 7, and 11 are not Miller–Rabin witnesses for 118901527. It turns
out that 118901527 is prime.
(g) n − 1 = 118915386 = 21 · 59457693.
259457693 ≡ −5081012 (mod 118915387)
Thus 118915387 is composite. It factors as n = 6571 · 18097.
3.15. Looking back at Exercise 3.9, let’s suppose that for a given N , the magic
box can produce only one decryption exponent. Equivalently, suppose that an
RSA key pair has been compromised and that the private decryption exponent
corresponding to the public encryption exponent has been discovered. Show
how the basic idea in the Miller–Rabin primality test can be applied to use
this information to factor N .

Exercises for Chapter 3

73

Solution to Exercise 3.15.
We are given an encryption/decryption pair (e, d), which means that
ade ≡ a

(mod N )

for all 1 ≤ a < N .

So for most values of a we have ade−1 ≡ 1 (mod N ). (This is true unless gcd(a, N ) > 1, in which case gcd(a, N ) is a nontrivial factor of N .) Using
the idea of the Miller–Rabin test, we factor
de = 2k r

with r odd.

Then for random choices of a, we look at
ar , a2r , a4r , . . . , a2

k

r

mod N.

We know that the last entry in the list is 1.
Now suppose that N factors as pq, where we do not know p and q. We
choose a value for a. The Miller–Rabin test applied to p tells us that either
ar ≡ 1

(mod p),

or else

a2

i

r

≡ −1

(mod p) for some 0 ≤ i < k.

(If the latter is true, we take i to be the smallest such value.) Note that we
do not know the value of i, because we do not know the value of p, but that’s
okay. Next we do the same thing with q. Thus the Miller–Rabin test tells us
that either
ar ≡ 1 (mod q),

or else

a2

j

r

≡ −1

(mod p) for some 0 ≤ j < k,

where again we choose the smallest such j.
We now consider several cases. If ar ≡ 1 (mod p) and αr 6≡ 1 (mod q),
then we recover p by computing
gcd(N, ar − 1) = p.
Similarly, if ar 6≡ 1 (mod p) and αr ≡ 1 (mod q), then gcd(N, ar − 1) = q, so
again we win. On the other hand, if ar ≡ 1 (mod N ), then we get no useful
information, so we need to go try a different value for a.
In the remaining cases we have ar 6≡ 1 (mod p) and αr 6≡ 1 (mod q).
Suppose that i and j are different, say i < j. Then
a2

i

r

≡ −1

(mod p) and

i

a2

i

r

6≡ −1

(mod q),

So computing gcd(N, a2 r +1) = p recovers p. A similar method works if j < i.
And finally, if i = j, then we get no useful information and need to try a
different value for a.
We can summarize the above solution as the following algorithm:
1. Choose a random value 1 < a < N .

74

Exercises for Chapter 3
2. Compute gcd(a, N ). If it is not equal to 1, then it is a nontrivial factor
of N .
3. Let (e, d) be the encryption/decryption pair. Factor de − 1 = 2k r with r
odd.
4. Compute gcd(N, ar −1). If it is a nontrivial factor of N , you’re are done.
i

5. For each 0 ≤ i < k, compute gcd(N, a2 r + 1). If it is a nontrivial factor
of N , you’re done.
6. If you haven’t found a factor of N , go back to Step 1 and choose a new
value of a.
3.16. The function π(X) counts the number of primes between 2 and X.
(a) Compute the values of π(20), π(30), and π(100).
(b) Write a program to compute π(X) and use it to compute π(X) and
the ratio π(X)/(X/ ln(X)) for X = 100, X = 1000, X = 10000, and
X = 100000. Does your list of ratios make the prime number theorem
plausible?
Solution to Exercise 3.16.
X π(X) π(X)/(X/ ln(X)
10
4
0.921
20
8
1.198
30
10
1.134
100
25
1.151
1000
168
1.161
10000
1229
1.132
100000
9592
1.104
1000000 78498
1.084
3.17. Let
π1 (X) = (# of primes p between 2 and X satisfying p ≡ 1 (mod 4)),
π3 (X) = (# of primes p between 2 and X satisfying p ≡ 3 (mod 4)).
Thus every prime other than 2 gets counted by either π1 (X) or by π3 (X).
(a) Compute the values of π1 (X) and π3 (X) for each of the following values
of X.
(i) X = 10.
(ii) X = 25.
(iii) X = 100.
(b) Write a program to compute π1 (X) and π3 (X) and use it to compute their
values and the ratio π3 (X)/π1 (X) for X = 100, X = 1000, X = 10000,
and X = 100000.
(c) Based on your data from (b), make a conjecture about the relative sizes
of π1 (X) and π3 (X). Which one do you think is larger? What do you
think is the limit of the ratio π3 (X)/π1 (X) as X → ∞?
Solution to Exercise 3.17.

Exercises for Chapter 3

75

X π1 (X) π3 (X) π3 (X)/π1 (X)
10
1
2
2.0000
25
3
5
1.6667
100
11
13
1.1818
1000
80
87
1.0875
10000
609
619
1.0164
100000
4783
4808
1.0052
1000000 39175 39322
1.0038
(c) From the data, it appears that π3 (X) > π1 (X) for all X. This is
actually false, but the first X for which the inequality is reversed is extremely large. In any case, the ratio satisfies limX→∞ π3 (X)/π1 (X) = 1.
This is a special case of Dirichlet’s theorem on primes in arithmetic progressions, which says the following. Let gcd(a, N ) = 1 and let πa,N (X) be
the number of primes p between 2 and X satisfying p ≡ a (mod N ). Then
limX→∞ πa,N (X)/π(X) = 1/φ(N ).
3.18. We noted in Section 3.4 that it really makes no sense to say that the
number n has probability 1/ ln(n) of being prime. Any particular number that
you choose either will be prime or will not be prime; there are no numbers
that are 35% prime and 65% composite! In this exercise you will prove a
result that gives a more sensible meaning to the statement that a number has
a certain probability of being prime. You may use the prime number theorem
(Theorem 3.20) for this problem.
(a) Fix a (large) number N and suppose that Bob chooses a random number n
in the interval 21 N ≤ n ≤ 32 N . If he repeats this process many times, prove
that approximately 1/ ln(N ) of his numbers will be prime. More precisely,
define
number of primes between 12 N and 32 N
number of integers between 12 N and 23 N
"
#
Probability that an integer n in the
1
3
= interval 2 N ≤ n ≤ 2 N is a prime num- ,
ber

P (N ) =

and prove that
lim

N →∞

P (N )
= 1.
1/ ln(N )

This shows that if N is large, then P (N ) is approximately 1/ ln(N ).
(b) More generally, fix two numbers c1 and c2 satisfying c1 < c2 . Bob chooses
random numbers n in the interval c1 N ≤ n ≤ c2 N . Keeping c1 and c2
fixed, let
"

#
Probability that an integer n in the inP (c1 , c2 ; N ) = terval c1 N ≤ n ≤ c2 N is a prime num- .
ber

76

Exercises for Chapter 3
In the following formula, fill in the box with a simple function of N so
that the statement is true:
lim

N →∞

P (c1 , c2 ; N )

= 1.

Solution to Exercise 3.18.
We will just write P (N ), instead of P (c1 , c2 ; N ).
P (N ) =
=
=
=
=

# of primes between c1 N and c2 N
N
π(c2 N ) − π(c2 N )
N
µ
¶
c2
c1
1
−
+o
from the prime number theorem
ln(c2 N ) ln(c1 N )
ln(N )
µ
¶
(c2 − c1 ) ln(N ) + O(1)
1
+o
ln(c1 N ) ln(c2 N )
ln(N )
µ
¶
c2 − c1
1
+o
ln(N )
ln(N )

Hence P (N ) divided by (c2 − c1 )/ ln(N ) goes to 1 as N → ∞, or equivalently,
P (N )
= c2 − c1 .
N →∞ ln(N )
lim

For part (a), we have c1 =

1
2

and c2 = 23 , so the limit is 1.

3.19. Continuing with the previous exercise, explain how to make mathematical sense of the following statements.
(a) A randomly chosen odd number N has probability 2/ ln(N ) of being
prime. (What is the probability that a randomly chosen even number is
prime?)
(b) A randomly chosen number N satisfying N ≡ 1 (mod 3) has probability
3/(2 ln(N )) of being prime.
(c) A randomly chosen number N satisfying N ≡ 1 (mod 6) has probability
3/ ln(N ) of being prime.
(d) Let m = p1 p2 · · · pr be a product of distinct primes and let k be a number
satisfying gcd(k, m) = 1. What number should go into the box to make
statement (3.35) correct? Why?
A randomly chosen number N satisfying
N ≡ k (mod m)
has
probability
/ ln(N ) of being prime.

(3.2)

(e) Same question, but for arbitrary m, not just for m that are products of
distinct primes.

Exercises for Chapter 3

77

Solution to Exercise 3.19.
(a,b,c) A solution for this exercise is not currently available.
(d) If m = p1 · · · pr , then the probability that N ≡ k (mod m) is prime is
approximately
¶
r µ
Y
pi
1
·
.
p
−
1
ln(N
)
i
i=1
(e) More generally, for arbitrary m and k satisfying gcd(m, k) = 1, the probability that N ≡ k (mod m) is prime is approximately
Yµ p ¶
1
·
.
p−1
ln(N )
p|m

This is often written as
Yµ

1−

p|m

1
p

¶−1

1
,
ln(N )

·

which is also equal to N/(φ(N ) ln(N )), where φ(N ) is Euler’s phi function.
3.20. The logarithmic integral function Li(X) is defined to be
Z

X

Li(X) =
2

(a) Prove that
X
Li(X) =
+
ln X

Z
2

dt
.
ln t

X

dt
+ O(1).
(ln t)2

(Hint. Integration by parts.)
(b) Compute the limit
lim

X→∞

Li(X)
.
X/ ln X

√
√
(Hint. Break the integral in (a) into two pieces, 2 ≤ t ≤ X and X ≤
t ≤ X, and estimate each piece separately.)
(c) Use (b) to show that formula (3.12) on page 131 implies the prime number
theorem (Theorem 3.20).
Solution to Exercise 3.20.
A solution for this exercise is not currently available.
Section. Pollard’s p − 1 factorization algorithm
3.21. Use Pollard’s p − 1 method to factor each of the following numbers.
(a) n = 1739

(b) n = 220459

(c) n = 48356747

78

Exercises for Chapter 3

Be sure to show your work and to indicate which prime factor p of n has the
property that p − 1 is a product of small primes.
Solution to Exercise 3.21.
(a)
23! − 1 ≡ 63

(mod 1739)

gcd(23! − 1, 1739) = 1

24! − 1 ≡ 1082 (mod 1739)

gcd(24! − 1, 1739) = 1

25! − 1 ≡ 1394 (mod 1739)

gcd(25! − 1, 1739) = 1

26! − 1 ≡ 1443 (mod 1739)

gcd(26! − 1, 1739) = 37

This give 1739 = 37 · 47. Note that p − 1 = 36 = 22 · 32 and q − 1 = 46 = 2 · 23.
(b)
23! − 1 ≡ 63

(mod 220459)

gcd(23! − 1, 220459) = 1

24! − 1 ≡ 22331

(mod 220459)

gcd(24! − 1, 220459) = 1

25! − 1 ≡ 85053

(mod 220459)

gcd(25! − 1, 220459) = 1

26! − 1 ≡ 4045

(mod 220459)

gcd(26! − 1, 220459) = 1

27! − 1 ≡ 43102

(mod 220459)

gcd(27! − 1, 220459) = 1

28! − 1 ≡ 179600 (mod 220459)

gcd(28! − 1, 220459) = 449

This gives 220459 = 449 · 491. Note that p − 1 = 448 = 26 · 7 and q − 1 =
490 = 2 · 5 · 72 .
(c)
215! − 1 ≡ 46983890 (mod 48356747)

gcd(215! − 1, 48356747) = 1

216! − 1 ≡ 8398520

(mod 48356747)

gcd(216! − 1, 48356747) = 1

217! − 1 ≡ 9367159

(mod 48356747)

gcd(217! − 1, 48356747) = 1

218! − 1 ≡ 17907955 (mod 48356747)

gcd(218! − 1, 48356747) = 1

219! − 1 ≡ 13944672 (mod 48356747)

gcd(219! − 1, 48356747) = 6917

This gives 48356747 = 6917 · 6991. Note that p − 1 = 6916 = 22 · 7 · 13 · 19 and
q − 1 = 6990 = 2 · 3 · 5 · 233.
3.22. A prime of the form 2n − 1 is called a Mersenne prime.
(a) Factor each of the numbers 2n − 1 for n = 2, 3, . . . , 10. Which ones are
Mersenne primes?
(b) Find the first seven Mersenne primes. (You may need a computer.)
(c) If n is even and n > 2, prove that 2n − 1 is not prime.
(d) If 3 | n and n > 3, prove that 2n − 1 is not prime.
(e) More generally, prove that if n is a composite number, then 2n − 1 is not
prime. Thus all Mersenne primes have the form 2p − 1 with p a prime
number.

Exercises for Chapter 3

79

(f) What is the largest known Mersenne prime? Are there any larger primes
known? (You can find out at the “Great Internet Mersenne Prime Search”
web site www.mersenne.org/prime.htm.)
(g) Write a one page essay on Mersenne primes, starting with the discoveries
of Father Mersenne and ending with GIMPS.
Solution to Exercise 3.22.
The factorization of 2n − 1 for 2 ≤ n ≤ 20 is
211 − 1 = 2047 = 23 · 89
22 − 1 = 3 = 3

212 − 1 = 4095 = 32 · 5 · 7 · 13

23 − 1 = 7 = 7

213 − 1 = 8191 = 8191

24 − 1 = 15 = 3 · 5

214 − 1 = 16383 = 3 · 43 · 127

25 − 1 = 31 = 31

215 − 1 = 32767 = 7 · 31 · 151

26 − 1 = 63 = 32 · 7

216 − 1 = 65535 = 3 · 5 · 17 · 257

27 − 1 = 127 = 127

217 − 1 = 131071 = 131071

28 − 1 = 255 = 3 · 5 · 17

218 − 1 = 262143 = 33 · 7 · 19 · 73

29 − 1 = 511 = 7 · 73

219 − 1 = 524287 = 524287

210 − 1 = 1023 = 3 · 11 · 31

220 − 1 = 1048575 = 3 · 52 · 11 · 31 · 41

Thus the first few Mersenne primes are
22 − 1 = 3,
213 − 1 = 8191,

23 − 1 = 7,
217 − 1 = 131071,

25 − 1 = 31,

27 − 1 = 127,

219 − 1 = 524287.

Notice that 2p − 1 is prime for all primes p < 20 except for p = 11. However,
this is somewhat misleading. For the primes 20 < p < 40, only 231 − 1 yields
a Mersenne prime.
223 − 1 = 8388607 = 47 · 178481
229 − 1 = 536870911 = 233 · 1103 · 2089
231 − 1 = 2147483647 = 2147483647
237 − 1 = 137438953471 = 223 · 616318177
241 − 1 = 2199023255551 = 13367 · 164511353
243 − 1 = 8796093022207 = 431 · 9719 · 2099863
247 − 1 = 140737488355327 = 2351 · 4513 · 13264529

(c) If n is even, say n = 2m, then 2n − 1 = 22m − 1 = (2m − 1)(2m + 1), so
2n − 1 is composite unless 2m − 1 = 1, i.e. unless m = 1 and n = 2.

80

Exercises for Chapter 3

(d) Similarly, 23m − 1 = (2m − 1)(22m + 2m + 1), so it is composite unless
m = 1.
(e) More generally,
2km − 1 = (2m − 1)(2(k−1)m + 2(k−2)m + · · · + 22m + 2m + 1),
so 2km − 1 is composite unless m = 1 or k = 1. Notice that what we are really
doing is using the standard identity
xk − 1 = (x − 1)(x(k−1) + x(k−2) + · · · + x2 + x + 1)
with x = 2m .
(f) As of January 2008, the largest known Mersenne prime is 232582657 − 1,
which was discovered in September 2006 as part of the GIMPS project.

Exercises for Chapter 3

81

Section. Factorization via difference of squares
3.23. For each of the following numbers N , compute the values of
N + 12 ,

N + 22 ,

N + 32 ,

N + 42 ,

...

as we did in Example 3.33 until you find a value N + b2 that is a perfect
square a2 . Then use the values of a and b to factor N .
(a) N = 53357

(b) N = 34571

(c) N = 25777

(d) N = 64213

Solution to Exercise 3.23.
(a)
53357 + 12 = 53358
2

not a square,
2

53357 + 2 = 53361 = 231
Thus

** square **.

53357 = 2312 − 22 = (231 + 2)(231 − 2) = 233 · 229.

(b)
34571 + 12 = 34572

not a square,

2

34571 + 2 = 34575

not a square,

2

not a square,

34571 + 3 = 34580
2

34571 + 4 = 34587
2

not a square,
2

34571 + 5 = 34596 = 186
Thus

** square **.

34571 = 1862 − 52 = (186 + 5)(186 − 5) = 191 · 181.

(c)
25777 + 12 = 25778

not a square

2

25777 + 2 = 25781

not a square

2

25777 + 3 = 25786

not a square

2

25777 + 4 = 25793

not a square

2

25777 + 5 = 25802

not a square

2

25777 + 6 = 25813

not a square

2

25777 + 7 = 25826

not a square

2

25777 + 8 = 25841

not a square

2

25777 + 9 = 25858

not a square

2

not a square

25777 + 10 = 25877
2

25777 + 11 = 25898
2

25777 + 12 = 25921 = 161

not a square
2

** square **

82

Exercises for Chapter 3

Thus

25777 = 1612 − 122 = (161 + 12)(161 − 12) = 173 · 149.

(d) Most people will give up before finishing this one unless they write a
computer program! It is included to make people aware that this method
doesn’t always work.
64213 + 12 = 64214

not a square

2

64213 + 2 = 64217

not a square

2

64213 + 3 = 64222

not a square

2

64213 + 4 = 64229
..
.

not a square
..
.

64213 + 1212 = 78854

not a square

2

not a square

2

not a square

2

not a square

64213 + 122 = 79097
64213 + 123 = 79342
64213 + 124 = 79589
2

64213 + 125 = 79838
2

not a square

64213 + 126 = 80089 = 283

2

** square **

Thus
64213 = 2832 − 1262 = (283 + 126)(283 − 126) = 409 · 157.

3.24. For each of the listed values of N , k, and binit , factor N by making
a list of values of k · N + b2 , starting at b = binit and incrementing b until
k · N + b2 is a perfect square. Then take greatest common divisors as we did
in Example 3.34.
(a)
(b)
(c)

N = 143041
N = 1226987
N = 2510839

k = 247
k=3
k = 21

binit = 1
binit = 36
binit = 90

Solution to Exercise 3.24.
(a)
247 · 143041 + 12 = 35331128

not a square

2

247 · 143041 + 2 = 35331131
2

not a square
2

247 · 143041 + 3 = 35331136 = 5944

** square **

Thus
247 · 143041 = 59442 − 32 = (5944 + 3)(5944 − 3) = 5947 · 5941.

Exercises for Chapter 3

83

gcd(143041, 5947) = 313,

gcd(143041, 5941) = 457

(b)
3 · 1226987 + 362 = 3682257

not a square

2

not a square

2

not a square

3 · 1226987 + 37 = 3682330
3 · 1226987 + 38 = 3682405
2

3 · 1226987 + 39 = 3682482

not a square

2

2

3 · 1226987 + 40 = 3682561 = 1919

** square **

Thus
3 · 1226987 = 19192 − 402 = (1919 + 40)(1919 − 40) = 1959 · 1879.
gcd(1226987, 1959) = 653,

gcd(1226987, 1879) = 1879

(c)
21 · 2510839 + 902 = 52735719

not a square

2

not a square

2

not a square

2

not a square

21 · 2510839 + 91 = 52735900
21 · 2510839 + 92 = 52736083
21 · 2510839 + 93 = 52736268
2

21 · 2510839 + 94 = 52736455
2

not a square
2

21 · 2510839 + 95 = 52736644 = 7262

** square **

Thus
21 · 2510839 = 72622 − 952 = (7262 + 95)(7262 − 95) = 7357 · 7167.
gcd(2510839, 7357) = 1051,

gcd(2510839, 7167) = 2389

3.25. For each part, use the data provided to find values of a and b satisfying
a2 ≡ b2 (mod N ), and then compute gcd(N, a − b) in order to find a nontrivial
factor of N , as we did in Examples 3.36 and 3.37.
(a) N = 61063
18822 ≡ 270

270 = 2 · 33 · 5

(mod 61063)

and

18982 ≡ 60750 (mod 61063)

and

60750 = 2 · 35 · 53

(b) N = 52907
3992 ≡ 480

(mod 52907)

and

480 = 25 · 3 · 5

7632 ≡ 192

(mod 52907)

and

192 = 26 · 3

7732 ≡ 15552

(mod 52907)

and

15552 = 26 · 35

9762 ≡ 250

(mod 52907)

and

250 = 2 · 53

84

Exercises for Chapter 3

(c) N = 198103
11892 ≡ 27000

(mod 198103)

and

27000 = 23 · 33 · 53

16052 ≡ 686

(mod 198103)

and

23782 ≡ 108000

(mod 198103)

and

108000 = 25 · 33 · 53

28152 ≡ 105

(mod 198103)

and

105 = 3 · 5 · 7

686 = 2 · 73

(d) N = 2525891
15912 ≡ 5390

(mod 2525891)

and

5390 = 2 · 5 · 72 · 11

31822 ≡ 21560

(mod 2525891)

and

21560 = 23 · 5 · 72 · 11

47732 ≡ 48510

(mod 2525891)

and

48510 = 2 · 32 · 5 · 72 · 11

52752 ≡ 40824

(mod 2525891)

and

40824 = 23 · 36 · 7

54012 ≡ 1386000 (mod 2525891)

and

1386000 = 24 · 32 · 53 · 7 · 11

Solution to Exercise 3.25.
(a)
18822 · 18982 ≡ (2 · 33 · 5)(2 · 35 · 53 )
4

(mod 61063)

2 2

= (2 · 3 · 5 )
= 40502

gcd(61063, 1882 · 1898 − 4050) = 227

Eureka!

(b) The most natural combination to try first is
7632 · 7732 ≡ (26 · 3)(26 · 35 ) (mod 52907)
= (26 · 33 )2
= 17282
gcd(52907, 763 · 773 − 1728) = 277

Eureka!

So this works. However, if instead we use
3992 · 7632 · 9762 ≡ (25 · 3 · 5)(26 · 3)(2 · 53 ) (mod 52907)
= (26 · 3 · 52 )2
= 48002 ,
then we do not win, since
gcd(52907, 399 · 763 · 976 − 4800) = 52907
(c) First we try

No help

Exercises for Chapter 3

85

11892 · 23782 ≡ (23 · 33 · 53 )(25 · 33 · 53 )
4

3

(mod 198103)

3 2

= (2 · 3 · 5 )
= 540002

gcd(198103, 1189 · 2378 − 54000) = 198103

No help

This didn’t work, so next we try
11892 · 16052 · 28152 ≡ (23 · 33 · 53 )(2 · 73 )(3 · 5 · 7) (mod 198103)
= (22 · 32 · 52 · 72 )2
= 441002
gcd(198103, 1189 · 1605 · 2815 − 44100) = 499

Eureka!

(d) First we try
15912 · 31822 ≡ (2 · 5 · 72 · 11)(23 · 5 · 72 · 11)
2

2

(mod 2525891)

2

= (2 · 5 · 7 · 11)
= 107802

gcd(2525891, 1591 · 3182 − 10780) = 2525891

No help

Next we try
15912 · 47732 ≡ (2 · 5 · 72 · 11)(2 · 32 · 5 · 72 · 11)
2

(mod 2525891)

2

= (2 · 3 · 5 · 7 · 11)
= 161702

gcd(2525891, 1591 · 4773 − 16170) = 2525891

No help

Finally we win when we try
15912 · 52752 · 54012
≡ (2 · 5 · 72 · 11)(23 · 36 · 7)(24 · 32 · 53 · 7 · 11) (mod 2525891)
= (24 · 34 · 52 · 72 · 11)2
= 174636002
gcd(2525891, 1591 · 5275 · 5401 − 17463600) = 1637

Eureka!

Section. Smooth numbers, sieves, and building relations for factorization
3.26. Compute the following values of ψ(X, B), the number of B-smooth
numbers between 2 and X (see page 146).
(a) ψ(25, 3)

(b) ψ(35, 5)
(c) ψ(50, 7)
(e) ψ(100, 7)

(d) ψ(100, 5)

86

Exercises for Chapter 3

Solution to Exercise 3.26.
(a) ψ(25, 3) = 10
(b) ψ(35, 5) = 18
(c) ψ(50, 7) = 30
(d) ψ(100, 5) = 33
(e) ψ(100, 7) = 45
3.27. An integer M is called B-power-smooth if every prime power pe dividing M satisfies pe ≤ B. For example, 180 = 22 · 32 · 5 is 10-power-smooth,
since the largest prime power dividing 180 is 9, which is smaller than 10.
(a) Suppose that M is B-power-smooth. Prove that M is also B-smooth.
(b) Suppose that M is B-smooth. Is it always true that M is also B-powersmooth? Either prove that it is true or give an example for which it is
not true.
(c) The following is a list of 20 randomly chosen numbers between 1 and 1000,
sorted from smallest to largest. Which of these numbers are 10-powersmooth? Which of them are 10-smooth?
{84, 141, 171, 208, 224, 318, 325, 366, 378, 390, 420, 440,
504, 530, 707, 726, 758, 765, 792, 817}
(d) Prove that M is B-power-smooth if and only if M divides the least common multiple of [1, 2, . . . , B]. (The least common multiple of a list of
numbers k1 , . . . , kr is the smallest number K that is divisible by every
number in the list.)
Solution to Exercise 3.27.
(a,b,d) A solution for this exercise is not currently available.
(c) The numbers 84 = 22 · 3 · 7, 420 = 22 · 3 · 5 · 7, 504 = 23 · 32 · 7, and are
10-power-smooth. They are also 10-smooth, of course, as are the additional
numbers 224 = 25 · 7 and 378 = 2 · 33 · 7.
√
3.28. Let L(N ) = e (ln N )(ln ln N ) as usual. Suppose that a computer does
one billion operations per second.
(a) How many seconds does it take to perform L(2100 ) operations?
(b) How many hours does it take to perform L(2250 ) operations?
(c) How many days does it take to perform L(2350 ) operations?
(d) How many years does it take to perform L(2500 ) operations?
(e) How many years does it take to perform L(2750 ) operations?
(f) How many years does it take to perform L(21000 ) operations?
(g) How many years does it take to perform L(22000 ) operations?
(For simplicity, you may assume that there are 365.25 days in a year.)
Solution to Exercise 3.28.
(a) N = 2100 : L(N ) = 224.73 steps takes 0.03 seconds.
(b) N = 2250 : L(N ) = 243.12 steps takes 2.65 hours.

Exercises for Chapter 3

87

= 2350 : L(N ) = 252.66 steps takes 82.24 days.
= 2500 : L(N ) = 264.95 steps takes 1129.30 years.
= 2750 : L(N ) = 282.26 steps takes 108.26 years.
= 21000 : L(N ) = 297.14 steps takes 1012.74 years.
= 22000 : L(N ) = 2144.48 steps takes 1026.99 years.
√
3.29. Prove that the function L(X) = e (ln X)(ln ln X) is subexponential. That
is, prove the following two statements.
¡
¢
(a) For every positive constant α, no matter how large, L(X) = Ω (ln X)α .
¡ β
(b) For every positive constant β, no matter how small, L(X) = O X ).
(c)
(d)
(e)
(f)
(g)

N
N
N
N
N

Solution to Exercise 3.29.
A solution for this exercise is not currently available.
3.30. For any fixed positive constants a and b, define the function
1/a

Fa,b (X) = e(ln X)

(ln ln X)1/b

.

Prove the following properties of Fa,b (X).
(a) If a > 1, prove that Fa,b (X) is subexponential.
(b) If a = 1, prove that Fa,b (X) is exponential.
(c) What happens if a < 1?
Solution to Exercise 3.30.
A solution for this exercise is not currently available.
3.31. This exercise asks you to verify an assertion
√ in the proof of Corollary 3.44. Let L(X) be the usual function L(X) = e (ln X)(ln ln X) .
(a) Prove that there is a value of ² > 0 such that
(ln X)² < ln L(X) < (ln X)1−²

for all X > 10.

(b) Let c > 0, let Y = L(X)c , and let u = (ln X)/(ln Y ). Prove that
1

u−u = L(X)− 2c (1+o(1)) .
Solution to Exercise 3.31.
(a) is clear, any ² < 12 will work.
(b) We first compute
ln X
ln X
1
u=
= p
=
c
ln L(X)
c
c (ln X)(ln ln X)
Then

r

ln X
.
ln ln X

88

Exercises for Chapter 3
à r
!
ln X
1
ln X
ln
ln ln X
c ln ln X
r
·
¸
1
1
ln X
1
=
ln ln X − ln ln ln X − ln c
c ln ln X 2
2
¡
¢
1√
=
ln X ln ln X 1 + o(1)
2c
¢¡
¢
1¡
=
ln L(X) 1 + o(1) .
2c

1
u ln u =
c

Hence

r

1

uu = L(X) 2c (1+o(1)) .

3.32. Proposition 3.47 assumes that we choose random numbers a modulo N ,
compute a2 (mod N ), and check whether the result is B-smooth. We can
achieve better results if we take values for a of the form
¥√ ¦
a=
N +k
for 1 ≤ k ≤ K.
(For simplicity, you may treat K as a fixed integer, independent of N . More
rigorously, it is necessary to take K equal to a power of L(N ), which has a
small effect on the final answer.)
√
2
2
(a) Prove that a2 −N ≤
√2K N +K , so in particular, a (mod N ) is smaller
than a multiple of N .
√
√
(b) Prove that L( N ) ≈ L(N )1/ 2 by showing that
√
log L( N )
√ = 1.
lim
N →∞ log L(N )1/ 2
√

More generally, prove that in the same sense, L(N 1/r ) ≈ L(N )1/ r for
any fixed r > 0.
(c) Re-prove Proposition 3.47 using this better choice of values for a. Set
B = L(N )c and find the optimal value of c. Approximately how many
relations are needed to factor N ?
Solution to Exercise 3.32.
√
(a) We
some 0 ≤ ² < 1. Hence a2 − N =
√ have a =2 N +√² + k for
2
2(² + k) N + (² + k) ≤ 2K N + K .
(b) A rough computation shows that
√ √
√
√
L( N ) = e (ln N )(ln ln N )
√ 1
1
= e ( 2 ln N )(ln 2 ln N )
√ 1
√
≈ e ( 2 ln N )(ln ln N ) = L(N )1/ 2 .

Exercises for Chapter 3

89

More precisely, we first simplify
√
log L(N 1/r )
ln N 1/r ln ln N 1/r
√ =
√ √
1/
r
log L(N )
(1/ r) ln N ln ln N
s
(1/r)(ln N )(ln(1/r) + ln ln N )
=
(1/r)(ln N )(ln ln N )
r
− ln r
=
+ 1.
ln ln N
It is clear that if r is fixed, then this last expression goes to 1 as N → ∞.
(c) We mimic the proof of Proposition
3.47. The probability
that
√
√
√ a random
number that is approximately N is B-smooth is ψ( N , B)/ N , and we
need approximately π(B) relations, so we need to check approximately
π(B)
√
√
ψ( N , B)/ N

numbers.

(3.3)

√
We set B = L( N )c , substitute into ??, and use Theorem ?? and the prime
number theorem (Theorem 3.20) to get (we ignore various lower-order log
terms)
√
√
1
1
π(L( N )c )
L( N )c
√
(c+ 2c
)
√
√
√ ≈ √
≈ L(N ) 2
.
c
−1/2c
ψ( N , L( N ) )/ N
L( N )
The exponent is minimized when c =

√1 ,
2

√
√
so we should take B = L( N )1/ 2 ≈

L(N )1/2 and we need to check approximately L(N ) numbers in order to factor N . Of course, our assumptions mean that this is an underestimate, but
this exercise suggests that without some significant new idea, the running time
of this method will be at least O(L(N )).
3.33. Illustrate the quadratic sieve, as was done in Figure 3.3 (page 157), by
sieving prime powers up to B on the values of F (T ) = T 2 −N in the indicated
range.
(a) Sieve N = 493 using prime powers up to B = 11 on values from F (23)
to F (38). Use the relation(s) that you find to factor N .
(b) Extend the computations in (a) by using prime powers up to B = 16 and
sieving values from F (23) to F (50). What additional value(s) are sieved
down to 1 and what additional relation(s) do they yield?
Solution to Exercise 3.33.
(a) We sieve the following values, as illustrated in Table ??:
• The congruence t2 ≡ 493 ≡ 1 (mod 2) has solution t ≡ 1 (mod 2), so we
sieve 2 from F (23), F (25), F (27),. . . .

90

Exercises for Chapter 3
• The congruence t2 ≡ 493 ≡ 1 (mod 3) has solutions t ≡ 1 (mod 3) and
t ≡ 2 (mod 3), so first we sieve 3 from F (23), F (26), F (29),. . . , and
then we sieve 3 from F (25), F (28), F (31),. . . .
• The congruence t2 ≡ 493 ≡ 1 (mod 4) has solution t ≡ 1 (mod 2), so we
sieve another 2 from F (23), F (25), F (27),. . . .
• The congruence t2 ≡ 493 ≡ 3 (mod 5) has no solutions.
• The congruence t2 ≡ 493 ≡ 3 (mod 7) has no solutions.
• The congruence t2 ≡ 493 ≡ 5 (mod 8) has no solutions.
• The congruence t2 ≡ 493 ≡ 7 (mod 9) has solutions t ≡ 4 (mod 9) and
t ≡ 5 (mod 9), so first we sieve another 3 from F (31) and then we sieve
another 3 from F (23) and F (32).
• The congruence t2 ≡ 493 ≡ 9 (mod 11) has solutions t ≡ 3 (mod 11)
and t ≡ 8 (mod 11), so first we sieve 11 from F (25) and F (36) and then
we sieve 11 from F (30).

The two values F (23) and F (25) have been sieved down to 1, yielding the
congruences
F (23) ≡ 36 ≡ 22 ·32 (mod 493)

F (25) ≡ 132 ≡ 22 ·3·11 (mod 493).

and

Since F (23) is itself congruent to a square, we can compute
gcd(23 − 2 · 3, 493) = 17,
which gives the factorization 493 = 17 · 29.
(b) The first step is to make Table ?? wider, i.e. sieve the values from F (23)
to F (50) using prime powers up to B = 11. The next step is to sieve out the
additional prime powers up to B = 16.
The congruence t2 ≡ 493 ≡ 12 (mod 13) has solutions t ≡ 5 (mod 13)
and t ≡ 8 (mod 13), so first we sieve 13 from F (31) and F (44), and then we
sieve 13 from F (34) and F (47). The only other prime power up to B = 16
is 16, and the congruence t2 ≡ 493 ≡ 13 (mod 16) has no solutions (as indeed
it cannot, since we already noted that t2 ≡ 493 (mod 8) has no solutions).
We do not give the entire sieve table, but merely observe that two more
values have been sieved down to 1, namely
F (31) = 468 ≡ 22 ·32 ·13 (mod 493)

and

F (47) = 1716 ≡ 22 ·3·11·13 (mod 493).

Combining these with the earlier fully sieved values gives the relation
(25·31·47)2 ≡ (22 ·3·11)·(22 ·32 ·13)·(22 ·3·11·13) ≡ (23 ·32 ·11·13)3 (mod 493).
Unfortunately,
gcd(25 · 31 · 47 − 23 · 32 · 11 · 13, 493) = gcd(26129, 493) = 493,
so this relation does not give a factorization of 493.

Exercises for Chapter 3
23
36
↓2
18
↓3
6
6
↓2
3
3
↓3
1
1
1

24 25 26 27 28 29 30 31 32 33 34
83 132 183 236 291 348 407 468 531 596 663
↓2
↓2
↓2
↓2
↓2
83 66 183 118 291 174 407 234 531 298 663
↓3
↓3
↓3
83 66 61 118 291 58 407 234 177 298 663
↓3
↓3
↓3
↓3
83 22 61 118 97 58 407 78 177 298 221
↓2
↓2
↓2
↓2
↓2
83 11 61 59 97 29 407 39 177 149 221
↓3
83 11 61 59 97 29 407 13 177 149 221
↓3
83 11 61 59 97 29 407 13 59 149 221
↓11
83 1
61 59 97 29 407 13 59 149 221
↓11
83 1
61 59 97 29 37 13 59 149 221

91
35 36 37 38
732 803 876 951
↓2
↓2
366 803 438 951
↓3
↓3
122 803 438 317
↓3
122 803 146 317
↓2
↓2
61 803 73 317
61

803

73

317

61

73

317

61

803
↓11
73

73

317

61

73

73

317

Table 3.1: Sieving N = 493
3.34. Let Z[β] be the ring described in Example 3.54, i.e., β is a root of f (x) =
1 + 3x − 2x3 + x4 . For each of the following pairs of elements u, v ∈ Z[β],
compute the sum u + v and the product uv. Your answers should involve only
powers of β up to β 3 .
(a) u = −5 − 2β + 9β 2 − 9β 3 and v = 2 + 9β − 7β 2 + 7β 3 .
(b) u = 9 + 9β + 6β 2 − 5β 3 and v = −4 − 6β − 2β 2 − 5β 3 .
(c) u = 6 − 5β + 3β 3 + 3β 3 and v = −2 + 7β + 6β 2 .
Solution to Exercise 3.34.
(a) u + v = −3 + 7β + 2β 2 − 2β 3 and uv = 148 + 425β + 98β 2 − 85β 3 .
(b) u + 4 = 5 + 3β + 4β 2 − 10β 3 and uv = −69 − 219β − 211β 2 − 88β 3 .
(c) u + v = 4 + β + 9β 2 + 3β 3 and uv = −87 − 189β − 66β 2 + 129β 3 .
Section. The index calculus and discrete logarithms
3.35. This exercise asks you to use the index calculus to solve a discrete
logarithm problem. Let p = 19079 and g = 17.
(a) Verify that g i (mod p) is 5-smooth for each of the values i = 3030,
i = 6892, and i = 18312.
(b) Use your computations in (a) and linear algebra to compute the discrete
logarithms logg (2), logg (3), and logg (5). (Note that 19078 = 2 · 9539 and
that 9539 is prime.)
(c) Verify that 19 · 17−12400 is 5-smooth.

92

Exercises for Chapter 3

(d) Use the values from (b) and the computation in (c) to solve the discrete
logarithm problem
17x ≡ 19 (mod 19079).
Solution to Exercise 3.35.
(a) We have
g 3030 ≡ 22 · 36 · 5,

g 6892 ≡ 211 · 32 ,

g 18312 ≡ 24 · 3 · 53 .

(b) We get the linear equations
3030 = 2x2 + 6x3 + x5
6892 = 11x2 + 2x3
18312 = 4x2 + x3 + 2x5
Solving modulo 2 and modulo 9539 gives
(x2 , x3 , x5 ) ≡ (0, 0, 0) (mod 2),
(x2 , x3 , x5 ) ≡ (8195, 1299, 7463) (mod 9539).
Hence
(x2 , x3 , x5 ) ≡ (17734, 10838, 17002) (mod 19079).
(c) We compute
h · g −12224 ≡ 213

(mod 19079).

(d) Hence
logg (h) = 12224 + 13 · logg (2) = 242766 ≡ 13830 (mod p − 1).
We check that 1713830 ≡ 19 (mod 19079). X
Section. Quadratic residues and quadratic reciprocity
3.36. Let p be an odd prime and let a be an integer with p - a.
(a) Prove that a(p−1)/2 is congruent to either 1 or −1 modulo p.
(b) Prove that a(p−1)/2 is congruent to 1 modulo p if and only if a is a
quadratic residue modulo p. (Hint. Let g be a primitive root for p and
use the fact, proven during the course of proving Proposition 3.60, that g m
is a quadratic residue if
¡ and
¢ only if m is even.)
(c) Prove that a(p−1)/2 ≡ ap (mod p). (This holds even if p | a.)
(d) Use (c) to prove Theorem 3.61(a), that is, prove that
µ ¶ (
1
if p ≡ 1 (mod 4),
−1
=
p
−1 if p ≡ 3 (mod 4).
Solution to Exercise 3.36.
A solution for this exercise is not currently available.

Exercises for Chapter 3

93

3.37. Prove that the three parts of the quadratic reciprocity theorem (Theorem 3.61) are equivalent to the following three concise formulas, where p and q
are odd primes:
µ ¶
µ ¶µ ¶
µ ¶
p−1
p2 −1
p−1 q−1
2
p q
−1
2
8
(b)
(c)
(a)
= (−1)
= (−1)
= (−1) 2 · 2
p
p
q
p
Solution to Exercise 3.37.
A solution for this exercise is not currently available.
3.38. Let p be a prime satisfying p ≡ 3 (mod 4).
(a) Let a be a quadratic residue modulo p. Prove that the number
b≡a

p+1
4

(mod p)

p−1
has the property that b2 ≡ a (mod p). (Hint. Write p+1
2 as 1 + 2 and
use Exercise 3.36.) This gives an easy way to take square roots modulo p
for primes that are congruent to 3 modulo p.
(b) Use (a) to compute the following square roots modulo p. Be sure to check
your answers.

(i) Solve b2 ≡ 116 (mod 587).
(ii) Solve b2 ≡ 3217 (mod 8627).
(iii) Solve b2 ≡ 9109 (mod 10663).
Solution to Exercise 3.38.
This was proven in Chapter 2, see Proposition 2.27, but it is included here
as an exercise because of its importance, and because the use of the Legendre
symbol makes for a short proof.
(a)
µ ¶
p+1
p−1
a
b2 ≡ a 2 ≡ a1+ 2 ≡ a ·
≡ a (mod p).
p
¡ ¢
p−1
We are using a 2 ≡ ap from the previous exercise and the assumption that a
¡ ¢
is a quadratic residue, which tells us that ap = 1.
(b) (i) 116(587+1)/4 = 116147 ≡ 65 (mod 587). Check: 652 ≡ 116 (mod 587).
(ii) 3217(8627+1)/4 ≡ 18652157 ≡ 2980 (mod 8627). Check: 29802 ≡ 3217
(mod 8627).
2
(iii) 9109(10663+1)/4 ≡ 91092666 ≡ 3502 (mod 10663). Check:
3502
¡ 9109
¢ ≡ 1554
(mod 10663). Oops, what’s going on? The problem is that 10663
= −1, so
9109 is not a quadratic
residue
modulo
10663.
In
fact,
the
previous
exercise
¡ ¢
tells us that b2 ≡ ap a (mod p), and indeed in this case we have 35022 ≡ −9109
(mod 10663).
3.39. Recall that for any a ∈ F∗p , the discrete logarithm of a (with respect to
a primitive root g) is a number logg (a) satisfying

94

Exercises for Chapter 3
g logg (a) ≡ a

Prove that

µ ¶
a
= (−1)logg (a)
p

(mod p).
for all a ∈ F∗p .

Thus quadratic reciprocity gives a fast method to compute the parity of logg (a).
Solution to Exercise 3.39.
To ease notation,let k = logg (a). Then
µ ¶
a
≡ a(p−1)/2 (mod p)
from earlier exercise,
p
≡ g k(p−1)/2
≡ (−1)

k

(mod p)

from definition of discrete log,
since g (p−1)/2 ≡ 1 (mod p).

(mod p)

3.40. Let p ≥ 5 be a prime. We say that a is a cubic residue modulo p if p - a
and there is an integer c satisfying a ≡ c3 (mod p).
(a) Let a and b be cubic residues modulo p. Prove that ab is a cubic residue
modulo p.
(b) Give an example to show that (unlike the case with quadratic residues)
it is possible for none of a, b, and ab to be a cubic residue modulo p.
(c) Let g be a primitive root modulo p. Prove that a is a cubic residue
modulo p if and only if 3 | logg (a), where logg (a) is the discrete logarithm
of a.
Solution to Exercise 3.40.
It is easiest to prove (c) first, but we give a direct proof of (a). The assumption is that there are numbers c and d satisfying
a ≡ c3

mod p

and

b ≡ d3

(mod p).

Then ab = (cd)3 (mod p), so ab is also a cubic residue modulo p.
(b,c) A solution for this exercise is not currently available.
Section. Probabilistic encryption and the Goldwasser–Micali cryptosystem
3.41. Perform the following encryptions and decryptions using the Goldwasser–Micali public key cryptosystem (Table 3.9).
(a) Bob’s public key is the pair N = 1842338473 and a = 1532411781. Alice
encrypts three bits and sends Bob the ciphertext blocks
1794677960,

525734818,

and

420526487.

Decrypt Alice’s message using the factorization
N = pq = 32411 · 56843.

Exercises for Chapter 3

95

(b) Bob’s public key is N = 3149 and a = 2013. Alice encrypts three bits
and sends Bob the ciphertext blocks 2322, 719, and 202. Unfortunately,
Bob used primes that are much too small. Factor N and decrypt Alice’s
message.
(c) Bob’s public key is N = 781044643 and a = 568980706. Encrypt the
three bits 1, 1, 0 using, respectively, the three random values
r = 705130839,

r = 631364468,

r = 67651321.

Solution to Exercise 3.41.
¡
¢
(a) Decrypt c = 1794677960 by computing 1794677960
= −1,
32411
¡ which¢gives
the plaintext bit m = 1. Decrypt c = 525734818 by computing 525734818
= 1,
32411
which
gives
the
plaintext
bit
m
=
0.
Decrypt
c
=
420526487
by
computing
¡420526487¢
= −1, which gives the plaintext bit m = 1. Alice’s plaintext is
32411
(1, 0, 1).
(b) The factorization
of m is m = 3149 = 47 · 57. Decrypt c = 2322 by
¡
¢
computing 2322
=
−1,
which gives the plaintext bit m = 1. Decrypt c = 719
47¡
¢
719
by computing ¡47 ¢= 1, which gives the plaintext bit m = 0. Decrypt c = 202
by computing 202
= 1, which gives the plaintext bit m = 0. Thus Alice’s
47
plaintext is (1, 0, 0).
(c) Although it is not needed to do this problem, the factorization of m is m =
781044643 = 22109 · 35327. Encrypt m = 1 using r = 705130839. Compute
c ≡ ar2 ≡ 568980706 · 7051308392 ≡ 517254876 (mod 781044643). Encrypt
m = 1 using r = 631364468. Compute c ≡ ar2 ≡ 568980706 · 6313644682 ≡
4308279 (mod 781044643). Encrypt m = 0 using r = 67651321. Compute c ≡
r2 ≡ 676513212 ≡ 660699010 (mod 781044643). The ciphertext for (1, 1, 0) is
(517254876, 4308279, 660699010).
3.42. Suppose that the plaintext space M of a certain cryptosystem is the
set of bit strings of length 2b. Let ek and dk be the encryption and decryption
functions associated with a key k ∈ K. This exercise describes one method
of turning the original cryptosystem into a probabilistic cryptosystem. Most
practical cryptosystems that are currently in use rely on more complicated
variants of this idea in order to thwart certain types of attacks. (See Section 8.6
for further details.)
Alice sends Bob an encrypted message by performing the following steps:
1. Alice chooses a b-bit message m0 to be encrypted.
2. Alice chooses a string r consisting of b random bits.
3. Alice sets m = rk(r⊕m0 ), where k denotes concatenation1 and ⊕ denotes
exclusive or (see Section 1.7.4). Notice that m has length 2b bits.
4. Alice computes c = ek (m) and sends the ciphtertext c to Bob.
1 The concatenation of two bit strings is formed by placing the first string before the
second string. For example, 1101 k 1001 is the bit string 11011001.

96

Exercises for Chapter 3

(a) Explain how Bob decrypts Alice’s message and recovers the plaintext m0 .
We assume, of course, that Bob knows the decryption function dk .
(b) If the plaintexts and the ciphertexts of the original cryptosystem have the
same length, what is the message expansion ratio of the new probabilistic
cryptosystem?
(c) More generally, if the original cryptosystem has a message expansion
ratio of µ, what is the message expansion ratio of the new probabilistic
cryptosystem?
Solution to Exercise 3.42.
(a) Bob decrypts c to recover m = dk (c). He splits m up into two pieces
m = r k s, where r consists of the first b bits of m and s consists of the
last b bits of m. Then he recovers Alice’s plaintext m0 by computing r ⊕ s.
(b) The new probabilistic cryptosystem has plaintext length b bits and ciphertext length 2b bits, so its message expansion ratio is 2.
(c) The plaintexts in the original cryptosystem have length 2b bits, and it
has message expansion µ, so its ciphertexts have length 2bµ bits. The new
probabilistic cryptosystem has plaintext length b bits, so its message expansion
ratio is 2bµ/b = 2µ.

Chapter 4

Combinatorics, Probability,
and Information Theory
Exercises for Chapter 4
Section. Basic principles of counting
4.1. The Rhind papyrus is an ancient Egyptian mathematical manuscript
that is more than 3500 years old. Problem 79 of the Rhind papyrus poses a
problem that can be paraphrased as follows: there are seven houses; in each
house lives seven cats; each cat kills seven mice; each mouse has eaten seven
spelt seeds1 ; each spelt seed would have produced seven hekat2 of spelt. What
is the sum of all of the named items? Solve this 3500 year old problem.
Solution to Exercise 4.1.
7 + |{z}
72 + |{z}
73 + |{z}
74 + |{z}
75 = 19607.
|{z}
houses

cats

mice

spelt

hekat

As stated in the Rhind papyrus, the problem and solution looks more or
less as follows:
1
2
4

2,801
5,602
11,204
Total 19,607

1 Spelt
2A

houses
cats
mice
spelt
hekat
Total

7
49
343
2,301
16,807
19,607

is an ancient type of wheat.
1
of a cubic cubit, which is approximately 4.8 liters.
hekat is 30

97

98

Exercises for Chapter 4

Notice that the author has made a mistake in the value of 74 = 2401, but
that his final answer is correct. The last column in the Rhind papyrus is
the same as our solution, adding up powers of 7. In the first column the
author gives an alternative computational method based on the fact that
2801 = 1 + 7 + 72 + 73 + 74 . Thus he computes
7 + 72 + 73 + 74 + 75 = 7 · (1 + 7 + 72 + 73 + 74 )
= (1 + 2 + 4) · (1 + 7 + 72 + 73 + 74 )
= 2801 + 2 · 2801 + 4 · 2801.
This double-and-add method is very reminiscent of many modern algorithms.
4.2. (a) How many n-tuples (x1 , x2 , . . . , xn ) are there if the coordinates are
required to be integers satisfying 0 ≤ xi < q?
(b) Same question as (a), except now there are separate bounds 0 ≤ xi < qi
for each coordinate.
(c) How many n-by-n matrices are there if the entries xi,j of the matrix are
integers satisfying 0 ≤ xi,j < q?
(d) Same question as (a), except now the order of the coordinates does not
matter. So for example, (0, 0, 1, 3) and (1, 0, 3, 0) are considered the same.
(This one is rather tricky.)
(e) Twelve students are each taking four classes, for each class they need two
loose-leaf notebooks, for each notebook they need 100 sheets of paper, and
each sheet of paper has 32 lines on it. Altogether, how many students,
classes, notebooks, sheets, and lines are there? (Bonus. Make this or a
similar problem of your own devising into a rhyme like the St. Ives riddle.)
Solution to Exercise 4.2.
(a) There are q choices for each coordinate, so a total of q n possible ntuples.
(b) Now there are q1 choices for x1 , and q2 choices for x2 , and so on. Hence
the total number of possibilities is the product q1 q2 · · · qn .
(c) This is the same as (a), except now there are n2 entries to be filled in.
2
So there are q n possible matrices.
(d) The idea is to count the quantity of each number that appears. Say there
are k0 zeros, k1 ones, etc. Then k0 + k1 + · · · + kq−1 = n, so we need to count
the number
q nonnegative
pieces. The answer
¡ of ways
¢ to split n into a sum ¡ofq+n−1
¢
to this is q+n−1
,
which
is
also
equal
to
.
q−1
n
(e) The total number of students, classes, notebooks, sheets, and lines is
307200 = |{z}
12 · |{z}
4 ·

2
|{z}

· |{z}
100 · |{z}
32 .

students classes notebooks sheets

lines

4.3. (a) List all of the permutations of the set {A, B, C}.
(b) List all of the permutations of the set {1, 2, 3, 4}.

Exercises for Chapter 4

99

(c) How many permutations are there of the set {1, 2, . . . , 20}?
(d) Seven students are to be assigned to seven dormitory rooms, each student
receiving his or her own room. In how many ways can this be done?
(e) How many different words can be formed with the four symbols A, A, B, C?
Solution to Exercise 4.3.
(a)
(A, B, C), (A, C, B), (B, A, C), (B, C, A), (C, A, B), (C, B, A).
(b) There are 24 permutations of {1, 2, 3, 4}. They are
(1,2,3,4) (1,2,4,3) (1,3,2,4) (1,3,4,2) (1,4,2,3) (1,4,3,2)
(2,1,3,4) (2,1,4,3) (2,3,1,4) (2,3,4,1) (2,4,1,3) (2,4,3,1)
(3,1,2,4) (3,1,4,2) (3,2,1,4) (3,2,4,1) (3,4,1,2) (3,4,2,1)
(4,1,2,3) (4,1,3,2) (4,2,1,3) (4,2,3,1) (4,3,1,2) (4,3,2,1)
(c) There are 20! = 2432902008176640000 ≈ 2.43 · 1018 permutations of
{1, 2, . . . , 20}.
(d) If the rooms are labeled 1, 2, . . . , 7, then each permutation of the students
gives a way of assigning rooms, by putting the first listed student in room #1,
the second listed student in room #2, etc. So there are 7! = 5040 ways to
assign rooms.
(e) There are 4 choices for placement of B, then 3 choices for placement of C,
after which the two A’s go in the remaining places, so there are 12 words.
4.4. (a) List the 24 possible permutations of the letters A1 , A2 , B1 , B2 . If A1 is
indistinguishable from A2 , and B1 is indistinguishable from B2 , show how
the permutations become grouped into 6 distinct letter arrangements,
each containing 4 of the original 24 permutations.
(b) Using the seven symbols A, A, A, A, B, B, B, how many different seven
letter words can be formed?
(c) Using the nine symbols A, A, A, A, B, B, B, C, C, how many different nine
letter words can be formed?
(d) Using the seven symbols A, A, A, A, B, B, B, how many different five letter
words can be formed?
Solution to Exercise 4.4.
(a) Here are the 24 permutations.
(A1 , A2 , B1 , B2 ) (A1 , A2 , B2 , B1 ) (A2 , A1 , B1 , B2 ) (A2 , A1 , B2 , B1 )
(A1 , B1 , A2 , B2 ) (A1 , B2 , A2 , B1 ) (A2 , B1 , A1 , B2 ) (A2 , B2 , A1 , B1 )
(A1 , B1 , B2 , A2 ) (A1 , B2 , B1 , A2 ) (A2 , B1 , B2 , A1 ) (A2 , B2 , B1 , A1 )
(B1 , B2 , A2 , A1 ) (B1 , B2 , A1 , A2 ) (B2 , B1 , A1 , A2 ) (B2 , B1 , A2 , A1 )
(B1 , A1 , A2 , B2 ) (B1 , A2 , A1 , B2 ) (B2 , A1 , A2 , B1 ) (B2 , A2 , A1 , B1 )
(B1 , A1 , B2 , A2 ) (B1 , A2 , B2 , A1 ) (B2 , A1 , B1 , A2 ) (B2 , A2 , B1 , A1 )
If A1 = A2 and B1 = B2 , then the four entries in each row become the
same.

100

Exercises for Chapter 4

(b) We need to pick 4 of the 7 spots
¡ ¢ for the A’s, then the B’s go into the
remaining 3 spots. Hence there are 74 = 35 such words.
(c) We need to pick 4 of the 9 spots for the A’s, then we need to pick 3 of
the remaining 5 spots
¡ ¢¡ ¢for the B’s, then the C’s go into the remaining 2 spots.
Hence there are 94 53 = 126 · 10 = 1260 such words.
(d) We can form five letter words using anywhere from two to four A’s. So
we need to count the number of five letter words using each of
{A, A, A, A, B},
So there are

{A, A, A, B, B},

µ ¶ µ ¶ µ ¶
5
5
5
+
+
= 25
4
3
2

and

{A, A, B, B, B}.

different five letter words.

4.5. (a) There are 100 students eligible for an award, and the winner gets
to choose from among 5 different possible prizes. How many possible
outcomes are there?
(b) Same as in (a), but this time there is a first place winner, a second place
winner, and a third place winner, each of whom gets to select a prize.
However, there is only one of each prize. How many possible outcomes
are there?
(c) Same as in (b), except that there are multiple copies of each prize, so
each of the three winners may choose any of the prizes. Now how many
possible outcomes are there? Is this larger or smaller than your answer
from (b)?
(d) Same as in (c), except that rather than specifying a first, second, and
third place winner, we just choose three winning students without differentiating between them. Now how many possible outcomes are there?
Compare the size of your answers to (b), (c), and (d).
Solution to Exercise 4.5.
(a) There are 100 · 5 = 500 outcomes.
(b) This can be split into first choosing the three winners (in order), which
can be done in 100 · 99 · 98 ways, and then choosing the three prizes (in order),
which can be done in 5 · 4 · 3 ways. Then using the basic counting principle,
the total number of outcomes is
100 · 99 · 98 · 5 · 4 · 3 = 58212000 ≈ 107.77 .
(b) This time there are 5 · 5 · 5 ways to choose the prizes, so the total number
of outcomes is
100 · 99 · 98 · 5 · 5 · 5 = 121275000 ≈ 108.08 .
¡ ¢
(c) Since the order of the students does not matter, there are now 100
=
3
100·99·98
ways
to
choose
the
students.
Hence
the
total
number
of
outcomes
is
3!

Exercises for Chapter 4

101

100 · 99 · 98
· 5 · 5 · 5 = 20212500 ≈ 107.31 .
3!

4.6. Use the binomial theorem (Theorem 4.10) to compute each of the following quantities.
(a) (5z + 2)3
(b) (2a − 3b)4
(c) (x − 2)5
Solution to Exercise 4.6.
(a) (5z + 2)3 = 125z 3 + 225z 2 + 135z + 27.
(b) (2a − 3b)4 = 16a4 − 96a3 b + 216a2 b2 − 216ab3 + 81b4 .
(c) (x − 2)5 = x5 − 10x4 + 40x3 − 80x2 + 80x − 32.
4.7. The binomial coefficients satisfy many interesting identities. Give three
proofs of the identity
µ ¶ µ
¶ µ
¶
n
n−1
n−1
=
+
.
j
j−1
j
¡ ¢
n!
(a) For Proof #1, use the definition of nj as (n−j)!j!
.
(b) For Proof #2, use the binomial theorem (Theorem 4.10) and compare the
coefficients of xj y n−j on the two sides of the identity
(x + y)n = (x + y)(x + y)n−1 .
(c) For Proof #3, argue directly that choosing j objects from a set of n
objects can be decomposed into either choosing j − 1 objects from n − 1
objects or choosing j objects from n − 1 objects.
Solution to Exercise 4.7.
Proof #1:
µ
¶ µ
¶
(n − 1)!
n−1
n−1
(n − 1)!
+
+
=
(n − j)!(j − 1)! (n − 1 − j)!j!
j−1
j
·
¸
(n − 1)!
1
1
=
+
(n − 1 − j)!(j − 1)! n − j
j
(n − 1)!
n
=
·
(n − 1 − j)!(j − 1)! (n − j)j
n!
=
(n − j)!j!
µ ¶
n
=
.
j
Proof #2: Expand both sides of (x + y)n = (x + y)(x + y)n−1 using the
binomial theorem:

102

Exercises for Chapter 4
n µ ¶
X
n
j=0

j

j n−j

x y

= (x + y)

n−1
Xµ

¶
n − 1 j n−1−j
x y
j

j=0

=

n−1
Xµ
j=0

¶
¶
n−1 µ
n − 1 j+1 n−1−j X n − 1 j n−j
x y
+
x y
j
j
j=0

¶
¶
n µ
n−1 µ
X
n − 1 j n−j X n − 1 j n−j
=
x y
+
x y
j−1
j
j=1
j=0
n

=x +

n−1
X ·µ
j=1

¶ µ
¶
¸
n−1
n − 1 j n−j
+
x y
+ yn .
j−1
j

Comparing the coefficients of xj y n−j on the two sides gives the desired identity.
Another way to illustrate the same proof is to write the expansion of (x +
y)n for n = 0, 1, 2, 3, . . . in the form of a triangle called Pascal’s triangle.
Proof #3: Let the n objects be A1 , · · · , An . Treat the last one as special, so
label them as A1 , . . . , An−1 , B. In choosing j of these n objects, there are two
possibilities, namely either B is chosen
¡
¢or it is not chosen. The number of ways
to choose j objects without B is n−1
, since we are choosing j objects from
j
among
the
n
−
1
A’s.
The
number
of
ways
to choose j objects including B
¡
¢
is n−1
,
since
having
already
selected
B,
we
are need to choose j − 1 objects
j−1
from among the n − 1 A’s.
4.8. Let p be a prime number. This exercise sketches another proof of Fermat’s
little theorem (Theorem 1.25).
¡ ¢
(a) If 1 ≤ j ≤ p − 1, prove that the binomial coefficient pj is divisible by p.
(b) Use (a) and the binomial theorem (Theorem 4.10) to prove that
(a + b)p ≡ ap + bp

(mod p)

for all a, b ∈ Z.

(c) Use (b) with b = 1 and induction on a to prove that ap ≡ a (mod p) for
all a ≥ 0.
(d) Use (c) to deduce that ap−1 ≡ 1 (mod p) for all a with gcd(p, a) = 1.
Solution to Exercise 4.8.
(a)
µ ¶
p
p(p − 1)(p − 2) · · · (p − j + 1)
=
.
j
j!
The denominator has no factors of p, so the p in the numerator does not
cancel.
(b)
p µ ¶
X
p j p−j
(a + b)p =
a b
≡ ap + bp (mod p),
j
j=0

Exercises for Chapter 4

103

since (a) tells us that the middle terms in the sum are all divisible by p.
(c) Suppose we know that ap ≡ a (mod p), which we do for the starting
value a = 0. Then using (b) we have
(a + 1)p ≡ ap + 1 ≡ a + 1

(mod p).

Hence the result is also true for a + 1. By induction, it is true for all a ≥ 0.
(d) If p - a, then we can multiply both sides of ap ≡ a (mod p) by a−1 mod p.
4.9. We know that there are n! different permutations of the set {1, 2, . . . , n}.
(a) How many of these permutations leave no number fixed?
(b) How many of these permutations leave at least one number fixed?
(c) How many of these permutations leave exactly one number fixed?
(d) How many of these permutations leave at least two numbers fixed?
For each part of this problem, give a formula or algorithm that can be used to
compute the answer for an arbitrary value of n, and then compute the value
for n = 10 and n = 26. (This exercise generalizes Exercise 1.5.)
Solution to Exercise 4.9.
Let S(n, k) denote the number of permutations of n elements that fix at
least k elements, let R(n, k) denote the number of permutations of n elements that fix exactly k elements, and let !n (the subfactorial of n) denote
the number of permutations of n elements that fix no elements (such permutations are called derangements). Notice that !n = R(n, 0). See the solution
to Exercise exercise:derangement for the derivation of the following formulas:
!n = n!

n
X
(−1)k
k=0

R(n, k) =
S(n, k) =

k!

= bn!/ee,

µ ¶
µ ¶¹
¼
n
n
(n − k)!
!(n − k) =
,
k
k
e
n
X
j=k

R(n, j) = n! −

k−1
X

R(n, j).

j=0

(a) No letters fixed is R(n, 0) =!n. This is called the nth derangement number.
For n = 10 we get
R(10, 0) =!10 = b10!/ee = b1334960.916e = 1334961.
For n = 26 we get
R(26, 0) =!26 = b26!/ee = b148362637348470135821287824.964e
= 148362637348470135821287825.
(b) At least one letter fixed is n! minus no letters fixed, so

104

Exercises for Chapter 4
S(n, 1) = n! − R(n, 0) = n!−!n = n! − bn!/ee.

Hence
S(10, 1) = 10! − b10!/ee = 2293839,
S(26, 1) = 26! − b26!/ee = 254928823778135499762712175.
(c) Exactly 1 letter fixed is
¹

¼
(n − 1)!
R(n, 1) = n·!(n − 1) = n
,
e
so

¼
9!
R(10, 1) = 10
= 1334960,
e
¹

¹

¼
25!
R(26, 1) = 26
= 148362637348470135821287824.
e
(d) At least two letters fixed is n! minus zero or one letters fixed, so
S(n, 1) = n! − R(n, 0) − R(1, 0) = n!−!n − n·!(n − 1)
= n! − bn!/ee − nb(n − 1)!/ee.
Hence
S(10, 1) = 10! − b10!/ee − 10 · b9!/ee = 958879,
S(26, 1) = 26! − b26!/ee − 26 · b25!/ee = 106566186429665363941424351.

Section. The Vigenère cipher
4.10. Encrypt each of the following Vigenère plaintexts using the given keyword and the Vigenère tableau (Table 4.1).
(a) Keyword: hamlet
Plaintext: To be, or not to be, that is the question.
(b) Keyword: fortune
Plaintext: The treasure is buried under the big W.
Solution to Exercise 4.10.
(a) Vigenère Keyword: hamlet
t o b e o r n o t t o b e
h a m l e t h a m l e t h
a o n p s k u o f e s u l
(b) Vigenère Keyword: fortune
t h e t r e a s u r e i s
f o r t u n e f o r t u n
y v v m l r e x i i x c f

t h a t i s t h e q u e s t i o n
a m l e t h a m l e t h a m l e t
t t l x b z t t p u n l s f t s g
b u r i e d u n d e r t h e b i g w
e f o r t u n e f o r t u n e f o r
f z f z x x h r i s i m b r f n u n

Exercises for Chapter 4

105

4.11. Decrypt each of the following Vigenère ciphertexts using the given keyword and the Vigenère tableau (Table 4.1).
(a) Keyword: condiment
Ciphertext: r s g h z b m c x t d v f s q h n i g q x r n b m
pdnsq smbtr ku
(b) Keyword: rabbithole
Ciphertext: k h f e q y m s c i e t c s i g j v p w f f b s q
moapx zcsfx epsox yenpk daicx
cebsm ttptx zooeq laflg kipoc
zswqm taujw ghboh vrjtq hu
Solution to Exercise 4.11.
(a) Vigenère Keyword:
Ciphertext: r s g h z
Keyword: c o n d i
Plaintext: p e t e r
Ciphertext: p d n s q
Keyword: n t c o n
Plaintext: c k l e d

condiment
b m c x t
m e n t c
p i p e r
s m b t r
d i m e n
p e p p e

d v f s q
o n d i m
p i c k e

h n i g q
e n t c o
d a p e c

x r n b m
n d i m e
k o f p i

k u
t c
r s

Plaintext. Peter Piper picked a peck of pickled peppers!
(b) Vigenère Keyword: rabbithole
Ciphertext: k h f e q y m s c i e t c s i g j v p w
Keyword: r a b b i t h o l e r a b b i t h o l e
Plaintext: t h e d i f f e r e n t b r a n c h e s

f f b s q
r a b b i
o f a r i

Ciphertext: m o a p x
Keyword: t h o l e
Plaintext: t h m e t

z c s f x
r a b b i
i c r e p

e p s o x
t h o l e
l i e d t

y e n p k
r a b b i
h e m o c

d a i c x
t h o l e
k t u r t

Ciphertext: c e b s m
Keyword: r a b b i
Plaintext: l e a r e

t t p t x
t h o l e
a m b i t

z o o e q
r a b b i
i o n d i

l a f l g
t h o l e
s t r a c

k i p o c
r a b b i
t i o n u

Ciphertext: z s w q m t a u j w g h b o h v r j t q h u
Keyword: t h o l e r a b b i t h o l e r a b b i t h
Plaintext: g l i f i c a t i o n a n d d e r i s i o n
Plaintext. The different branches of arithmetic, replied the Mock Turtle, are
ambition, distraction, uglification, and derision. (From Lewis Carroll’s Alice
in Wonderland.)
4.12. Explain how a cipher wheel with rotating inner wheel (see Figure 1.1
on page 3) can be used in place of a Vigeǹere tableau (Table 4.1) to perform
Vigenère encryption and decryption. Illustrate by describing the sequence of
rotations used to perform a Vigenère encryption with the keyword mouse.

106

Exercises for Chapter 4

Solution to Exercise 4.12.
A solution for this exercise is not currently available.
4.13. Let
s = “I am the very model of a modern major general.”
t = “I have information vegetable, animal, and mineral.”
(a) Make frequency tables for s and t.
(b) Compute IndCo(s) and IndCo(t).
(c) Compute MutIndCo(s, t).
Solution to Exercise 4.13.
A B C D E F G H I J K L M N O P Q R S T U V W X Y Z
Freq s 4 0 0 2 6 1 1 1 1 1 0 2 4 2 4 0 0 4 0 1 0 1 0 0 1 0
(a)
Freq t 8 1 0 1 4 1 1 1 5 0 0 3 3 5 2 0 0 2 0 2 0 2 0 0 0 0
(b) IC(s) = 0.0424 and IC(t) = 0.0544.
(c) M IC(s, t) = 0.0517
4.14. The following strings are blocks from a Vigenère encryption. It turns
out that the keyword contains a repeated letter, so two of these blocks were
encrypted with the same shift. Compute MutIndCo(si , sj ) for 1 ≤ i < j ≤ 3
and use these values to deduce which two strings were encrypted using the
same shift.
s1 = iwseesetftuonhdptbunnybioeatneghictdnsevi
s2 = qibfhroeqeickxmirbqlflgkrqkejbejpepldfjbk
s3 = iesnnciiheptevaireittuevmhooottrtaaflnatg
Solution to Exercise 4.14.
a b c d
Freq s1 1 2 1 2
(a)
Freq s2 0 4 1 1
Freq s3 4 0 1 0

e
6
5
5

f
1
3
1

g
1
1
1

h
2
1
2

i
4
3
5

j
0
3
0

k
0
4
0

l
0
3
1

m
0
1
1

n
5
0
3

o
2
1
3

p
1
2
1

q
0
4
0

r
0
3
2

s
3
0
1

t
5
0
7

u
2
0
1

v
1
0
2

w
1
0
0

x
0
1
0

y
1
0
0

z
0
0
0

MutIndCo(s1 , s2 ) = 0.0375,
MutIndCo(s1 , s3 ) = 0.0744,
MutIndCo(s2 , s3 ) = 0.0369.
Thus s1 and s3 were probably encrypted using the same shift, so the first and
third letters of the keyword are probably the same.
4.15. (a) One of the following two strings was encrypted using a simple substitution cipher, while the other is a random string of letters. Compute
the index of coincidence of each string and use the results to guess which
is which.

Exercises for Chapter 4

107

s1 = RCZBWBFHSLPSCPILHBGZJTGBIBJGLYIJIBFHCQQFZBYFP,
s2 = KHQWGIZMGKPOYRKHUITDUXLXCWZOTWPAHFOHMGFEVUEJJ.
(b) One of the following two strings was encrypted using a simple substitution
cipher, while the other is a random permutation of the same set of letters.
s1 = NTDCFVDHCTHKGUNGKEPGXKEWNECKEGWEWETWKUEVHDKK
CDGCWXKDEEAMNHGNDIWUVWSSCTUNIGDSWKE
s2 = IGWSKGEHEXNGECKVWNKVWNKSUTEHTWHEKDNCDXWSIEKD
AECKFGNDCPUCKDNCUVWEMGEKWGEUTDGTWHD
Thus their Indices of Coincidence are identical. Develop a method to
compute a bigram index of coincidence, i.e., the frequency of pairs of
letters, and use it to determine which string is most likely the encrypted
text.
(Bonus: Decrypt the encrypted texts in (a) and (b), but be forewarned that
the plaintexts are in Latin.)
Solution to Exercise 4.15.
(a) The Indices of Coincidence of the two strings are IndCo(s1 ) = 0.0576
and IndCo(s2 ) = 0.0303, so most likely s1 is the encrypted text and s2 is
the random string. The plaintext for s1 is “Facilius per partes in cognitionem
totius adducimur,” which translated into English says “We are more easily led
part by part to an understanding of the whole.” The phrase is due to Seneca.
(b) The Indices of Coincidence are identical, IndCo(s1 ) = IndCo(s2 ) =
0.0672. In general, let A = {a1 , a2 , . . . , ak } be a set of distinct objects (letters,
bigrams, turtles, etc.) and let B = (b1 , b2 , . . . , bn ) be a list of elements from A,
where the bi do not need to be distinct. For each 1 ≤ i ≤ k, let Fi denote
the number of b’s that are equal to ai , i.e., Fi is the frequency with which ai
appears in the list B. Then the index of coincidence of the set B is
k

IndCo(B) =

X
1
Fi (Fi − 1).
n(n − 1) i=1

So now we can apply the theory of Index of Coincidence to the set of
bigrams that appear in a string. And we would expect that the index should
be higher for the string that is the encrypted message and lower for the string
with the same letters, but randomly rearranged. We find that
IndCo(Bigrams in s1 ) = 0.004,
IndCo(Bigrams in s2 ) = 0.010.
Thus it seems likely that the second string s2 is the encrypted plaintext.
This is the case, and the plaintext for s2 is “Frustra laborant quotquot se
calculationibus fatigant pro inventione quadraturae circuli,” which translated
into English says “Futile is the labor of those who fatigue themselves with
calculations to square the circle.” The phrase is due to Michael Stifel (1544).

108

Exercises for Chapter 4

nhqrk
bkkcj
mrkwn
dxjfg
lzavs
fkwis
qyhdv
cuwwz
trggr
mvrgw

vvvfe
vqazx
nsuhy
nywus
hyigh
tfylk
rhhny
rgusl
dxfgs
kmirt

fwgjo
wnvll
iecru
rwoar
rvwpn
ysnir
wqhyi
zgfhy
ceyts
twfer

mzjgc
zetjc
ljjvs
xhvvx
ljazl
rddpb
rjdqm
etfre
tiiih
oimsb

kocgk
zwgqz
qlvvw
ssmja
nispv
svsux
iwutf
ijjvy
vjjvt
qgrgc

lejrj
zwhah
zzxyv
vkrwt
jahym
zjgqk
nkzgd
ghfau
tcxfj

wossy
kwdxj
woenx
uhktm
ntewj
xouhs
vvibg
wvwtn
hciiv

wgvkk
fgnyw
ujgyr
malcz
jvrzg
zzrjj
oenwb
xlljv
voaro

hnesg
gdfgh
kqbfj
ygrsz
qvzcr
kyiwc
kolca
vywyj
lrxij

kwebi
bitig
lvjzx
xwnvl
estul
zckov
mskle
apgzw
vjnok

Table 4.1: A Vigenère ciphertext for Exercise 4.16
4.16. Table 4.13 is a Vigenère ciphertext in which we have marked some of
the repeated trigrams for you. How long do you think the keyword is? Why?
Bonus: Complete the cryptanalysis and recover the plaintext.
Solution to Exercise 4.16.
Trigram
hyi

Appears at places
109, 206 and 313

jjv

117, 235, 372, and 422

nyw

88, 156, and 309

Differences
97 = 97
107 = 107
204 = 22 · 3 · 17
118 = 2 · 59
137 = 137
255 = 3 · 5 · 17
305 = 5 · 61
187 = 11 · 17
50 = 2 · 52
68 = 22 · 17
221 = 13 · 17
153 = 32 · 17

The keyword has length 17 . The keyword used for encryption was
fourscoreandseven .
The plaintext is
It was the best of times, it was the worst of times, it was the age of
wisdom, it was the age of foolishness, it was the epoch of belief, it
was the epoch of incredulity, it was the season of Light, it was the
season of Darkness, it was the spring of hope, it was the winter of
despair, we had everything before us, we had nothing before us,
we were all going direct to Heaven, we were all going direct the
other way—in short, the period was so far like the present period,

Exercises for Chapter 4
togmg
phumt
thkef
gfwsl
asgvs

gbymk
whlsf
fwptj
jsfse
fwrqs

kcqiv
yovww
ggviv
ooqhw
fsfvq

109

dmlxk
knhhm
cgdra
tofsh
rhdrs

kbyif
rcqfq
pgwvm
aciin
nmvmk

vcuek
vvhkw
osqxg
gfbif
cbhrv

cuuis
psued
hkdvt
gabgj
kblxk

vvxqs
ugrsf
whuev
adwsy
gzi

pwwej
ctwij
kcwyj
topml

koqgg
khvfa
psgsn
ecqzw

Table 4.2: A Vigenère ciphertext for Exercise 4.17
Blocks
i
j
1
2
1
3
1
4
1
5
2
3
2
4
2
5
3
4
3
5
4
5

0
.044
.038
.025
.050
.035
.040
.042
.032
.043
.045

1
.047
.031
.039
.050
.037
.033
.040
.033
.043
.033

2
.021
.027
.053
.025
.039
.046
.042
.035
.040
.044

3
.054
.037
.043
.031
.031
.031
.029
.049
.034
.046

4
.046
.045
.023
.038
.031
.033
.033
.053
.033
.021

Shift Amount
5
6
7
.038 .022 .034
.036 .034 .032
.035 .032 .043
.045 .037 .028
.035 .047 .048
.023 .052 .027
.035 .035 .038
.027 .030 .022
.034 .043 .035
.032 .030 .038

8
.057
.039
.029
.032
.034
.031
.037
.047
.026
.047

9
.035
.039
.040
.038
.031
.039
.057
.036
.030
.040

10
.040
.047
.041
.063
.031
.078
.039
.040
.050
.025

11
.023
.038
.050
.033
.067
.034
.038
.036
.068
.037

12
.038
.050
.027
.034
.053
.029
.040
.052
.044
.068

Blocks
i
j
1
2
1
3
1
4
1
5
2
3
2
4
2
5
3
4
3
5
4
5

13
.040
.026
.042
.030
.039
.027
.033
.040
.039
.049

14
.063
.046
.050
.048
.015
.048
.035
.048
.029
.033

15
.033
.042
.042
.039
.030
.050
.039
.041
.045
.029

16
.025
.053
.031
.030
.045
.037
.033
.044
.040
.043

17
.032
.027
.024
.034
.049
.032
.037
.033
.033
.028

Shift Amount
18
19
20
.055 .038 .030
.024 .040 .047
.052 .027 .051
.038 .042 .035
.037 .023 .036
.021 .035 .043
.047 .037 .028
.028 .039 .027
.028 .031 .037
.033 .020 .040

21
.032
.048
.020
.036
.030
.047
.034
.036
.038
.040

22
.045
.018
.037
.043
.049
.041
.066
.017
.036
.041

23
.035
.037
.042
.055
.039
.047
.054
.038
.033
.039

24
.030
.034
.069
.030
.050
.042
.032
.051
.051
.039

25
.044
.066
.031
.035
.037
.035
.022
.065
.036
.059

Table 4.3: Mutual indices of coincidence for Exercise 4.17
that some of its noisiest authorities insisted on its being received,
for good or for evil, in the superlative degree of comparison only.
These are the opening lines of A Tale of Two Cities by Charles Dickens.
4.17. We applied a Kasiski test to the Vigenère ciphertext listed in Table 4.14
and found that the key length is probably 5. We then performed a mutual
index of coincidence test to each shift of each pair of blocks and listed the
results for you in Table 4.15. (This is the same type of table as Table 4.5 in
the text, except that we haven’t underlined the large values.) Use Table 4.15
to guess the relative rotations of the blocks, as we did in Table 4.6. This
will give you a rotated version of the keyword. Try rotating it, as we did in
Table 4.7, to find the correct keyword and decrypt the text.
Solution to Exercise 4.17.
The table of likely shift relations gives

110

Exercises for Chapter 4
i
2
2
3
4
1
1
2
3

j
3
4
5
5
3
4
5
4

Shift
11
10
11
12
25
24
22
25

MutIndCo
0.067
0.078
0.068
0.068
0.066
0.069
0.066
0.065

Shift Relation
β2 − β3 = 11
β2 − β4 = 10
β3 − β5 = 11
β4 − β5 = 12
β1 − β3 = 25
β1 − β4 = 24
β2 − β5 = 22
β3 − β4 = 25

Table 4.4: Large indices of coincidence and shift relations
Shift
0
1
2
3
4
5
6
7
8
.
.
.

Keyword
AMBCQ
BNCDR
CODES
DPEFT
EQFGU
FRGHV
GSHIW
HTIJX
IUJKY
.
.
.

Decrypted Text
tcfkqgpxkukqpgfdakvukpxgpvqtcucitgcvjwocpk
sbejpfowjtjpofeczjutjowfoupsbtbhsfbuivnboj
radioenvisionedbyitsinventorasagreathumani
qzchndmuhrhnmdcaxhsrhmudmsnqzrzfqdzsgtlzmh
pybgmcltgqgmlcbzwgrqgltclrmpyqyepcyrfskylg
oxaflbksfpflkbayvfqpfksbkqloxpxdobxqerjxkf
nwzekajreoekjazxuepoejrajpknwowcnawpdqiwje
mvydjziqdndjizywtdondiqziojmvnvbmzvocphvid
luxciyhpcmcihyxvscnmchpyhnilumualyunboguhc
.
.
.

Table 4.5: Decryption using shifts of the keyword AJCHWJZ
β2 = β1 + 12,

β3 = β1 + 1,

β4 = β1 + 2,

β5 = β1 + 16.

Hence the keyword is a rotation of AMBCQ. The table lists the rotations of
this word with the corresponding decryptions. We see immediately that the
keyword is CODES. The full plaintext reads as follows:
Radio, envisioned by its inventor as a great humanitarian contribution, was seized upon by the generals soon after its birth and
impressed as an instrument of war. But radio turned over to the
commander a copy of every enemy cryptogram it conveyed. Radio
made cryptanalysis an end in itself.
The Code-Breakers, Chapter 10, 1967, David Kahn
4.18. Table 4.16 gives a Vigenère ciphertext for you to analyze from scratch.
It is probably easiest to do so by writing a computer program, but you are
welcome to try to decrypt it with just paper and pencil.
(a) Make a list of matching trigrams as we did in Table 4.3. Use the Kasiski
test on matching trigrams to find the likely key length.

Exercises for Chapter 4
mgodt
uuqtu
uaafv
psgki
igxhr
xsyor

beida
habxw
eqgjo
puxfb
oyazd
tcylf

psgls
dgkie
ewabz
uxfuq
rakce
egcy

111

akowu
ktsnp
saawl
cvymy
dxeyr

hxukc
sekld
rzjpv
okagl
pdobr

iawlr
zlvnh
feyky
sactt
buehr

csoyh
wefss
gylwu
uwlrx
uwcue

prtrt
glzrn
btlyd
psgiy
ekfic

udrqh
peaoy
kroec
ytpsf
zehrq

cengx
lbyig
bpfvt
rjfuw
ijezr

Table 4.6: A Vigenère ciphertext for Exercise 4.18
Trigram
awl
ehr
gki
gls
lsa
psg
sgl
tps
uxf
wlr

Appears at places
27 and 118
228 and 242
62 and 153
13 and 174
14 and 175
11 and 151 and 186
12 and 85
150 and 192
157 and 161
28 and 119 and 182

Difference
91
14
91
161
161
140 and 35
73
42
4
91 and 63

Table 4.7: Repeated trigrams in the ciphertext
(b) Make a table of indices of coincidence for various key lengths, as we did
in Table 4.4. Use your results to guess the probable key length.
(c) Using the probable key length from (a) or (b), make a table of mutual
indices of coincidence between rotated blocks, as we did in Table 4.5. Pick
the largest indices from your table and use them to guess the relative
rotations of the blocks, as we did in Table 4.6.
(d) Use your results from (c) to guess a rotated version of the keyword, and
then try the different rotations as we did in Table 4.7 to find the correct
keyword and decrypt the text.
Solution to Exercise 4.18.
A list of repeated trigrams for the Kasiski test is given in the Table. The
list of differences (sorted) is
{4, 14, 35, 42, 63, 73, 91, 91, 91, 140, 154, 161, 161, 175}.
Thus a good guess for the period is 7.
Solving the relations in the table gives
β2 = β1 + 9,

β3 = β1 + 2,

β4 = β1 + 7,

β5 = β1 + 22,

β6 = β1 + 9.

(There is actually one erroneous relation, namely β2 −β5 = 24, but our solution
satisfies the other 10 relations, which makes it likely that it is correct.) In order

112

Block
Size
4
5
6
7
8
9

Exercises for Chapter 4

Average
Index
0.043
0.044
0.042
0.060
0.046
0.041

0.038,
0.048,
0.036,
0.058,
0.042,
0.041,

0.043,
0.052,
0.050,
0.060,
0.051,
0.053,

0.042,
0.046,
0.042,
0.081,
0.030,
0.042,

Individual Indices
of Coincidence
0.046
0.030, 0.041
0.051, 0.038, 0.035
0.054, 0.059, 0.065, 0.047
0.053, 0.040, 0.051, 0.057, 0.040
0.037, 0.052, 0.030, 0.054, 0.030, 0.030

Table 4.8: Index of coincidence for various block sizes

Blocks
i
j
1
2
1
3
1
4
1
5
1
6
1
7
2
3
2
4
2
5
2
6
2
7
3
4
3
5
3
6
3
7
4
5
4
6
4
7
5
6
5
7
6
7

0
.037
.020
.038
.053
.024
.048
.040
.038
.041
.067
.030
.033
.034
.050
.033
.053
.020
.040
.022
.039
.032

1
.035
.046
.031
.037
.027
.055
.035
.045
.038
.034
.030
.048
.040
.042
.025
.040
.037
.038
.045
.034
.029

2
.043
.035
.045
.027
.048
.038
.026
.070
.051
.028
.047
.035
.054
.022
.032
.036
.041
.030
.050
.028
.052

3
.037
.041
.035
.034
.044
.036
.046
.029
.032
.050
.031
.033
.042
.029
.059
.021
.040
.028
.031
.038
.049

4
.045
.046
.039
.065
.039
.033
.039
.034
.021
.048
.035
.044
.026
.047
.038
.042
.043
.052
.034
.044
.028

Shift Amount
5
6
7
.035
.053
.046
.030
.033
.039
.030
.046
.043
.048
.038
.036
.043
.043
.040
.031
.037
.047
.027
.051
.071
.044
.035
.037
.028
.043
.025
.027
.036
.045
.035
.056
.031
.046
.040
.023
.026
.056
.042
.038
.036
.033
.039
.028
.037
.032
.031
.038
.041
.031
.015
.032
.041
.041
.053
.047
.023
.020
.039
.050
.037
.035
.031

8
.035
.037
.050
.048
.024
.041
.022
.042
.031
.028
.034
.044
.036
.041
.033
.035
.030
.058
.037
.057
.031

9
.034
.033
.041
.028
.036
.023
.026
.038
.049
.023
.051
.028
.032
.041
.053
.033
.049
.029
.044
.028
.058

10
.046
.040
.026
.022
.053
.035
.062
.030
.039
.034
.048
.048
.046
.030
.039
.038
.043
.030
.030
.035
.055

11
.030
.048
.035
.036
.043
.041
.033
.042
.031
.056
.031
.037
.031
.030
.026
.058
.035
.036
.024
.050
.024

12
.024
.036
.039
.044
.039
.049
.039
.037
.044
.031
.033
.037
.035
.038
.039
.045
.030
.045
.044
.038
.033

Blocks
i
j
1
2
1
3
1
4
1
5
1
6
1
7
2
3
2
4
2
5
2
6
2
7
3
4
3
5
3
6
3
7
4
5
4
6
4
7
5
6
5
7
6
7

13
.047
.033
.035
.031
.046
.040
.035
.050
.064
.025
.043
.027
.028
.047
.043
.036
.038
.044
.065
.034
.036

14
.047
.051
.037
.028
.038
.040
.038
.029
.041
.038
.034
.034
.050
.049
.046
.032
.035
.037
.031
.033
.042

15
.021
.030
.058
.054
.014
.048
.029
.045
.032
.034
.033
.033
.033
.042
.035
.044
.036
.037
.021
.050
.032

16
.030
.047
.039
.052
.033
.038
.042
.035
.033
.036
.036
.042
.035
.050
.041
.038
.041
.021
.039
.028
.048

17
.070
.044
.021
.036
.066
.026
.037
.039
.044
.028
.041
.048
.054
.026
.041
.037
.047
.041
.042
.028
.050

Shift Amount
18
19
20
.043
.030
.046
.032
.026
.055
.036
.062
.042
.040
.049
.041
.039
.024
.043
.023
.053
.041
.044
.023
.044
.033
.026
.033
.025
.020
.038
.043
.047
.038
.023
.048
.044
.055
.022
.021
.054
.036
.033
.018
.065
.048
.042
.027
.029
.033
.040
.029
.048
.034
.038
.038
.053
.037
.028
.028
.058
.035
.045
.030
.025
.032
.037

21
.038
.031
.036
.022
.036
.033
.035
.033
.037
.038
.056
.073
.040
.027
.039
.029
.046
.043
.035
.042
.056

22
.028
.016
.042
.032
.021
.031
.040
.039
.037
.048
.031
.039
.043
.050
.047
.053
.041
.032
.024
.053
.035

23
.030
.046
.024
.039
.036
.053
.049
.028
.037
.048
.042
.033
.031
.041
.036
.048
.038
.041
.043
.056
.030

24
.039
.080
.033
.030
.055
.030
.033
.054
.072
.026
.038
.042
.024
.024
.033
.046
.063
.033
.057
.029
.045

25
.029
.024
.039
.030
.041
.029
.033
.036
.048
.043
.039
.035
.041
.035
.059
.034
.043
.042
.042
.037
.038

Table 4.9: Mutual indices of coincidence for shifted blocks

Exercises for Chapter 4
i
1
2
2
2
1
1
1
2
3
3
5

j
5
3
4
6
2
3
6
5
4
6
6

Shift
4
7
2
0
17
24
17
24
21
19
13

113
MutIndCo
0.065
0.071
0.070
0.067
0.070
0.080
0.066
0.072
0.073
0.065
0.065

Shift Relation
β1 − β5 = 4
β2 − β3 = 7
β2 − β4 = 2
β2 − β6 = 0
β1 − β2 = 17
β1 − β3 = 24
β1 − β6 = 17
β2 − β5 = 24
β3 − β4 = 21
β3 − β6 = 19
β5 − β6 = 13

Table 4.10: Large indices of coincidence and shift relations
to find β7 , we look at the mutual indices that involve Block 7 and are greater
than 0.058. There are three of them:
i
3
3
4

j
7
7
7

Shift
3
25
8

MutIndCo
0.059
0.059
0.058

Shift Relation
β3 − β7 = 3
β3 − β7 = 25
β4 − β7 = 8

Only one of the first two can be correct, but the third yields β7 = β1 + 25.
This agrees with β7 = β3 − 3 = 25, so is probably correct. Thus the amounts
that Blocks 2 through 7 are rotated exceed the amount that Block 1 is rotated
by 9, 2, 7, 22, 9, and 25, respectively. For example, if the first letter of the
keyword is A, then the full keyword is AJCHWJZ. The shifts of this keyword
and decryptions are listed in the table.
We find that the keyword is ENGLAND, and the full plaintext reads as follows:
It is to be questioned whether in the whole length and breadth
of the world there is a more admirable spot for a man in love to
pass a day or two than the typical English village. It combines the
comforts of civilization with the restfulness of solitude in a manner
equalled by no other spot except the New York Public Library.
A Damsel in Distress, 1919, P.G. Wodehouse
4.19. The autokey cipher is similar to the Vigenère cipher, except that rather
than repeating the key, it simply uses the key to encrypt the first few letters
and then uses the plaintext itself (shifted over) to continue the encryption. For
example, in order to encrypt the message “The autokey cipher is cool”
using the keyword random, we proceed as follows:

114

Exercises for Chapter 4

Shift
0
1
2
3
4
5
6
7
8
9
.
.
.

Keyword
AJCHWJZ
BKDIXKA
CLEJYLB
DMFKZMC
ENGLAND
FOHMBOE
GPINCPF
HQJODQG
IRKPERH
JSLQFSI
.
.
.

Decrypted Text
mxmwxsfiuyiwxmsrihalixlivmrxlialspipirkxle
lwlvwrehtxhvwlrqhgzkhwkhulqwkhzkrohohqjwkd
kvkuvqdgswguvkqpgfyjgvjgtkpvjgyjqngngpivjc
jujtupcfrvftujpofexifuifsjouifxipmfmfohuib
itistobequestionedwhetherinthewholelengtha
hshrsnadptdrshnmdcvgdsgdqhmsgdvgnkdkdmfsgz
grgqrmzcoscqrgmlcbufcrfcpglrfcufmjcjclerfy
fqfpqlybnrbpqflkbatebqebofkqebtelibibkdqex
epeopkxamqaopekjazsdapdanejpdasdkhahajcpdw
dodnojwzlpznodjizyrczoczmdioczrcjgzgzibocv
.
.
.

Table 4.11: Decryption using shifts of the keyword AJCHWJZ
Plaintext
t h e a u t o k e y c i p h e r i s c o o l
Key
r a n d o m t h e a u t o k e y c i p h e r
Ciphertext k h r d i f h r i y w b d r i p k a r v s c
The autokey cipher has the advantage that different messages are encrypted
using different keys (except for the first few letters). Further, since the key does
not repeat, there is no key length, so the autokey is not directly susceptible
to a Kasiski or index of coincidence analysis. A disadvantage of the autokey
is that a single mistake in encryption renders the remainder of the message
unintelligible. According to [58], Vigenère invented the autokey cipher in 1586,
but his invention was ignored and forgotten before being reinvented in the
1800s.
(a) Encrypt the following message using the autokey cipher:
Keyword: LEAR
Plaintext: Come not between the dragon and his wrath.
(b) Decrypt the following message using the autokey cipher:
Keyword:
CORDELIA
Ciphertext: pckkm yowvz ejwzk knyzv vurux cstri tgac
(c) Eve intercepts an autokey ciphertext and manages to steal the accompanying plaintext:
Plaintext
ifmusicbethefoodofloveplayon
Ciphertext azdzwqvjjfbwnqphhmptjsszfjci
Help Eve to figure out the keyword that was used for encryption. Describe
your method in sufficient generality to show that the autokey cipher is
susceptible to chosen plaintext attacks.
(d) Bonus Problem: Try to formulate a statistical or algebraic attack on the
autokey cipher, assuming that you are given a large amount of ciphertext
to analyze.

Exercises for Chapter 4

115

Solution to Exercise 4.19.
(a)
c o m e n o t b e t w e e n t h e d r a g o n a n d h i s w r a t h
l e a r c o m e n o t b e t w e e n t h e d r a g o n a n d h i s w
n s m v p c f f r h p f i g p l i q k h k r e a t r u i f z y i l d
The ciphertext is nsmvp cffrh pfigp liqkh kreat ruifz yild.
(b)
p c k k m y o w v z e j w z k k n y z v v u r u x c s t r i t g a c
c o r d e l i a n o t h i n g w i l l c o m e o f n o t h i n g s p
n o t h i n g w i l l c o m e o f n o t h i n g s p e a k a g a i n
The plaintext is Nothing will come of nothing. Speak again. These are
King Lear’s tragically inaccurate words to his youngest daughter Cordelia.
(c) The keyword is SURFEIT. The line is from Shakespeare’s Twelfth Night,
and the full encryption is
i f m u s i c b e t h e f o o d o f l o v e p l a y o n
s u r f e i t i f m u s i c b e t h e f o o d o f l o v
a z d z w q v j j f b w n q p h h m p t j s s z f j c i

116

Exercises for Chapter 4

Section. Probability theory
4.20. Use the definition (4.15) of the probability of an event to prove the
following basic facts about probability theory.
(a) Let E and F be disjoint events. Then
Pr(E ∪ F ) = Pr(E) + Pr(F ).
(b) Let E and F be events that need not be disjoint. Then
Pr(E ∪ F ) = Pr(E) + Pr(F ) − Pr(E ∩ F ).
(c) Let E be an event. Then Pr(E c ) = 1 − Pr(E).
(d) Let E1 , E2 , E3 be events. Prove that
Pr(E1 ∪ E2 ∪ E3 ) = Pr(E1 ) + Pr(E2 ) + Pr(E3 ) − Pr(E1 ∩ E2 )
− Pr(E1 ∩ E3 ) − Pr(E2 ∩ E3 ) + Pr(E1 ∩ E2 ∩ E3 ).
The formulas in (b) and (d) and their generalization to n events are known
as the inclusion–exclusion principle.
Solution to Exercise 4.20.
A solution for this exercise is not currently available.
4.21. We continue with the coin tossing scenario from Example 4.23, so our
experiment consists in tossing a fair coin ten times. Compute the probabilities
of the following events.
(a) The first and last tosses are both heads.
(b) Either the first toss or the last toss (or both) are heads.
(c) Either the first toss or the last toss (but not both) are heads.
(d) There are exactly k heads and 10 − k tails. Compute the probability for
each value of k between 0 and 10. (Hint. To save time, note that the
probability of exactly k heads is the same as the probability of exactly k
tails.)
(e) There is an even number of heads.
(f) There is an odd number of heads.
Solution to Exercise 4.21.
We label the events in the parts of this problem as E(a) , E(b) , E(c,k) , etc.
(a) Pr(E(a) ) = 14 .
c
(b) Pr(E(b) ) = 1 − Pr(E(b)
) = 1 − 14 = 43 .
(c) Pr(E(c) ) = Pr(E(b) ) − Pr(E(a) ) = 12 .
(d)
µ ¶
10
1
1
1
Pr(E(d,0) ) =
· 10 =
=
≈ 0.0010
0
2
1024
1024

Exercises for Chapter 4
µ ¶
10
Pr(E(d,1) ) =
·
1
µ ¶
10
Pr(E(d,2) ) =
·
2
µ ¶
10
Pr(E(d,3) ) =
·
3
µ ¶
10
Pr(E(d,4) ) =
·
4
µ ¶
10
Pr(E(d,5) ) =
·
5
µ ¶
10
Pr(E(d,6) ) =
·
6
µ ¶
10
Pr(E(d,7) ) =
·
7
µ ¶
10
Pr(E(d,8) ) =
·
8
µ ¶
10
Pr(E(d,9) ) =
·
9
µ ¶
10
Pr(E(d,10) ) =
·
10

117
1
210
1
210
1
210
1
210
1
210
1
210
1
210
1
210
1
210
1
210

=
=
=
=
=
=
=
=
=
=

10
1024
45
1024
120
1024
210
1024
252
1024
210
1024
120
1024
45
1024
10
1024
1
1024

=
=
=
=
=
=
=
=
=
=

5
≈ 0.0098
512
45
≈ 0.0439
1024
15
≈ 0.1172
128
105
≈ 0.2051
512
63
≈ 0.2461
256
105
≈ 0.2051
512
15
≈ 0.1172
128
45
≈ 0.0439
1024
5
≈ 0.0098
512
1
≈ 0.0010
1024

(e)
Pr(E(e) ) = Pr(Even number of heads) =

X

Pr(E(c,k) ) =

k even

1
.
2

(f) Pr(E(f) ) = Pr(E(d) ) = 21 .
4.22. Alice offers to make the following bet with you. She will toss a fair
coin 14 times. If exactly 7 heads come up, she will give you $4; otherwise you
must give her $1. Would you take this bet? If so, and if you repeated the bet
10000 times, how much money would you expect to win or lose?
Solution to Exercise 4.22.
The probability of winning the bet is
µ ¶
14
1
3432
429
· 1 =
=
≈ 0.2095.
7
2 4
16384
2048
Thus your probability of winning the bet is slightly larger than 15 , so it is
worthwhile making the bet. (Note that if the probability of winning were
exactly 51 , then in five trials you would expect to win once for plus $4 and
lose four times for minus $4, so you would end up even.) In 10000 trials, you

118

Exercises for Chapter 4

would expect to win the bet approximately 2095 times, for a gain of $8380,
and to lose the bet approximately 7905 times, for a loss of $7905. Hence your
average net gain for 10000 trials is $475.
4.23. Let E and F be events.
(a) Prove that Pr(E | E) = 1. Explain in words why this is reasonable.
(b) If E and F are disjoint, prove that Pr(F | E) = 0. Explain in words why
this is reasonable.
(c) Let F1 , . . . , Fn be events satisfying Fi ∩ Fj = ∅ for all i 6= j. We say
that F1 , . . . , Fn are pairwise disjoint. Prove then that
µ[
¶ X
n
n
Pr
Fi =
Pr(Fi ).
i=1

i=1

(d) Let F1 , . . . , Fn be pairwise disjoint as in (c), and assume further that
F1 ∪ · · · ∪ Fn = Ω,
where recall that Ω is the entire sample space. Prove the following general
version of the decomposition formula (4.20) in Proposition 4.24(a):
Pr(E) =

n
X

Pr(E | Fi ) Pr(Fi ).

i=1

(e) Prove a general version of Bayes’s formula:
Pr(Fi | E) =

Pr(E | Fi ) Pr(Fi )
.
Pr(E | F1 ) Pr(F1 ) + Pr(E | F2 ) Pr(F2 ) + · · · + Pr(E | Fn ) Pr(Fn )

Solution to Exercise 4.23.
Pr(E ∩ E)
Pr(E)
(a) Pr(E | E) =
=
= 1. It is clear that if we know
Pr(E)
Pr(E)
that E occurs, then the probability that E occurs is 1.
Pr(F ∩ E)
Pr(∅)
(b) Pr(F | E) =
=
= 0. If E occurs and F is disjoint
Pr(E)
Pr(E)
from E, then none of the individual events in F can possibly occur, so the
probability of F is clearly 0.
(c) One can argue directly by summing over the elements in the Fi ’s or use
induction on n, since we already know the formula for n = 2.
(d) The assumptions of F1 , . . . , Fn imply that
E=

n
[

(E ∩ Fi )

i=1

Hence

and

(E ∩ Fi ) ∩ (E ∩ Ej ) = ∅

for i 6= j.

Exercises for Chapter 4
Pr(E) = Pr

µ[
n

119

¶
(E ∩ Fi )

i=1

=
=

n
X
i=1
n
X

Pr(E ∩ Fi )

since the E ∩ Fi are disjoint from one another,

Pr(E | Fi ) Pr(Fi ).

i=1

4.24. There are two urns containing pens and pencils. Urn #1 contains three
pens and seven pencils and Urn #2 contains eight pens and four pencils.
(a) An urn is chosen at random and an object is drawn. What is the probability that it is a pencil?
(b) An urn is chosen at random and an object is drawn. If the object drawn
is a pencil, what is the probability that it came from Urn #1?
(c) If an urn is chosen at random and two objects are drawn simultaneously,
what is the probability that both are pencils?
Solution to Exercise 4.24.
Define events
E = {Urn #1 is selected},
F = {A pencil is selected}.
(a) We compute
Pr(F ) = Pr(F | E) Pr(E) + Pr(F | E c ) Pr(E c )
7 1
4 1
=
· +
·
10 2 12 2
31
=
≈ 0.517.
60
(b) We compute
Pr(F | E) Pr(E)
Pr(F )
(7/10) · (1/2)
=
31/60
21
=
≈ 0.677.
31

Pr(E | F ) =

Baye’s law,
using (a) to get Pr(F ),

(c) We need slightly different events, so we let
E = {Urn #1 is selected},
F = {First item selected is a pencil},
G = {Second item selected is a pencil}.

120

Exercises for Chapter 4

Then
Pr(F and G) = Pr(F ) Pr(G | F ).
We already know Pr(F ) = 31/60 from (a). To compute Pr(G | F ), we do a
calculation similar to the calculation in (a). Thus
Pr(G | F ) = Pr(G | F &E) Pr(F &E) + Pr(G | F &E c ) Pr(F &E c )
= Pr(G | F &E) Pr(F | E) Pr(E) + Pr(G | F &E c ) Pr(F | E c ) Pr(E c )
3 4 1
6 7 1
· +
·
·
= ·
9 10 2 11 12 2
46
=
≈ 0.279.
165

4.25. An urn contains 20 silver coins and 10 gold coins. You are the sixth
person in line to randomly draw and keep a coin from the urn.
(a) What is the probability that you draw a gold coin?
(b) If you draw a gold coin, what is the probability that the five people ahead
of you all drew silver coins?
Solution to Exercise 4.25.
(a) It doesn’t matter if you are the sixth to draw a coin, or the first, or
the last, your chance of getting a gold coin is 10/30, since there are 10 gold
coins and 30 coins altogether. (If you had some information about the color
of the coins drawn by the people ahead of you, that would change the answer,
but the problem does not give you any such information.)
(b) This part is more difficult. We define events:
E = {You draw a gold coin},
F = {Previous 5 people drew silver coins}.
We want to compute Pr(F | E) and we will use Baye’s law in the form
Pr(F | E) =

Pr(E | F ) Pr(F )
.
Pr(E)

As already explained, Pr(E) = 1/3. Similarly, it is easy to compute Pr(E | F ).
The assumption that F is true means that when you draw your coin, the urn
now contains 15 silver coins and 10 gold coins, so your probability of drawing
a gold coin is Pr(E | F ) = 10/25 = 2/5.
Finally, to compute Pr(F ), define events F1 , . . . , F5 by
Fi = {Person #i draws a silver coin}.
Then

Exercises for Chapter 4

121

Pr(F ) = Pr(F1 &F2 &F3 &F4 &F5 )
= Pr(F1 ) · Pr(F2 | F1 ) · Pr(F3 | F1 &F2 ) · Pr(F4 | F1 &F2 &F3 )
· Pr(F5 | F1 &F2 &F3 &F4 )
20 19 18 17 16
=
·
·
·
·
30 29 28 27 26
2584
=
≈ 0.109.
23751
We now have the values needed to solve the problem:
Pr(F | E) =

Pr(E | F ) Pr(F )
(2/5) · (2584/23751)
5168
=
=
≈ 0.131.
Pr(E)
1/3
39585

Thus with no other knowledge, there is approximately an 11% chance
that the first five coins chosen are silver, but if we know that the sixth coin
chosen is gold, then the probability that the first five were silver increases to
approximately 13%.
4.26. (The Monty Hall Problem) Monty Hall gives Dan, a contestant, the
choice of three boxes. One box contains a valuable prize and the other two
contain nothing. Dan chooses a box, but does not yet open it. Monty Hall
then opens one of the other boxes, shows that it is empty, and offers Dan the
option of keeping his original box or of switching it for the remaining box.
The Monty Hall problem is to figure out Dan’s best strategy: “To hold or to
switch?”
The answer may depend on the strategy that Monty Hall employs in deciding which box to open when he has a choice, i.e., when Dan initially chooses
the prize box and the other two boxes are empty. This problem considers
various strategies. (We assume in all cases that Dan is aware of Monty Hall’s
chosen strategy.)
(a) Suppose that when Monty Hall has a choice, he randomly opens one of the
two empty boxes. Should Dan hold or switch, and what is his probability
of winning?
(b) Suppose that Monty Hall has mentally labeled the boxes 0, 1, and 2, and
that if Dan chooses Box n and if the other two boxes are empty, then
Monty Hall opens Box n + 1. (If n = 2, then he opens Box 0.) Should
Dan hold or switch, and what is his probability of winning?
(c) Again assume that Monty Hall has mentally labeled the boxes 0, 1, and 2,
but now suppose that Monty Hall always opens the lowest-numbered
empty box. What is Dan’s best strategy and what is his probability of
winning? (You may assume that the prize is placed in each box with equal
probability.)
(d) Same questions as in (b) and (c), except that Dan also knows how the
boxes are labeled.
(e) With the same assumptions as in (c), suppose that Dan employs his best
strategy and that Monty Hall knows that Dan is employing this strategy.

122

Exercises for Chapter 4

Can Monty Hall hurt Dan’s chances of winning by placing the prize in
one box more often than the others? But if he does so and if Dan knows,
can Dan do better by changing his strategy?
(f) Suppose that we return to the scenario in (a), but we give Monty Hall
another option, namely he can force Dan to keep the box that Dan initially
chose. Now what is Dan’s best strategy to win the prize and what is Monty
Hall’s best strategy to stop Dan?
Solution to Exercise 4.26.
A solution for this exercise is not currently available.
4.27. Let S be a set, let A be a property of interest, and suppose that for m ∈
S, we have Pr(m has property A) = δ. Suppose further that a Monte Carlo
algorithm applied to m and a random number r satisfy:
(1) If the algorithm returns Yes, then m definitely has property A.
(2) If m has property A, then the probability that the algorithm returns
Yes is at least p.
Notice that we can restate (1) and (2) as conditional probabilities:
(1) Pr(m has property A | algorithm returns Yes) = 1,
(2) Pr(algorithm returns Yes | m has property A) ≥ p.
Suppose that we run the algorithm N times on the number m, and suppose
that the algorithm returns No every single time. Derive a lower bound, in
terms of δ, p, and N , for the probability that m does not have property A.
(This generalizes the version of the Monte Carlo method that we studied in
Section 4.3.3 with δ = 0.01 and p = 21 . Be careful to distinguish p from 1 − p
in your calculations.)
Solution to Exercise 4.27.
Let
E = {an element m ∈ S does not have property A}.
F = {the algorithm returns No N times in a row}.
We want a lower bound for the conditional probability Pr(E | F ), that is,
the probability that m does not have property A despite the fact that the
algorithm returned No N times. We compute this probability using Bayes’s
formula
Pr(F | E) Pr(E)
Pr(E | F ) =
.
Pr(F | E) Pr(E) + Pr(F | E c ) Pr(E c )
We are given that the probability of have property A is δ, so
Pr(E) = Pr(not A) = 1 − δ

and

Pr(E c ) = Pr(A) = δ.

Exercises for Chapter 4

123

Next consider Pr(F | E). If m does not have property A, which is our assumption on this conditional probability, then the algorithm always returns No,
since Property (1) tells us that a Yes output forces m to have property A.
Thus
Pr(No | not A) = Pr(A | Yes) = 1,
from which it follows that Pr(F | E) = Pr(No | not A)N = 1.
Finally, we must compute the value of Pr(F | E c ). Since the algorithm is
run N independent times, we have
Pr(F | E c ) = Pr(Output is No | m has property A)N
¡
¢N
= 1 − Pr(Output is Yes | m has property A)
N

≤ (1 − p)

from Property (2) of the Monte Carlo method.

Substituting these values into Bayes’s formula, we find that if the algorithm
returns No N times in a row, then the probability that the integer m does not
have property A is
Pr(E | F ) ≥

1 · (1 − δ)
1−δ
=
.
N
1 · (1 − δ) + (1 − p) · δ
1 − δ + (1 − p)N · δ

If δ and p are not too small and N is large, this can be approximated by
Pr(E | F ) ≥ 1 −

(1 − p)N · δ
(1 − p)N
(1 − p)N · δ
≈
1
−
=
1
−
.
1 − δ + (1 − p)N · δ
1−δ
δ −1 − 1

4.28. We continue with the setup described in Exercise 4.27.
9
and p = 34 . If we run the algorithm 25 times on the
(a) Suppose that δ = 10
input m and always get back No, what is the probability that m does not
have property A?
(b) Same question as (a), but this time we run the algorithm 100 times.
99
(c) Suppose that δ = 100
and p = 12 . How many times should we run the
algorithm on m to be 99% confident that m does not have property A,
assuming that every output is No?
(d) Same question as (c), except now we want to be 99.9999% confident.
Solution to Exercise 4.28.
A solution for this exercise is not currently available.
4.29. If an integer n is composite, then the Miller–Rabin test has at least
a 75% chance of succeeding in proving that n is composite, while it never
misidentifies a prime as being composite. (See Table 3.2 in Section 3.4 for a
description of the Miller–Rabin test.) Suppose that we run the Miller–Rabin
test N times on the integer n and that it fails to prove that n is composite.
Show that the probability that n is prime satisfies (approximately)

124

Exercises for Chapter 4
Pr(n is prime | the Miller–Rabin test fails N times) ≥ 1 −

ln(n)
.
4N

(Hint. Use Exercise 4.27 with appropriate choices of A, S, δ, and p. You may
also use the estimate from Section 3.4.1 that the probability that n is prime
is approximately 1/ ln(n).)
Solution to Exercise 4.29.
In Exercise 4.27 we let A be the property of being composite and we
let p = 34 , since we know that if n is composite, then the Miller–Rabin test
returns Yes at least 75% of the time. Further, we have δ ≈ 1 − 1/ ln(n), since δ
is the probability that n is composite, which is 1 minus the probability that
it is prime. The solution to that exercise says that (approximately)
Pr(n is prime | the Miller–Rabin test fails N times)
(1 − p)N
δ −1 − 1
ln(n) − 1
≈1−
4N
ln(n)
≈1− N .
4
≥1−

4.30. Let fX (k) be the binomial
Pn density function (4.23). Prove directly, using
the binomial theorem, that k=0 fX (k) = 1.
Solution to Exercise 4.30.
Let q = 1 − p, so p + q = 1. Then we use the binomial theorem to compute
n
n µ ¶
n µ ¶
X
X
X
n k
n k n−k
n−k
fX (k) =
p (1 − p)
=
p q
= (p + q)n = 1n = 1.
k
k
k=0

k=0

k=0

4.31. In Example 4.37
P∞we used a differentiation trick to compute the value
of the infinite series n=1 np(1 − p)n−1 . This exercise further develops this
useful technique. The starting point is the formula for the geometric series
∞
X
n=0

(a) Prove that

xn =

1
1−x

∞
X
n=1

nxn−1 =

for |x| < 1.

(4.1)

1
(1 − x)2

(4.2)

by differentiating both sides of (4.57) with respect to x. For which x does
the left-hand side of (4.58) converge? (Hint. Use the ratio test.)

Exercises for Chapter 4

125

(b) Differentiate again to prove that
∞
X

n(n − 1)xn−2 =

n=2

2
.
(1 − x)3

(4.3)

(c) More generally, prove that for every k ≥ 0,
¶
∞ µ
X
n+k n
1
.
x =
(1 − x)k+1
k
n=0
(Hint. Use induction on k.)
(d) Prove that
∞
X

n2 xn =

n=0

x + x2
.
(1 − x)3

(4.4)

(4.5)

(Hint. Multiply (4.58) by x and (4.59) by x2 and then add them together.)
(e) Find a formula for
∞
X
n3 xn .
(4.6)
n=0

(f) Prove that for every value of k there is a polynomial Fk (x) such that
∞
X
n=0

nk x n =

Fk (x)
.
(1 − x)k+1

(4.7)

(Hint. Use induction on k.) Compute the polynomials F0 (x), F1 (x),
and F2 (x).
(g) Prove that the polynomial Fk (x) in (f) has degree k.
Solution to Exercise 4.31.
(a) Term-by-term differentiation gives the formula. We compute
|(n + 1)xn+1 |
n+1
= lim
|x| = |x|.
n→∞
n→∞
|nxn |
n

ρ = lim

The ratio test tells us that the series converges for |x| < 1 and diverges
for |x| > 1. And the series clearly also diverges for |x| = 1.
(b) Term-by-term differentiation of (a) gives the formula.
(c) Differentiating k times gives
∞
X
n=0

n(n − 1)(n − 2) · · · (n − k + 1)xn−k =

1 · 2 · 3···k
.
(1 − x)k+1

(If one wants to be formal, one can prove this formula by induction on k.) Now
divide both sides by k! and use the definition of the combinatorial symbol to
get

126

Exercises for Chapter 4
∞ µ ¶
X
n n−k
1
.
x
=
k
(1 − x)k+1

n=k

(Notice that we only need to start the summation with n = k, since terms
with n = 0, 1, . . . , k−1 give 0.) Finally, in the summation on the left, replace n
by n + k, so now the sum starts at n = 0. This gives
¶
∞ µ
X
n+k n
1
x =
.
k
(1
−
x)k+1
n=0
(d) Following the hint, we first compute
x

∞
X

nxn−1 + x2

n=1

∞
X

n(n − 1)xn−2 =

n=2

∞
X
¡

∞
X
¢
n + n(n − 1) xn =
n2 xn .

n=0

n=0

Then, according to our results in (a) and (b), this is also equal to
2x2
x + x2
x
+
=
.
(1 − x)2
(1 − x)3
(1 − x)3
(e) One way to do this is to differentiate (b) and then combine various quantities to get the desired result. An easier method is to differentiate both sides
of (d). Thus
¶
µ∞
µ
¶
d X 2 n
d
x + x2
n x
=
dx n=0
dx (1 − x)3
∞
X

n3 xn−1 =

n=0

1 + 4x + x2
.
(1 − x)4

Multiplying both sides by x gives the desired result,
∞
X

n3 xn =

n=0

x + 4x2 + x3
.
(1 − x)4

(f, g)From the geometric series we have F0 (x) = 1, and (a) and (d) give F1 (x) =
x and F2 (x) = x + x2 . Now assume that
∞
X
n=0

nk xn =

Fk (x)
(1 − x)k+1

is true for k, where Fk (x) is a polynomial of degree k. Differentiating both
sides and using the differentiation rule for quotients yields
∞
X

nk+1 xn−1 =

n=0

=

(1 − x)k+1 Fk0 (x) − (k + 1)(1 − x)k (−1)Fk (x)
(1 − x)2k+2
(1 − x)Fk0 (x) + (k + 1)Fk (x)
(1 − x)k+2

Exercises for Chapter 4

127

Multiplying both sides by x gives
∞
X

nk+1 xn =

n=0

(x − x2 )Fk0 (x) + (k + 1)xFk (x)
,
(1 − x)k+2

so the desired formula is true with
Fk+1 (x) = (x − x2 )Fk0 (x) + (k + 1)xFk (x).
Since the degree of Fk0 (x) is one smaller than the degree of Fk (x), we also see
that
deg Fk+1 (x) = 1 + deg Fk (x).
Since deg F0 (x) = 0, we conclude that deg Fk (x) = k.
4.32. In each case, compute the expectation of the random variable X.
(a) The values of X are uniformly distributed on the set {0, 1, 2, . . . , N − 1}.
(See Example 4.28.)
(b) The values of X are uniformly distributed on the set {1, 2, . . . , N }.
(c) The values of X are uniformly distributed on the set {1, 3, 7, 11, 19, 23}.
(d) X is a random variable with a binomial density function (see (4.23) on
page 221).
Solution to Exercise 4.32.
(a)
1
1
1
1
+1·
+2·
+ · · · + (N − 1) ·
N
N
N
N
0 + 1 + 2 + · · · + (N − 1)
=
N
1
(N
−
1)N
= 2
N
N −1
=
.
2

E(X) = 0 ·

(b)
1
1
1
1
+2·
+3·
+ ··· + N ·
N
N
N
N
1 + 2 + 3 + ··· + N
=
N
1
N (N + 1)
= 2
N
N +1
=
.
2

E(X) = 1 ·

(c)

128

Exercises for Chapter 4
E(X) =

1 + 3 + 7 + 11 + 19 + 23
64
32
=
=
.
6
6
3

(d)
E(X) =

n
X

k · fX (k)

k=0
n
X

µ ¶
n k
p (1 − p)n−k
k
=
k
k=0
µ ¶µ
¶k
n
X
n
p
n
= (1 − p)
k
.
k
1−p
k=0

If we let x = p/(1 − p), then we need to compute the value of the sum
µ ¶
n
X
n k
k
x .
k
k=0

To do this, we start with the binomial theorem
n µ ¶
X
n k
x = (x + 1)n
k
k=0

and differentiate both sides with respect to x to get
µ ¶
n
X
n k−1
k
x
= n(x + 1)n−1 .
k
k=0

Now multiply both sides by x to get
µ ¶
n
X
n k
k
x = nx(x + 1)n−1 .
k
k=0

This gives the value
E(X) = (1−p)n ·n·

µ
¶n−1
µ
¶n−1
p
p
p
1
·
+1
= (1−p)n ·n·
·
= np.
1−p 1−p
1−p 1−p

This makes sense, since if we perform the experiment n times and have a
probability p of succeeding each time, we would expect to succeed, on average,
a total of np times.
4.33. Let X be a random variable on the probability space Ω. It might seem
more natural to define the expected value of X by the formula
X
X(ω) · Pr(ω).
(4.8)
ω∈Ω

Prove that the formula (4.64) gives the same value as equation (4.27) on
page 225, which we used in the text to define E(X).

Exercises for Chapter 4

129

Solution to Exercise 4.33.
We compute (the key step comes in the middle where we reverse the order
of summation):
E(X) =
=
=

n
X
i=1
n
X
i=1
n
X

xi · fX (xi )
xi · Pr(X = xi )
©
ª
xi Pr ω ∈ Ω : X(ω) = xi

i=1

=

n
X

xi

X

ω∈Ω
X(ω)=xi

i=1

=

X

ω∈Ω

=

X

Pr(ω)

Pr(ω)

X

xi

1≤i≤n
xi =X(ω)

Pr(ω) · X(ω),

ω∈Ω

where for the final equality we use that fact that x1 , . . . , xn are distinct, so
each X(ω) is equal to exactly one of the xi values.
Section. Collision algorithms and the birthday paradox
4.34. (a) In a group of 23 strangers, what is the probability that at least two
of them have the same birthday? How about if there are 40 strangers?
In a group of 200 strangers, what is the probability that one of them
has the same birthday as your birthday? (Hint. See the discussion in
Section 4.4.1.)
(b) Suppose that there are N days in a year (where N could be any number) and that there are n people. Develop a general formula, analogous
to (4.28), for the probability that at least two of them have the same
birthday. (Hint. Do a calculation similar to the proof of (4.28) in the collision theorem (Theorem 4.38), but note that the formula is a bit different
because the birthdays are being selected from a single list of N days.)
(c) Find a lower bound of the form
Pr(at least one match) ≥ 1 − e−(some function of n and N )
for the probability in (b), analogous to the estimate (4.29).
Solution to Exercise 4.34.
We start by doing (b).

130

Exercises for Chapter 4
µ
¶
µ
¶
at least one match
all n birthdays
Pr
= 1 − Pr
in n attempts
are different
 th

n
i birthday is different
Y

Pr from all of the
=1−
i=1
previous i − 1 birthdays
n
Y
N − (i − 1)
=1−
N
i=1
µ
¶
n−1
Y
i
=1−
1−
.
N
i=1

Then the answer to the first part of (a) is obtained by setting N = 365
and n = 23, which gives
(a)

Pr(match) = 1 −

¶
22 µ
Y
i
1−
≈ 50.73%.
365
i=1

Similarly, N = 365 and n = 40 gives the answer to the second part of (a),
(b)

¶
39 µ
Y
i
Pr(match) = 1 −
1−
≈ 89.12%.
365
i=1

The final part of (a) is
Pr(someone has your birthday) = 1 − Pr(no one has your birthday)
= 1 − Pr(one person does not have your birthday)200
µ
¶200
364
=1−
365
≈ 42.23%.
For (c) we use the lower bound e−x ≥ 1 − x with x = i/N to compute
µ
Pr

¶
¶
n−1
Yµ
i
at least one match
=1−
1−
in n attempts
N
i=1
≥1−

n−1
Y

e−i/N

i=1
−(1+2+···+(n−1))/N

=1−e

= 1 − e−(n−1)n/2N
2

≈ 1 − e−n

/2N

Notice that we have used the well known formula

.

Exercises for Chapter 4

131

1 + 2 + · + (n − 1) =

n(n − 1)
.
2

4.35. A deck of cards is shuffled and the top eight cards are turned over.
(a) What is the probability that the king of hearts is visible?
(b) A second deck is shuffled and its top eight cards are turned over. What
is the probability that a visible card from the first deck matches a visible
card from the second deck? (Note that this is slightly different from Example 4.39 because the cards in the second deck are not being replaced.)
Solution to Exercise 4.35.
A solution for this exercise is not currently available.
4.36. (a) Prove that
e−x ≥ 1 − x

for all values of x.

(Hint. Look at the graphs of e−x and 1 − x, or use calculus to compute
the minimum of the function f (x) = e−x − (1 − x).)
(b) Prove that for all a > 1, the inequality
1
e−ax ≤ (1 − x)a + ax2
2

is valid for all 0 ≤ x ≤ 1.

(This is a challenging problem.)
(c) We used the inequality in (a) during the proof of the lower bound (4.29)
in the collision theorem (Theorem 4.38). Use (b) to prove that
Pr(at least one red) ≤ 1 − e−mn/N +

mn2
.
2N 2

Thus if N is large and m and n are not much larger than
estimate
Pr(at least one red) ≈ 1 − e−mn/N

√

N , then the

is quite accurate. (Hint. Use (b) with a = m and x = n/N .)
Solution to Exercise 4.36.
(a) Let f (x) = e−x − 1 + x. Then f (0) = f 0 (0) = 0. Then generalized mean
value theorem says that
1
f (x) = f (0) + f 0 (0)x + f 00 (z)x2
2

for some 0 ≤ z ≤ x,

so we find that f (x) = 12 e−z x2 ≥ 0. This is the desired inequality.
(b) Let
1
f (x) = (1 − x)a + ax2 − e−ax .
2

132

Exercises for Chapter 4

Since f (0) = 0, it suffices to prove that
f 0 (x) = −a(1 − x)a−1 + ax + ae−ax
is positive for 0 < x < 1. We can divide by a, and for notational convenience,
we let a = b + 1. So we need to prove that
g(x) = −(1 − x)b + x + e−(b+1)x
is positive for 0 < x < 1 and b > 0.
From (a) we know that e−x > 1 − x, so raising both sides to the bth power
and multiplying by −1 gives
−(1 − x)b > −e−bx .
Substituting this into g(x), we find that
g(x) > −e−bx + x + e−(b+1)x = x − e−bx (1 − e−x ).
It is clear that this last expression is increasing as b increases. (To be more
formal, its derivative with respect to b is be−bx (1 − e−x ), which is strictly
positive for 0 < x < 1.) Hence the expression is minimized when b = 0, so we
get
g(x) > x − (1 − e−x ) = e−x − (1 − x).
Using (a) again gives g(x) > 0.
Remark: It appears to be true numerically that f (x) ≥ 0 provided that a >
0.8526055 . . . , where c = 0.8526055 . . . is the unique real solution to cec = 2.
(c) We have
µ
¶
³
n ´m
at
least
Pr
=
1
−
1
−
from the Collision Theorem
one red
N
¶
µ
n
mn2
use (b) with a = m and x = .
≤ 1 − e−mn/N −
2
2N
N
4.37. Solve the discrete logarithm problem 10x = 106 in the finite field F811
by finding a collision among the random powers 10i and 106 · 10i that are
listed in Table 4.17.
Solution to Exercise 4.37.
From Table 4.17 we see that
10234 = 106 · 10399 = 304
Hence

in F811 .

10234 · 10−399 = 10−165 = 10645 = 106

in F811 .

Exercises for Chapter 4
i
116
497
225
233
677
622

hi a · hi
96 444
326 494
757 764
517 465
787 700
523 290

133

hi a · hi
291 28
239 193
358 642
789 101
24 111
748 621

i
519
286
298
500
272
307

i
791
385
178
471
42
258

hi a · hi
496 672
437 95
527 714
117 237
448 450
413 795

i
406
745
234
556
326
399

hi a · hi
801 562
194 289
304 595
252 760
649 670
263 304

Table 4.12: Data for Exercise 4.37, h = 10, a = 106, p = 811

Section. Pollard’s ρ method
4.38. Table 4.18 gives some of the computations for the solution of the discrete
logarithm problem
11t = 41387 in F81799
(4.9)
using Pollard’s ρ method. (It is similar to Table 4.11 in Example 4.51.) Use
the data in Table 4.18 to solve (4.65).
Solution to Exercise 4.38.

α154 = 81756,
81756

11

x308 = x154 = 15386 in F81799 .
β154 = 9527,
γ154 = 67782,
· 41387

9527

= 11

67782

13974

11

28637

· 41387

δ154 = 28637.
in F81799 .

19110

= 41387
in F81799 .
gcd(19110, 81798) = 6.
81340 · 19110 ≡ 6 (mod 81798).
1113974·81340 = 111136645160 = 1161950 = 413876 in F81799 .
81798
61950
= 10325,
= 13633.
6
6
log11 (41387) ∈ {10325 + 13633 · k : 0 ≤ k < 6}
= {10325, 23958, 37591, 51224, 64857, 78490}.
1110325 = 73192,

1123958 = 40412,

1137591 = 49019,

1164857 = 41387 ,

1178490 = 32780.

1151224 = 8607,

4.39. Table 4.19 gives some of the computations for the solution of the discrete
logarithm problem
7t = 3018 in F7963
(4.10)

134

Exercises for Chapter 4
i
0
1
2
3
4
151
152
153
154

xi

yi

1
11
121
1331
14641

1
121
14641
42876
7150

0
1
2
3
4

0
0
0
0
0

0
2
4
12
25

0
0
0
2
4

33573
53431
23112
15386

..
.
40876
81754
81755
81756

45662
9527
9527
9527

29798
37394
67780
67782

73363
48058
28637
28637

4862
23112
8835
15386

αi

βi

γi

δi

Table 4.13: Computations to solve 11t = 41387 in F81799 for Exercise 4.38

using Pollard’s ρ method. (It is similar to Table 4.11 in Example 4.51.) Extend
Table 4.19 until you find a collision (we promise that it won’t take too long)
and then solve (4.66).

Solution to Exercise 4.39.
Extending the table:

i
87
88
89
90
91
92
93

xi
1329
1340
1417
1956
5729
2449
1217

yi
1494
1539
4767
1329
1417
5729
1217

αi
6736
6737
6738
6739
6740
6740
6741

βi
7647
7647
7647
7647
7647
7648
7648

γi
3148
3150
6302
4642
4644
4646
4647

δi
3904
3904
7808
7655
7655
7655
7656

Exercises for Chapter 4

135

x186 = x93 = 1217 in F7963 .
β93 = 7648,
γ93 = 4647,

α93 = 6741,

76741 · 30187648 = 74647 · 30187656
2094

7

= 3018

2094

7

8

δ93 = 7656.

in F7963 .

in F7963 .

8

= 3018 in F7963 .
gcd(8, 7962) = 2.

6967 · 8 ≡ 2 (mod 7962).
2094·6967

= 714588898 = 72514 = 30182 in F7963 .
7962
2514
= 1257,
= 3981.
2
2
log7 (3018) ∈ {1257 + 3981 · k : 0 ≤ k < 2} = {1257, 5238}.
7

71257 = 4945,

i
0
1
2
3
4

xi

yi

αi

1
7
49
343
2401

1
49
2401
6167
1399

87
88
89
90

1329
1340
1417
1956

1494
1539
4767
1329

0
1
2
3
4
..
.
6736
6737
6738
6739

75238 = 3018.

βi

γi

δi

0
0
0
0
0

0
2
4
6
7

0
0
0
0
1

7647
7647
7647
7647

3148
3150
6302
4642

3904
3904
7808
7655

Table 4.14: Computations to solve 7t = 3018 in F7963 for Exercise 4.39

4.40. Write a computer program implementing Pollard’s ρ method for solving
the discrete logarithm problem and use it to solve each of the following:
(a) 2t = 2495 in F5011 .
(b) 17t = 14226 in F17959 .
(c) 29t = 5953042 in F15239131 .
Solution to Exercise 4.40.
(a) 23351 = 2495 .
(b) 1714557 = 14226 .
(c) 292528453 = 5953042 .

136

Exercises for Chapter 4

R∞
2
4.41. Evaluate the integral I = 0 t2 e−t /2 dt appearing in the proof of
Theorem 4.47. (Hint. Write I 2 as an iterated integral,
Z ∞Z ∞
2
2
2
I =
x2 e−x /2 · y 2 e−y /2 dx dy,
0

0

and switch to polar coordinates.)
Solution to Exercise 4.41.
Following the hint, we have
Z ∞Z ∞
2
2
I2 =
x2 e−x /2 y 2 e−y /2 dx dy
Z0 ∞ Z0 ∞
2
2
=
x2 y 2 e−(x +y )/2 dx dy
0

Z

0

∞

Z

π/2

=
0

0
π/2

ÃZ
=

(r sin θ)2 (r cos θ)2 e−r
! µZ
2

∞

2

sin θ cos θ dθ

2

/2

¶
5 −r 2 /2

r e

0

r dr dθ
dr .

0

Each of these integrals is now a moderately hard freshman calculus exercise.
For the first one we can use
¶2
µ
1 1 − cos(4θ)
1
sin(2θ) = ·
.
sin2 θ cos2 θ = (sin θ cos θ)2 =
2
4
2
Then
Z

π/2

Z
2

π/2

2

sin θ cos θ dθ =
0

0

¯π/2
1 − cos(4θ)
θ sin(4θ) ¯¯
π
dθ = −
.
=
8
8
32 ¯0
16

For the second integral we substitute r2 = z and then integrate by parts twice.
Thus
Z ∞
Z ∞
2
1
r5 e−r /2 dr =
z 2 e−z/2 dz
2
0
0
Z ∞
¯∞
¯
ze−z/2 dz
= −z 2 e−z/2 ¯ + 2
0
0
Z ∞
=2
ze−z/2 dz
0
Z ∞
¯∞
−z/2 ¯
e−z/2 dz
= −4ze
¯ +4
0
0
Z ∞
e−z/2 dz
=4
0
¯∞
¯
= −8e−z/2 ¯ = 8.
2

Hence I = π/16 · 8 = π/2, so I =

p

0

π/2.

Exercises for Chapter 4

137

Section. Information theory
4.42. Consider the cipher that has three keys, three plaintexts, and four
ciphertexts that are combined using the following encryption table (which is
similar to Table 4.12 used in Example 4.53 on page 246).
k1
k2
k3

m1
c2
c1
c3

m2
c4
c3
c1

m3
c1
c2
c2

Suppose further that the plaintexts and keys are used with the following
probabilities:
f (m1 ) = f (m2 ) =

2
,
5

f (m3 ) =

1
,
5

f (k1 ) = f (k2 ) = f (k3 ) =

1
.
3

(a) Compute f (c1 ), f (c2 ), f (c3 ), and f (c4 ).
(b) Compute f (c1 | m1 ), f (c1 | m2 ), and f (c1 | m3 ). Does this cryptosystem
have perfect secrecy?
(c) Compute f (c2 | m1 ) and f (c3 | m1 ).
(d) Compute f (k1 | c3 ) and f (k2 | c3 ).
Solution to Exercise 4.42.
A solution for this exercise is not currently available.
4.43. Suppose that a shift cipher is employed such that each key, i.e., each
shift amount from 0 to 25, is used with equal probability and such that a new
key is chosen to encrypt each successive letter. Show that this cryptosystem
has perfect secrecy
P by filling in the details of the following steps.
(a) Show that k∈K fM (dk (c)) = 1 for every ciphertext c ∈ C.
(b) Compute the ciphertext density function fC using the formula
X
fC (c) =
fK (k)fM (dk (c)).
k∈K

(c) Compare fC (c) to fC|M (c | m).
Solution to Exercise 4.43.
A solution for this exercise is not currently available.
4.44. Suppose that a cryptosystem has the same number of plaintexts as it
does ciphertexts (#M = #C). Prove that for any given key k ∈ K and any
given ciphertext c ∈ C, there is a unique plaintext m ∈ M that encrypts to c
using the key k. (We used this fact during the proof of Theorem 4.55. Notice
that the proof does not require the cryptosystem to have perfect secrecy; all
that is needed is that #M = #C.)

138

Exercises for Chapter 4

Solution to Exercise 4.44.
Fix k ∈ K. The encryption map ek : M → C is injective by definition
of a cryptosystem, so our assumption that #M = #C implies that ek is also
surjective, and hence is a bijective map from M to C. This is equivalent to the
assertion that for every c ∈ C, there is a unique m ∈ M satisfying ek (m) = c,
which is the desired result.
©
ª
4.45. Let Sm,c = k ∈ K : ek (m) = c be the set used during the proof of
Theorem 4.55. Prove that if c 6= c0 , then Sm,c ∩ Sm,c0 = ∅. (Prove this for any
cryptosystem; it is not necessary to assume perfect secrecy.)
Solution to Exercise 4.45.
Suppose that k ∈ Sm,c ∩Sm,c0 . Then c = ek (m) = c0 . Hence Sm,c ∩Sm,c0 6= ∅
implies that c = c0 .
4.46. Suppose that a cryptosystem satisfies #K = #M = #C and that it
has perfect secrecy. Prove that every ciphertext is used with equal probability
and that every plaintext is used with equal probability. (Hint. We proved one
of these during the course of proving Theorem 4.55. The proof of the other is
similar.)
Solution to Exercise 4.46.
A solution for this exercise is not currently available.
4.47. Prove the “only if” part of Theorem 4.55, i.e., prove that if a cryptosystem with an equal number of keys, plaintexts, and ciphertexts satisfies
conditions (a) and (b) of Theorem 4.55, then it has perfect secrecy.
Solution to Exercise 4.47.
A solution for this exercise is not currently available.
4.48. Let X be an experiment (random variable) with outcomes x1 , . . . , xn
occurring with probabilities p1 , . . . , pn , and similarly let Y be an experiment
with outcomes y1 , . . . , ym occurring with probabilities q1 , . . . , qm . Consider the
experiment Z consisting of first performing X and then performing Y . Thus
the outcomes of Z are the mn pairs (xi , yj ) occurring with probabilities pi qj .
Use the formula for entropy (4.51) to prove that
H(Z) = H(X) + H(Y ).
Thus entropy is additive on independent compound events, which is a special
case of Property H3 on page 250.
Solution to Exercise 4.48.
Using the formula for entropy, we compute

Exercises for Chapter 4
H(Z) = −
=−
=−

139

n X
m
X
i=1 j=1
n X
m
X

pi qj log(pi qj )
pi qj (log pi + log qj )

i=1 j=1
m
X

n
X

i=1

j=1

pi log pi

qj −

n
X

pi

i=1

m
X

qj log qj

j=1

= H(X) · 1 + 1 · H(Y ).

4.49. Let F (t) be a twice differentiable function with the property that F 00 (t) < 0
for all x in its domain. Prove that F is concave in the sense of (4.52). Conclude
in particular that the function F (t) = log t is concave for all t > 0.
Solution to Exercise 4.49.
A solution for this exercise is not currently available.
4.50. Use induction to prove Jensen’s inequality (Theorem 4.59).
Solution to Exercise 4.50.
The case n = 2 is true by definition of concavity. Assume now that it is
true for n. The idea is to combine two of the terms in the sum α1 t1 + · · · +
αn tn + αn+1 tn+1 into one term, say the last two. In other words, we want to
write
αn tn + αn+1 tn+1 as βn yn ,
but we need to make sure that α1 + α2 + · · · + αn−1 + βn = 1. So we need to
take βn = αn + αn+1 , which means that we need to take
yn =

αn tn + αn+1 tn+1
.
αn + αn+1

With this choice of βn and yn , we have α1 + · · · + αn−1 + βn = 1, so we can
apply the induction hypothesis to conclude that
f (α1 t1 + · · · + αn−1 tn−1 + βn yn ) ≤ α1 f (t1 ) + · · · + αn−1 f (tn−1 ) + βn f (yn ).
We are also going to apply the induction hypothesis to f (yn ). We can write yn
as
αn+1
αn
tn +
tn+1 = γtn + δtn+1 ,
yn =
αn + αn+1
αn + αn+1
where notice that γ and δ satisfy γ + δ = 1. Hence the induction hypothesis
tells us that
f (yn ) = f (γtn + δtn+1 ) ≤ γf (tn ) + δf (tn+1 ).

140

Exercises for Chapter 4

Now multiplying both sides by βn and substituting in the values γ, δ, and βn
yields
βn f (yn ) ≤ βn γf (tn ) + βn δf (tn+1 ) = αn f (tn ) + αn+1 f (tn+1 ).
Finally, substituting this in above gives the desired inequality
f (α1 t1 + · · · + αn tn + αn+1 tn+1 ) ≤ α1 f (t1 ) + · · · + αn f (tn ) + αn+1 f (tn+1 ).
The induction proof that there is equality if and only if all of the ti ’s are equal
is similar.
4.51. Let X and Y be independent random variables.
(a) Prove that the equivocation H(X | Y ) is equal to the entropy H(X).
(b) If H(X | Y ) = H(X), is it necessarily true that X and Y are independent?
Solution to Exercise 4.51.
Independence means that f (x | y) = f (x), so
X
H(X | Y ) = −
f (y)f (x | y) log f (x | y)
x,y

=−

X

f (y)f (x) log f (x)

x,y

=−

X

f (y)

y

X

f (x) log f (x)

x

= 1 · H(X).
For the converse, notice that
H(X) = −

X

f (x) log f (x)

x

=−

X ³X
x

=−

X

´
f (x, y) log f (x)

y

f (y)f (x | y) log f (x),

x,y

so
H(X) − H(X | Y ) = −

X

f (y)f (x | y) log

x,y

=−

X
x,y

f (x, y) log

f (x)
f (x | y)

f (x)f (y)
.
f (x, y)

It is likely that one could come up with dependent random variables X and Y
making this quantity vanish.

Exercises for Chapter 4

141

4.52. Suppose a cryptosystem has two keys, K = {k1 , k2 }, each of which
is equally likely to be used, and suppose that it has three plaintexts M =
{m1 , m2 , m3 } that occur with probabilities f (m1 ) = 21 , f (m2 ) = 41 , and
f (m3 ) = 14 .
(a) Create an encryption function for this cipher, similar to Example 4.53,
such that there are three ciphertexts C = {c1 , c2 , c3 } and such that the
ciphertext c1 occurs with probability 21 . (There is more than one correct
answer to this problem.)
(b) Compute the entropies H(K), H(M ), and H(C) of your encryption
scheme in (a).
(c) Compute the key equivocation H(K | C).
(d) Use your answer in (c) to explain why each ciphertext leaks information.
Solution to Exercise 4.52.
A solution for this exercise is not currently available.
4.53. Suppose that the key equivocation of a certain cryptosystem vanishes,
i.e., suppose that H(K | C) = 0. Prove that even a single observed ciphertext
uniquely determines which key was used.
Solution to Exercise 4.53.
A solution for this exercise is not currently available.
4.54. Write a computer program that reads a text file and performs the
following tasks:
[1] Convert all alphabetic characters to lowercase and convert all strings of
consecutive nonalphabetic characters to a single space. (The reason for
leaving in a space is that when you count bigrams and trigrams, you will
want to know where words begin and end.)
[2] Count the frequency of each letter a-to-z, print a frequency table, and use
your frequency table to estimate the entropy of a single letter in English,
as we did in Section 4.6.3 using Table 1.3.
[3] Count the frequency of each bigram aa, ab,. . . ,zz, being careful to include only bigrams that appear within words. (As an alternative, also
allow bigrams that either start or end with a space, in which case
there are 272 − 1 = 728 possible bigrams.) Print a frequency table of
the 25 most common bigrams and their probabilities, and use your full
frequency table to estimate the entropy of bigrams in English. In the
notation of Section 4.6.3, this is the quantity H(L2 ). Compare 12 H(L2 )
with the value of H(L) from step [1].
[4] Repeat [3], but this time with trigrams. Compare 31 H(L3 ) with the values
of H(L) and 21 H(L2 ) from [2] and [3]. (Note that for this part, you will
need a large quantity of text in order to get some reasonable frequencies.)
Try running your program on some long blocks of text. For example, the
following noncopyrighted material is available in the form of ordinary text files
from Project Gutenberg at http://www.gutenberg.net/. To what extent

142

Exercises for Chapter 4

are the letter frequencies similar and to what extent do they differ in these
different texts?
(a) Alice’s Adventures in Wonderland by Lewis Carroll,
http://www.gutenberg.net/etext/11
(b) Relativity: the Special and General Theory by Albert Einstein,
http://www.gutenberg.net/etext/5001
(c) The Old Testament (translated from the original Hebrew, of course!),
http://www.gutenberg.net/etext/1609
(d) 20000 Lieues Sous Les Mers (20000 Leagues Under the Sea) by Jules
Verne, http://www.gutenberg.net/etext/5097. Note that this one is a
little trickier, since first you will need to convert all of the letters to their
unaccented forms.

Chapter 5

Elliptic Curves and
Cryptography
Exercises for Chapter 5
Section. Elliptic curves
5.1. Let E be the elliptic curve E : Y 2 = X 3 − 2X + 4 and let P = (0, 2) and
Q = (3, −5). (You should check that P and Q are on the curve E.)
(a) Compute P ⊕ Q.
(b) Compute P ⊕ P and Q ⊕ Q.
(c) Compute P ⊕ P ⊕ P and Q ⊕ Q ⊕ Q.
Solution to Exercise 5.1.
(a) P ⊕ Q = (22/9, 100/27).
(b) P ⊕ P = Q ⊕ Q = (1/4, −15/8).
(c) P ⊕ P ⊕ P = (240, 3718) and Q ⊕ Q ⊕ Q = (−237/121, −845/1331).
5.2. Check that the points P = (−1, 4) and Q = (2, 5) are points on the
elliptic curve E : Y 2 = X 3 + 17.
(a) Compute the points P ⊕ Q and P ª Q.
(b) Compute the points 2P and 2Q.
(Bonus. How many points with integer coordinates can you find on E?)
Solution to Exercise
¡ 8 5.2.109 ¢
(a) P + Q = − 9 , − 27 and P − Q = (8, 23).
¡
¢
¡
¢
2651
59
(b) 2P = 137
and 2Q = − 64
64 , − 512
25 , 125
Bonus. This curve has 16 points with integer coordinates, including one
that is quite large. This is somewhat surpising number. The points are
(−2, ±3), (−1, ±4), (2, ±5), (4, ±9), (8, ±23), (43, ±282), (52, ±375), (5234, ±378661).
There are no others, but that’s not so easy to prove.
143

144

Exercises for Chapter 5

5.3. Suppose that the cubic polynomial X 3 + AX + B factors as
X 3 + AX + B = (X − e1 )(X − e2 )(X − e2 ).
Prove that 4A3 + 27B 2 = 0 if and only if two (or more) of e1 , e2 , and e3 are
the same. (Hint. Multiply out the right-hand side and compare coefficients to
relate A and B to e1 , e2 , and e3 .)
Solution to Exercise 5.3.
We have
X 3 + AX + B = X 3 − (e1 + e2 + e3 )X 2 + (e1 e2 + e1 e3 + e2 e3 )X − e1 e2 e3 ,
and comparing the coefficients gives three relations
e1 + e2 + e3 = 0,
e1 e2 + e1 e3 + e2 e3 = A,
e1 e2 e3 = B.
Suppose first that two of the ei are the same, say e2 = e3 . Then we get
e1 + 2e2 = 0,

2e1 e2 + e22 = A,

e1 e22 = B.

So e1 = −2e2 , and substituting this into the second and third equations gives
−3e22 = A
Hence

and

− 2e32 = B.

4A3 + 27B 2 = 4(−3e22 )3 + 27(−2e32 )2 = 0.

Conversely, suppose that 4A3 + 27B 2 = 0. Substituting the expressions
for A and B from above and multiplying it out gives the rather complicated
expression
4A3 + 27B 2 = (4e32 + 12e3 e22 + 12e23 e2 + 4e33 )e31 + (12e3 e32 + 51e23 e22 + 12e33 e2 )e21
+ (12e23 e32 + 12e33 e22 )e1
+ 4e33 e32 .
Next we substitute e1 = −e2 − e3 to get
4A3 + 27B 2 = −4e62 − 12e3 e52 + 3e23 e42 + 26e33 e32 + 3e43 e22 − 12e53 e2 − 4e63 .
We’d like to know that this last expression vanishes if any two of the ei are
the same. It is not hard to check that it is a multiple of e2 − e3 , and indeed a
multiple of (e2 − e3 )2 . But we’d also like it to vanish when e3 = e1 , which is
the same as when e3 = −e2 − e3 . So we check and find that the expression is
divisible by e2 + 2e3 , and in fact it is divisible by (e2 + 2e3 )2 . Similarly, it is
divisible by (e3 + 2e2 )2 . So we find that

Exercises for Chapter 5

145

4A3 + 27B 2 = −(e2 − e3 )2 (e2 + 2e3 )2 (e3 + 2e2 )2 .
Hence using the fact that e1 + e2 + e3 = 0, we find that
4A3 + 27B 2

if and only if

(e2 − e3 )2 (e1 − e3 )2 (e1 − e2 )2 = 0,

5.4. Sketch each of the following curves, as was done in Figure 5.1 on
page 280.
(a) E : Y 2 = X 3 − 7X + 3.
(b) E : Y 2 = X 3 − 7X + 9.
(c) E : Y 2 = X 3 − 7X − 12.
(d) E : Y 2 = X 3 − 3X + 2.
(e) E : Y 2 = X 3 .
Notice that the curves in (d) and (e) have ∆E = 0, so they are not elliptic
curves. How do their pictures differ from the pictures in (a), (b), and (c)?
Each of the curves (d) and (e) has one point that is somewhat unusual. These
unusual points are called singular points.
Section. Elliptic curves over finite fields
5.5. For each of the following elliptic curves E and finite fields Fp , make a
list of the set of points E(Fp ).
(a) E : Y 2 = X 3 + 3X + 2 over F7 .
(b) E : Y 2 = X 3 + 2X + 7 over F11 .
(c) E : Y 2 = X 3 + 4X + 5 over F11 .
(d) E : Y 2 = X 3 + 9X + 5 over F11 .
(e) E : Y 2 = X 3 + 9X + 5 over F13 .
Solution to Exercise 5.5.
(a) #E(F7 ) = 9
E(F7 ) = {O, (0, 3), (0, 4), (2, 3), (2, 4), (4, 1), (4, 6), (5, 3), (5, 4)}
(b) #E(F11 ) = 7
E(F11 ) = {O, (6, 2), (6, 9), (7, 1), (7, 10), (10, 2), (10, 9)}
(c) #E(F11 ) = 8
E(F11 ) = {O, (0, 4), (0, 7), (3, 0), (6, 5), (6, 6), (9, 0), (10, 0)}
(d) #E(F11 ) = 14
E(F11 ) = {O, (0, 4), (0, 7), (1, 2), (1, 9), (2, 3), (2, 8), (3, 2), (3, 9), (6, 0),
(7, 2), (7, 9), (9, 1), (9, 10)}

146

Exercises for Chapter 5

(e) #E(F13 ) = 9
E(F13 ) = {O, (4, 1), (4, 12), (8, 2), (8, 11), (9, 3), (9, 10), (10, 4), (10, 9)}

5.6. Make an addition table for E over Fp , as we did in Table 5.1.
(a) E : Y 2 = X 3 + X + 2 over F5 .
(b) E : Y 2 = X 3 + 2X + 3 over F7 .
(c) E : Y 2 = X 3 + 2X + 5 over F11 .
You may want to write a computer program for (c), since E(F11 ) has a lot
of points!
Solution to Exercise 5.6.
(a) E(F5 ) = {O, (1, 2), (1, 3), (4, 0)}.
O
(1, 2)
(1, 3)
(4, 0)

O
O
(1, 2)
(1, 3)
(4, 0)

(1, 2)
(1, 2)
(4, 0)
O
(1, 3)

(1, 3)
(1, 3)
O
(4, 0)
(1, 2)

(4, 0)
(4, 0)
(1, 3)
(1, 2)
O

(b) E(F7 ) = {O, (2, 1), (2, 6), (3, 1), (3, 6), (6, 0)}.
O
(2, 1)
(2, 6)
(3, 1)
(3, 6)
(6, 0)

O
O
(2, 1)
(2, 6)
(3, 1)
(3, 6)
(6, 0)

(2, 1)
(2, 1)
(3, 6)
O
(2, 6)
(6, 0)
(3, 1)

(2, 6)
(2, 6)
O
(3, 1)
(6, 0)
(2, 1)
(3, 6)

(3, 1)
(3, 1)
(2, 6)
(6, 0)
(3, 6)
O
(2, 1)

(3, 6)
(3, 6)
(6, 0)
(2, 1)
O
(3, 1)
(2, 6)

(6, 0)
(6, 0)
(3, 1)
(3, 6)
(2, 1)
(2, 6)
O

(c) E(F11 ) = {O, (0, 4), (0, 7), (3, 4), (3, 7), (4, 0), (8, 4), (8, 7), (9, 2), (9, 9)}.
O
(0, 4)
(0, 7)
(3, 4)
(3, 7)
(4, 0)
(8, 4)
(8, 7)
(9, 2)
(9, 9)

O

(0, 4)

(0, 7)

(3, 4)

(3, 7)

(4, 0)

(8, 4)

(8, 7)

(9, 2)

(9, 9)

O
(0, 4)
(0, 7)
(3, 4)
(3, 7)
(4, 0)
(8, 4)
(8, 7)
(9, 2)
(9, 9)

(0, 4)
(9, 2)
O
(8, 7)
(9, 9)
(8, 4)
(3, 7)
(4, 0)
(3, 4)
(0, 7)

(0, 7)
O
(9, 9)
(9, 2)
(8, 4)
(8, 7)
(4, 0)
(3, 4)
(0, 4)
(3, 7)

(3, 4)
(8, 7)
(9, 2)
(8, 4)
O
(9, 9)
(0, 7)
(3, 7)
(4, 0)
(0, 4)

(3, 7)
(9, 9)
(8, 4)
O
(8, 7)
(9, 2)
(3, 4)
(0, 4)
(0, 7)
(4, 0)

(4, 0)
(8, 4)
(8, 7)
(9, 9)
(9, 2)
O
(0, 4)
(0, 7)
(3, 7)
(3, 4)

(8, 4)
(3, 7)
(4, 0)
(0, 7)
(3, 4)
(0, 4)
(9, 2)
O
(9, 9)
(8, 7)

(8, 7)
(4, 0)
(3, 4)
(3, 7)
(0, 4)
(0, 7)
O
(9, 9)
(8, 4)
(9, 2)

(9, 2)
(3, 4)
(0, 4)
(4, 0)
(0, 7)
(3, 7)
(9, 9)
(8, 4)
(8, 7)
O

(9, 9)
(0, 7)
(3, 7)
(0, 4)
(4, 0)
(3, 4)
(8, 7)
(9, 2)
O
(8, 4)

5.7. Let E be the elliptic curve
E : y 2 = x3 + x + 1.
Compute the number of points in the group E(Fp ) for each of the following
primes:
(a) p = 3.
(b) p = 5.
(c) p = 7.
(d) p = 11.

Exercises for Chapter 5

147

In each case, also compute the trace of Frobenius
tp = p + 1 − #E(Fp )
√
and verify that |tp | is smaller than 2 p.
Solution to Exercise 5.7.
p
3
5
7
11
13
17

#E(Fp )
4
9
5
14
18
18

tp
0
−3
3
−2
−4
0

√
2 p
3.46
4.47
5.29
6.63
7.21
8.25

Section. The elliptic curve discrete logarithm problem
5.8. Let E be the elliptic curve
E : y 2 = x3 + x + 1
and let P = (4, 2) and Q = (0, 1) be points on E modulo 5. Solve the elliptic
curve discrete logarithm problem for P and Q, that is, find a positive integer n
such that Q = nP .
Solution to Exercise 5.8.
We compute the multiples of P :
P = (4, 2),

2P = (3, 4),

6P = (2, 1),

3P = (2, 4),

7P = (3, 1),

4P = (0, 4),

8P = (4, 3),

5P = (0, 1)

9P = O.

Thus logP (Q) = 5 in E(F5 ). It turns out that E(F5 ) contains 9 points, and
the multiples of P give all of them.
5.9. Let E be an elliptic curve over Fp and let P and Q be points in E(Fp ).
Assume that Q is a multiple of P and let n0 > 0 be the smallest solution
to Q = nP . Also let s > 0 be the smallest solution to sP = O. Prove that
every solution to Q = nP looks like n0 + is for some i ∈ Z. (Hint. Write n as
n = is + r for some 0 ≤ r < s and determine the value of r.)
Solution to Exercise 5.9.
Following the hint, we write n as n = is + r for some 0 ≤ r < s. Then
Q = nP = (is + r)P = i(sP ) + rP = iO + rP = rP,
since by definition sP = O. But n0 P is the smallest multiple of P that is
equal to Q, so we must have r ≥ n0 . If r = n0 , we’re done, so suppose instead
that r > n0 .

148

Exercises for Chapter 5

Then
O = Q − Q = rP − n0 P = (r − n0 )P,
and we know that sP is the smallest (nonzero) multiple of P that is equal
to O, so r − n0 ≥ s. But this contradicts r < s. Hence r = n0 , which proves
that n = is + n0 .
5.10. Use the double-and-add algorithm (Table 5.3) to compute nP in E(Fp )
for each of the following curves and points, as we did in Figure 5.4.
(a)

E : Y 2 = X 3 + 23X + 13,
2

3

2

3

(b) E : Y = X + 143X + 367,
(c)

p = 83,

P = (24, 14),

n = 19;

p = 613,

P = (195, 9),

n = 23;

E : Y = X + 1828X + 1675, p = 1999, P = (1756, 348), n = 11;

(d) E : Y 2 = X 3 + 1541X + 1335, p = 3221, P = (2898, 439), n = 3211.
Solution to Exercise 5.10.
(a) Solution: 19 ∗ (24, 14) = (24, 69).
Step i n Q = 2i P
R
0
19 (24, 14)
O
1
9
(30, 8)
(24, 14)
2
4
(24, 69) (30, 75)
3
2
(30, 75) (30, 75)
4
1
(24, 14) (30, 75)
5
0
(30, 8)
(24, 69)
Compute 19 · (24, 14) on Y 2 = X 3 + 23X + 13 modulo 83.
(b) Solution: 23 ∗ (195, 9) = (485, 573).
Step i n
0
23
1
11
2
5
3
2
4
1
5
0
Compute 23 · (195, 9) on

Q = 2i P
R
(195, 9)
O
(407, 428)
(195, 9)
(121, 332) (182, 355)
(408, 110) (194, 565)
(481, 300) (194, 565)
(401, 150) (485, 573)
Y 2 = X 3 + 143X + 367 modulo 613

(c) Solution: 11 ∗ (1756, 348) = (1068, 1540).
Step i n
Q = 2i P
R
0
11 (1756, 348)
O
1
5 (1526, 1612) (1756, 348)
2
2 (1657, 1579) (1362, 998)
3
1
(1849, 225)
(1362, 998)
4
0
(586, 959)
(1068, 1540)
Compute 11 · (1756, 348) on Y 2 = X 3 + 1828X + 1675 modulo 1999

Exercises for Chapter 5

149

(d) Solution: 3211 ∗ (2898, 439) = (243, 1875).
Step i
n
Q = 2i P
R
0
3211 (2898, 439)
O
1
1605 (2964, 2977) (2898, 439)
2
802 (1372, 2349) (781, 2494)
3
401 (2956, 1288) (781, 2494)
4
200 (1045, 1606) (341, 1727)
5
100
(770, 285)
(341, 1727)
6
50
(2589, 1698) (341, 1727)
7
25
(2057, 2396) (341, 1727)
8
12
(1017, 828) (2117, 1162)
9
6
(1988, 1949) (2117, 1162)
10
3
(1397, 1477) (2117, 1162)
11
1
(420, 1274)
(2362, 757)
12
0
(2583, 2597) (243, 1875)
Compute 3211 · (2898, 439) on Y 2 = X 3 + 1541X + 1335 modulo 3221
5.11. Convert the proof of Proposition 5.18 into an algorithm and use it to
write each of the following numbers n as a sum of positive and negative powers
of 2 with at most 12 blog nc + 1 nonzero terms. Compare the number of nonzero
terms in the binary expansion of n with the number of nonzero terms in the
ternary expansion of n.
(a) 349.
(b) 9337.
(c) 38728.
(d) 8379483273489.
Solution to Exercise 5.11.
(a) Binary expansion has 6 terms. Ternary expansion has 5 terms.
349 = +21 + 23 + 24 + 25 + 27 + 29
= +21 − 23 − 26 − 28 + 210
(b) Binary expansion has 7 terms. Ternary expansion has 5 terms.
9337 = +21 + 24 + 25 + 26 + 27 + 211 + 214
= +21 − 24 + 28 + 211 + 214
(c) Binary expansion has 7 terms. Ternary expansion has 6 terms.
38728 = +24 + 27 + 29 + 210 + 211 + 213 + 216
= +24 + 27 − 29 − 212 + 214 + 216
(d) Binary expansion has 21 terms. Ternary expansion has 10 terms.
8379483273489 = +21 + 25 + 29 + 212 + 213 + 214 + 216 + 217 + 218 + 219
+ 220 + 221 + 233 + 234 + 235 + 236 + 237 + 240 + 241
+ 242 + 243
= +21 + 25 + 29 − 212 − 215 + 222 − 233 + 238 − 240 + 244

150

Exercises for Chapter 5

5.12. In Section 4.5 we gave an abstract description of Pollard’s ρ method,
and in Section 4.5.2 we gave an explicit version to solve the discrete logarithm
problem in Fp . Adapt this material to create a Pollard ρ algorithm to solve
the ECDLP.
Solution to Exercise 5.12.
We want to find n so that Q = nP , where P, Q ∈ E(Fp ) are given. We
also assume that we know an integer N such that N P = O and N Q = O.
For example, we can take N = #E(Fp ). To apply Pollard’s method, we need
a function f : E(Fp ) → E(Fp ) that mixes up the points reasonably well.
Following the ideas from Section 4.5.2, we define


P + T if 0 ≤ xT < p/3,
f : E(Fp ) −→ E(Fp ),
f (T ) = 2T
if p/3 ≤ xT < 2p/3,


Q + T if 2p/3 ≤ xT < p.
Then after i steps, we have
f i (O) = αi P + βi Q

and

f 2i (O) = γi P + δi Q

for certain integer values of αi , βi , γi , δi . We can keep track of the values of αi , βi , γi , δi just as we did in Section 4.5.2. Note that the values
of αi , βi , γi , δi should be computed modulo N , which prevents them from
getting too big.
√
After O( N ) steps, we expect to find a match
f i (O) = f 2i (O).
This means that
(αi − γi )P = (δi − βi )Q

in E(Fp ).

If gcd(δi − βi , N ) = 1, we can multiply both sides by
(δi − βi )−1 mod N
to express Q as a multiple of P . More generally, we can use the same sort of
calculation described in Section 4.5.2 to find gcd(δi − βi , N ) possible values
of n, and then we can test each of them to see if nP is equal to Q. (In
practice, N will be prime, or at worst a small multiple of a large prime, so
there will be few cases to check.)
Section. Elliptic curve cryptography
5.13. Alice and Bob agree to use elliptic Diffie–Hellman key exchange with
the prime, elliptic curve, and point
p = 2671,

E : Y 2 = X 3 + 171X + 853,

P = (1980, 431) ∈ E(F2671 ).

Exercises for Chapter 5

151

(a) Alice sends Bob the point QA = (2110, 543). Bob decides to use the secret
multiplier nB = 1943. What point should Bob send to Alice?
(b) What is their secret shared value?
(c) How difficult is it for Eve to figure out Alice’s secret multiplier nA ? If
you know how to program, use a computer to find nA .
(d) Alice and Bob decide to exchange a new piece of secret information using
the same prime, curve, and point. This time Alice sends Bob only the
x-coordinate xA = 2 of her point QA . Bob decides to use the secret
multiplier nB = 875. What single number modulo p should Bob send to
Alice, and what is their secret shared value?
Solution to Exercise 5.13.
(a) Bob sends B = g b = 2871 ≡ 805 (mod 1373) to Alice. Their shared
value (a) Bob sends the point QB = 1943P = (1432, 667) ∈ E(F2671 ) to
Alice.
(b) Their secret shared value is the x-coordinate x=2424 of the point
nB QA = 1943(2110, 543) = (2424, 911) ∈ E(F2671 ).
(c) By hand, it takes a long time to find nA . But p is small enough that it’s
not too hard on a computer. Alice’s secret value was nA = 2045, but it turns
out that the point P has order 1319, so in the smallest value that works is
nA = 726 , since 726P = (2110, 543) = QA ∈ E(F2671 ).
(d) Bob computes QB = 875P = (161, 2040) ∈ E(F2671 ), but he sends Alice only the x-coordinate xB = 161 . In order to find the shared value, Bob
computes
2
yA
= x3A + 171xA + 853 = 23 + 171 · 2 + 853 = 1203,

yA = 1203(2671+1)/4 = 1203668 ≡ 2575 (mod 2671),
nB (xA , yA ) = 875(2, 2575) = (1708, 1419) ∈ E(F2671 ).
The shared value is the x-coordinate x = 1708 .
5.14. Exercise 2.10 on page 107 describes a multistep public key cryptosystem
based on the discrete logarithm problem for Fp . Describe a version of this cryptosystem that uses the elliptic curve discrete logarithm problem. (You may
assume that Alice and Bob know the order of the point P in the group E(Fp ),
i.e., they know the smallest integer N ≥ 1 with the property that N P = O.)
Solution to Exercise 5.14.
A solution for this exercise is not currently available.
5.15. A shortcoming of using an elliptic curve E(Fp ) for cryptography is the
fact that it takes two coordinates to specify a point in E(Fp ). However, as
discussed briefly at the end of Section 5.4.2, the second coordinate actually
conveys very little additional information.

152

Exercises for Chapter 5

(a) Suppose that Bob wants to send Alice the value of a point R ∈ E(Fp ). Explain why it suffices for Bob to send Alice the x-coordinate of R = (xR , yR )
together with the single bit
(
0 if 0 ≤ yR < 12 p,
βR =
1 if 21 p < yR < p.
(You may assume that Alice is able to efficiently compute square roots
modulo p. This is certainly true, for example, if p ≡ 3 (mod 4); see Proposition 2.27.)
(b) Alice and Bob decide to use the prime p = 1123 and the elliptic curve
E : Y 2 = X 3 + 54X + 87.
Bob sends Alice the x-coordinate x = 278 and the bit β = 0. What point
is Bob trying to convey to Alice? What about if instead Bob had sent
β = 1?
Solution to Exercise 5.15.
(a) Alice computes x3R + AxR + B. This quantity has two square roots,
say b and p − b. One of b or p − b is between 0 and 12 p, the other is between 21 p
and p. So the value of βR tells Alice exactly which square root to take for yR .
(b) First compute u = 2783 + 54 · 278 + 87 ≡ 216 (mod 1123). Then compute
u(1123+1)/4 ≡ 487 (mod 1123). So the two possible points are (278, 487) and
(278, 636), since 636 = 1123 − 487. From the way that β is chosen, we have
β = 0 =⇒ R = (278, 487)

and

β = 1 =⇒ R = (278, 636)

5.16. The Menezes–Vanstone variant of the elliptic ElGamal public key cryptosystem improves message expansion while avoiding the difficulty of directly
attaching plaintexts to points in E(Fp ). The MV-ElGamal cryptosystem is
described in Table 5.12 on page 343.
(a) The last line of Table 5.12 claims that m01 = m1 and m02 = m2 . Prove
that this is true, so the decryption process does work.
(b) What is the message expansion of MV-ElGamal?
(c) Alice and Bob agree to use
p = 1201,

E : Y 2 = X 3 + 19X + 17,

P = (278, 285) ∈ E(Fp ),

for MV-ElGamal. Alice’s secret value is nA = 595. What is her public key?
Bob sends Alice the encrypted message ((1147, 640), 279, 1189). What is
the plaintext?
Solution to Exercise 5.16.

Exercises for Chapter 5

153

(a) Suppose that Bob has encrypted the plaintext (m1 , m2 ) using the random number k as described in Table 5.12 and that he sends Alice his ciphertext (R, c1 , c2 ). Alice’s first step is to compute T = nA R. However, using the
definition of R, S and QA , we see that Alice is actually computing
T = nA R = nA (kP ) = k(nA P ) = kQA = S.
Thus xS = xT and yS = yT , so Alice’s second step yields
−1
m01 ≡ x−1
T c1 ≡ xS (xS m1 ) ≡ m1

(mod p),

m02 ≡ yT−1 c2 ≡ yS−1 (yS m2 ) ≡ m2

(mod p).

This shows that Alice recovers Bob’s plaintext. Notice how she uses her secret
multiplier nA during the decryption process.
(b) The plaintext (m1 , m2 ) consists of two numbers modulo p. The ciphertext (R, c1 , c2 ) consists of four numbers modulo p, since R has two coordinates.
So the message expansion ratio is 2-to-1 . This can be improved if Bob sends
only the x-coordinate of R, plus one extra bit to enable Alice to determine the
correct y-coordinate. In that case, the cipher text is three numbers modulo p
(plus one bit), so the message expansion ratio is approximately 3-to-2 .
(c) Alice public key is QA = nA P = 595 · (278, 285) = (1104, 492) . To
decrypt, Alice computes T = nA (1147, 640) = 595(1147, 640) = (942, 476).
−1
She then computes x−1
· 279 ≡ 509 (mod 1201) and yT−1 c2 =
T c1 = 941
476−1 · 1189 ≡ 767 (mod 1201). So the plaintext is (509, 767) .
5.17. This exercise continues the discussion of the MV-ElGamal cryptosystem
described in Table 5.12 on page 343.
(a) Eve knows the elliptic curve E and the ciphertext values c1 and c2 . Show
how Eve can use this knowledge to write down a polynomial equation
(modulo p) that relates the two pieces m1 and m2 of the plaintext. In
particular, if Eve can figure out one piece of the plaintext, then she can
recover the other piece by finding the roots of a certain polynomial modulo p.
(b) Alice and Bob exchange a message using MV-ElGamal with the prime,
elliptic curve, and point in Exercise 5.16(c). Eve intercepts the ciphertext
((269, 339), 814, 1050) and, through other sources, she discovers that the
first part of the plaintext is m1 = 1050. Use your algorithm in (a) to
recover the second part of the plaintext.
Solution to Exercise 5.17.
(a) Eve knows the equation of the elliptic curve,
E : Y 2 = X 3 + AX + B.
The coordinates of the point S ∈ E(Fp ) satisfy

154

Exercises for Chapter 5
xS ≡ m−1
1 c1

(mod p),

m−1
2 c2

(mod p),

yS ≡

and the point (xS , yS ) satisfies the equation for E, so Eve knows that
−1
−1
2
3
(m−1
2 c2 ) ≡ (m1 c1 ) + A(m1 c1 ) + B

(mod p).

Eve clears denominators by multiplying by m31 m22 , so
c22 m31 ≡ c31 m22 + Ac1 m21 m22 + Bm31 m22

(mod p).

Thus (m1 , m2 ) is a solution to the congruence
c22 u3 ≡ c31 v 2 + Ac1 u2 v 2 + Bu3 v 2

(mod p),

so in particular, if Eve knows either m1 or m2 , then she can find the other
one by substituting in the known value and finding the roots modulo p of the
resulting polynomial.
(b) m2 = 179. A solution for this exercise is not currently available.
Section. Lenstra’s elliptic curve factorization algorithm
5.18. Use the elliptic curve factorization algorithm to factor each of the numbers N using the given elliptic curve E and point P .
(a)

E : Y 2 = X 3 + 4X + 9,

N = 589,

(b) N = 26167,

3

P = (2, 12).

2

3

P = (1, 1).

2

3

P = (7, 4).

E : Y = X + 4X + 128,

(c) N = 1386493,

E : Y = X + 3X − 3,

(d) N = 28102844557,

P = (2, 5).

2

E : Y = X + 18X − 453,

Solution to Exercise 5.18.
(a)
n
n! · P mod 589
1
P = (2, 5)
2 2! · P = (564, 156)
3 3! · P = (33, 460)
4 4! · P = (489, 327)
Factorial multiples of P on Y 2 = X 3 + 4X + 9 modulo 589
Computation of 5! · P gives 589 = 19 · 31.
(b)
n
1
2
3
4
5
6

n! · P mod 26167
P = (2, 12)
2! · P = (23256, 1930)
3! · P = (21778, 1960)
4! · P = (22648, 14363)
5! · P = (5589, 11497)
6! · P = (7881, 16198)

Exercises for Chapter 5

155

Public Parameter Creation
A trusted party chooses and publishes a (large) prime p,
an elliptic curve E over Fp , and a point P in E(Fp ).
Alice
Bob
Key Creation
Chooses a secret multiplier nA .
Computes QA = nA P .
Publishes the public key QA .
Encryption
Chooses plaintext values m1 and m2
modulo p.
Chooses a random number k.
Computes R = kP .
Computes S = kQA and writes it
as S = (xS , yS ).
Sets c1 ≡ xS m1 (mod p) and
c2 ≡ yS m2 (mod p).
Sends ciphtertext (R, c1 , c2 ) to Alice.
Decryption
Computes T = nA R and writes
it as T = (xT , yT ).
Sets m01 ≡ x−1
T c1 (mod p) and
m02 ≡ yT−1 c2 (mod p).
Then m01 = m1 and m02 = m2 .
Table 5.1: Menezes–Vanstone variant of ElGamal (Exercises 5.16, 5.17)
Factorial multiples of P on Y 2 = X 3 + 4X + 128 modulo 26167.
Computation of 7! · P gives 26167 = 191 · 137.
(c)
n
1
2
3
4
5
6
7
8
9
10

P
2! · P
3! · P
4! · P
5! · P
6! · P
7! · P
8! · P
9! · P
10! · P

n! · P mod 1386493
= (1, 1)
= (7, 1386474)
= (1059434, 60521)
= (81470, 109540)
= (870956, 933849)
= (703345, 474777)
= (335675, 1342927)
= (1075584, 337295)
= (149824, 1003869)
= (92756, 1156933)

156

Exercises for Chapter 5
Factorial multiples of P on Y 2 = X 3 + 3X − 3 modulo 1386493
Computation of 11! · P gives 1386493 = 1069 · 1297.

(d)
n
1
P
2
2! · P
3
3! · P
4
4! · P
5
5! · P
..
.
24 24! · P
25 25! · P
26 26! · P
27 27! · P
28 28! · P
Factorial multiples of P

n! · P mod 28102844557
= (7, 4)
= (1317321250, 11471660625)
= (15776264786, 10303407105)
= (27966589703, 26991329662)
= (11450520276, 14900134804)
..
.
= (25959867777, 9003083411)
= (10400016599, 11715538594)
= (22632202481, 6608272585)
= (25446531195, 2223850203)
= (12412875644, 7213676617)
on Y 2 = X 3 + 18X + 28102844104 modulo
28102844557.
Computation of 29! · P gives 28102844557 = 117763 · 238639.

Section. Elliptic curves over F2 and over F2k
5.19. Let E be an elliptic curve given by a generalized Weierstrass equation
E : Y 2 + a1 XY + a3 Y = X 3 + a2 X 2 + a4 X + a6 .
Let P1 = (x1 , y1 ) and P2 = (x2 , y2 ) be points on E. Prove that the following
algorithm computes their sum P3 = P1 + P2 .
First, if x1 = x2 and y1 + y2 + a1 x2 + a3 = 0, then P1 + P2 = O.
Otherwise define quantities λ and ν as follows:
[If x1 6= x2 ]
[If x1 = x2 ]

y2 − y1
y1 x2 − y2 x1
,
ν=
,
x2 − x1
x2 − x1
−x31 + a4 x1 + 2a6 − a3 y1
3x2 + 2a2 x1 + a4 − a1 y1
, ν=
.
λ= 1
2y1 + a1 x1 + a3
2y1 + a1 x1 + a3

λ=

Then
P3 = P1 + P2 = (λ2 + a1 λ − a2 − x1 − x2 , −(λ + a1 )x3 − ν − a3 ).
Solution to Exercise 5.19.
This is proven in any basic text on elliptic curves. See for example Group
Law Algorithm 2.3 in [123, §2.2].
5.20. Let F8 = F2 [T ]/(T 3 + T + 1) be as in Example 5.28, and let E be the
elliptic curve
E : Y 2 + XY + Y = X 3 + T X + (T + 1).

Exercises for Chapter 5

157

(a) Calculate the discriminant of E.
(b) Verify that the points
P = (1 + T + T 2 , 1 + T ),

Q = (T 2 , T ),

R = (1 + T + T 2 , 1 + T 2 ),

are in E(F8 ) and compute the values of P + Q and 2R.
(c) Find all of the points in E(F8 ).
(d) Find a point P ∈ E(F8 ) such that every point in E(F8 ) is a multiple of P .
Solution to Exercise 5.20.
(a) ∆ = 1 + T 2 .
(b) P + Q = (1 + T + T 2 , 1 + T 2 ) and 2R = (T 2 , T ).
(c, d) The point P = (1 + T + T 2 , 1 + T 2 ) satisfies
P = (1 + T + T 2 , 1 + T 2 )
2P = (T 2 , T )
3P = (1, 0)
4P = (T 2 , 1 + T + T 2 )
5P = (1 + T + T 2 , 1 + T )
6P = O,
and this is the complete set of points in E(F8 ). (One can check this directly, or
note that if there were more points, since the order of an element divides the
order of a group, it would follow
√ that #E(F8 ) is at least 12, which contradicts
the Hasse bound of 8 + 1 + 2 8 ≈ 11.83.) The multiples of the point 5P also
give all of #E(F8 ).
5.21. Let τ (α) = αp be the Frobenius map on Fpk .
(a) Prove that
τ (α+β) = τ (α)+τ (β)

and

τ (α·β) = τ (α)·τ (β)

for all α, β ∈ Fpk .

(Hint. For the addition formula, use the binomial theorem (Theorem 4.10).)
(b) Prove that τ (α) = α for all α ∈ Fp .
(c) Let E be an elliptic curve over Fp and let τ (x, y) = (xp , y p ) be the
Frobenius map from E(Fpk ) to itself. Prove that
τ (P + Q) = τ (P ) + τ (Q)

for all P ∈ E(Fpk ).

5.22. Let E0 be the Koblitz curve Y 2 + XY = X 3 + 1 over the field F2 , and
for every k ≥ 1, let
tk = 2k + 1 − #E(F2k ).
(a) Prove that t1 = −1 and t2 = −3.

158

Exercises for Chapter 5

(b) Prove that tk satisfies the recursion
tk = t1 tk−1 − ptk−2

for all t ≥ 3.

(You may use the formula (5.12) that we stated, but did not prove, on
page 313.)
(c) Use the recursion in (b) to compute #E(F16 ).
(d) Program a computer to calculate the recursion and use it to compute the
values of #E(F211 ), #E(F231 ), and #E(F2101 ).
Solution to Exercise 5.22.
(a) A solution for this exercise is not currently available.
(b) A solution for this exercise is not currently available.
(c) #E(F16 ) = 16.
(d) #E(F211 ) = 2116.
#E(F231 ) = 2147574356.
#E(F2101 ) = 2535301200456455833701195805484.
5.23. Let τ satisfy τ 2 = −2 − τ . Prove that the following algorithm gives
coefficients vi ∈ {−1, 0, 1} such that the positive integer n is equal to
n = v0 + v1 τ + v2 τ 2 + · · · + v` τ ` .

(5.1)

Further prove that at most one-third of the vi are nonzero and that ` ≤ log(n).
[1] Set n0 = n and n1 = 0 and i = 0
[2] Loop while n0 6= 0 or n1 6= 0
[3]
If n0 is odd
¡
¢
[4]
Set vi = 2 − (n0 − 2n1 ) mod 4
[5]
Set n0 = n0 − vi
[6]
Else
[7]
Set vi = 0
[8]
End If
[9]
Set i = i + 1 ¡
¢
[10]
Set (n0 , n1 ) = n1 − 12 n0 , − 12 n0
[11] End Loop
Solution to Exercise 5.23.
A solution for this exercise is not currently available.
5.24. Implement the algorithm in Exercise 5.23 and use it to compute the τ expansion (5.19) of the following integers. What is the highest power of τ that
appears and how many nonzero terms are there?
(a) n = 931

(b) n = 32755

Solution to Exercise 5.24.
(a)

(c) n = 82793729188

Exercises for Chapter 5

159

931 = −1 + τ 2 + τ 10 + τ 14 − τ 17 − τ 19 − τ 21 .
The highest power of τ is τ 21 and the τ -expansion has 7 nonzero terms.
(b)
32755 = −1 + τ 2 + τ 4 + τ 6 + τ 8 + τ 15 − τ 17 + τ 19 − τ 22 + τ 28 − τ 31 .
The highest power of τ is τ 31 and the τ -expansion has 11 nonzero terms.
(c)
82793729188 = τ 2 + τ 8 − τ 10 − τ 12 + τ 15 + τ 18
+ τ 20 − τ 24 − τ 27 + τ 30 − τ 34 + τ 36 − τ 40
+ τ 44 + τ 46 − τ 48 + τ 50 − τ 52 + τ 55 + τ 58
+ τ 61 + τ 68 − τ 71 − τ 73 .
The highest power of τ is τ 73 and the τ -expansion has 24 nonzero terms.
Section. Bilinear pairings on elliptic curves
5.25. Let R(x) and S(x) be rational functions. Prove that the divisor of a
product is the sum of the divisors, i.e.,
¡
¢
¡
¢
¡
¢
div R(x)S(x) = div R(x) + div S(x) .
5.26. Prove that the Weil pairing satisfies
em (P, Q) = em (Q, P )−1

for all P, Q, ∈ E[m].

(Hint. Use the fact that em (P + Q, P + Q) = 1 and expand using bilinearity.)
5.27. This exercise asks you to verify that the Weil pairing em is welldefined.
(a) Prove that the value of em (P, Q) is independent of the choice of rational
functions fP and fQ .
(b) Prove that the value of em (P, Q) is independent of the auxiliary point S.
(Hint. Fix the points P and Q and consider the quantity
Á
fP (Q + S) fQ (P − S)
F (S) =
fP (S)
fQ (−S)
as a function of S. Compute the divisor of F and use the fact that every
nonconstant function on E has at least one zero.)
You might also try to prove that the Weil pairing is bilinear, but do not be
discouraged if you do not succeed, since the standard proofs use more tools
than we have developed in the text.
5.28. Choose a basis {P1 , P2 } for E[m] and write each P ∈ E[m] as a linear
combination P = aP P1 + bP P2 . (See Remark 5.39.) Use the basic properties
of the Weil pairing described in Theorem 5.38 to prove that
³a a ´
em (P, Q) = em (P1 , P2 )

det

P

Q

bP bQ

= em (P1 , P2 )aP bQ −aQ bP .

160

Exercises for Chapter 5

5.29. Complete the proof of Proposition 5.51 by proving that φ(2P ) = 2φ(P ).
5.30. For each of the following elliptic curves E, finite fields Fp , points P
and Q of order m, and auxiliary points S, use Miller’s algorithm to compute
the Weil pairing em (P, Q). (See Example 5.43.)

(a)
(b)
(c)
(d)

E

p

P

Q

m

S

y 2 = x3 + 23

1051

(109 203)

(240 203)

5

(1,554)

2

3

y = x − 35x − 9

883

(5, 66)

(103, 602)

7

(1,197)

2

3

1009

(8, 703)

(49, 20)

7

(0,0)

2

3

1009

(417, 952)

(561, 153)

7

(0,0)

y = x + 37x
y = x + 37x

Notice that (c) and (d) use the same elliptic curve. Letting P 0 and Q0 denote
the points in (d), verify that
P 0 = 2P,

Q0 = 3Q,

and

e7 (P 0 , Q0 ) = e7 (P, Q)6 .

Solution to Exercise 5.30.
(a) We have #E(F1051 = 1075 = 52 ·43. The point S has order 215. Miller’s
algorithm gives
fP (Q + S)
109
=
= 203
fP (S)
306

and

fQ (P − S)
552
=
= 312.
fQ (−S)
406

Taking the ratio of these two values yields
e5 (P, Q) =

203
= 671 ∈ F1051 .
312

(b) We have #E(F883 ) = 882 = 2 · 32 · 72 The point S has order 126. Miller’s
algorithm gives
fP (Q + S)
387
=
= 730
fP (S)
413

and

fQ (P − S)
454
=
= 469.
fQ (−S)
161

Taking the ratio of these two values yields
e7 (P, Q) =

730
= 749 ∈ F883 .
469

(c) We have #E(F1009 ) = 980 = 22 · 5 · 72 . The point S has order 2. Miller’s
algorithm gives
92
fP (Q + S)
=
= 739
fP (S)
478

and

Taking the ratio of these two values yields

fQ (P − S)
800
=
= 574.
fQ (−S)
810

Exercises for Chapter 5

161

e7 (P, Q) =

739
= 105 ∈ F1009 .
574

(d) Miller’s algorithm gives
fP (Q + S)
86
=
= 384
fP (S)
531

and

fQ (P − S)
919
=
= 969.
fQ (−S)
759

Taking the ratio of these two values yields
e7 (P, Q) =

384
= 394 ∈ F1009 .
969

Finally, we check that
e7 (P, Q)6 = 1056 = 394 = e7 (P 0 , Q0 ),
which is in accordance with P 0 = 2P and Q0 = 3Q.
5.31. Let E over Fq and ` be as described in Theorem 5.44. Prove that the
modified Tate pairing is symmetric, in the sense that
τ̂ (P, Q) = τ̂ (Q, P )

for all P, Q ∈ E(Fq )[`].

Solution to Exercise 5.31.
By assumption we have E(Fq )[`] = Z/`Z, a cyclic group. Let T be a
generator. Then any P, Q ∈ E(Fq )[`] can be written as P = uT and Q = vT
for some u, v ∈ Z/`Z. But then the linearity of that Tate pairing gives
τ̂ (P, Q) = τ̂ (uT, vT ) = τ̂ (T, T )uv ,
τ̂ (Q, P ) = τ̂ (vT, uT ) = τ̂ (T, T )vu ,
which are clearly the same value.
5.32. Let E be an elliptic curve over Fq and let P, Q ∈ E(Fq )[`]. Prove that
the Weil pairing and the Tate pairing are related by the formula
e` (P, Q) =

τ (P, Q)
,
τ (Q, P )

provided that the Tate pairings on the right-hand side are computed properly.
Thus the Weil pairing requires approximately twice as much work to compute
as does the Tate pairing.
Section. The Weil pairing over fields of prime power order
5.33. Prove Proposition 5.51(b) in the case P1 = P2 .
5.34. Let E be an elliptic curve over Fp and let ` be a prime. Suppose
√
that E(Fp ) contains a point of order ` and that ` > p + 1. Prove that
E(Fp )[`] ∼
= Z/`Z.

162

Exercises for Chapter 5

Solution to Exercise 5.34.
Hasse’s theorem says that
√
√
#E(Fp ) ≤ p + 1 + 2 p = ( p + 1)2 .
The assumption on ` then tells us that #E(Fp ) < `2 . But if E(Fp )[`] is larger
than Z/`Z, then it is equal to Z/`Z × Z/`Z, so we would have `2 elements,
contradicting #E(Fp ) < `2 .
5.35. Let E be an elliptic curve over a finite field Fq and let ` be a prime.
Suppose that we are given four points P, aP, bP, cP ∈ E(Fq )[`]. The (elliptic)
decision Diffie–Hellman problem is to determine whether cP is equal to abP .
Of course, if we could solve the Diffie–Hellman problem itself, then we could
compute abP and compare it with cP , but the Diffie–Hellman problem is often
difficult to solve.
Suppose that there exists a distortion map φ for E[`]. Show how to use
the modified Weil pairing to solve the elliptic decision Diffie–Hellman problem
without actually having to compute abP .
Solution to Exercise 5.35.
Compute
ê` (aP, bP ) = ê` (P, P )ab

and

ê` (P, cP ) = ê` (P, P )c .

If they agree, then cP = abP , otherwise cP 6= abP .
5.36. Let E be the elliptic curve E : y 2 = x3 + x and let φ(x, y) = (−x, αy)
be the map described in Proposition 5.51. Prove that φ(φ(P
√ )) = −P for
all P ∈ E. (Intuitively, φ behaves like multiplication by −1 when it is
applied to points of E.)
Solution to Exercise 5.36.
Let P = (x, y). We compute
¡
¢
φ(φ(P )) = φ(−x, αy) = −(−x), α · αy = (x, α2 y) = (x, −y) = −P.

5.37. Let p ≡ 3 (mod 4), let E : y 2 = x3 + x, let P ∈ E(Fp )[`], and
let φ(x, y) = (−x, αy) be the `-distortion map for P described in Proposition 5.52. Suppose further that ` ≡ 3 (mod 4). Prove that φ is an `-distortion
map for every point in E[`]. In other words, if Q ∈ E is any point of order `,
prove that e` (Q, φ(Q)) is a primitive `th root of unity.
Solution to Exercise 5.37.
We can write Q = aP + bφ(P ), since {P, φ(P )} is a basis for E[`]. We have
φ(Q) = φ(aP + bφ(P )) = aφ(P ) + bφ(φ(P )).

Exercises for Chapter 5

163

Note that
φ(φ(P )) = φ(φ(x, y)) = φ(−x, αy) = (x, α2 y) = (x, −y) = −P.
(This was a previous exercise.) So φ(Q) = −bP + aφ(P ). Hence
e` (Q, φ(Q)) = e` (aP + bφ(P ), −bP + aφ(P ))
2

2

= e` (P, P )−ab e` (P, φ(P ))a e` (φ(P ), P )−b e` (φ(P ), φ(P ))ab
= e` (P, φ(P ))a

2

+b2

.

We know that e` (P, φ(P )) is a primitive `th -root of unity, so either e` (Q, φ(Q))
is a primitive `th -root of unity, or else a2 + b2 is a multiple of `. (Note that
we can assume that take 0 ≤ a, b < ` and that a and b are not both 0.) But
if ` divides a2 + b2 , then we get
µ 2 ¶ µ 2 ¶ µ ¶µ 2 ¶ µ ¶
a
−b
−1 b
−1
1=
=
=
=
,
`
`
`
`
`
so −1 is a square modulo `. From an easy piece of quadratic reciprocity, this
implies that ` ≡ 1 (mod 4), contradicting our assumption that ` ≡ 3 (mod 4).
5.38. Let E be the elliptic curve
E : y 2 = x3 + 1
over a field K, and suppose that K contains an element β 6= 1 satisfying β 3 =
1. (We say that β is a primitive cube root of unity.) Define a map φ by
φ(x, y) = (βx, y) and

φ(O) = O.

(a) Let P ∈ E(K). Prove that φ(P ) ∈ E(K).
(b) Prove that φ respects the addition law on E, i.e., φ(P1 + P2 ) = φ(P1 ) +
φ(P2 ) for all P1 , P2 ∈ E(K).
5.39. Let E : y 2 = x3 + 1 be the elliptic curve in Exercise 5.38.
(a) Let p ≥ 3 be a prime with p ≡ 2 (mod 3). Prove that Fp does not contain
a primitive cube root of unity, but that Fp2 does contain a primitive cube
root of unity.
(b) Let β ∈ Fp2 be a primitive cube root of unity and define a map φ(x, y) =
(βx, y) as in Exercise 5.38. Suppose that E(Fp ) contains a point P of
prime order ` ≥ 5. Prove that φ is an `-distortion map for P .
Solution to Exercise 5.39.
(b) This is the same as the proof of Proposition 5.52. The multiples of P
are in E(Fp ), but φ(P ) is not unless its x-coordinate is 0. Then on checks
that points on E of the form (0, y) are points of order 3. Hence φ(P ) is
not a multiple of P , and then Proposition 5.49 tells us that e` (P, φ(P )) is a
primitive `th -root of unity.

164

Exercises for Chapter 5

5.40. Let E be the elliptic curve E : y 2 = x3 + x over the field F691 . The
point P = (301, 14) ∈ #E(F691 ) has order 173. Use the distortion map on E
from Exercises 5.38 and 5.39 to compute ê173 (P, P ) (cf. Example 5.54). Verify
that the value is a primitive 173rd root of unity.
Solution to Exercise 5.40.
We have φ(P ) = (−301, 14i) = (390, 14i). We randomly choose a point
S = (499 + 325i, 41 + 140i) ∈ E(F6912 )
and use Miller’s algorithm to compute
452 + 325i
fP (φ(P ) + S)
=
= 432 + 271i,
fP (S)
236 + 219i
fφ(P ) (P − S)
48 + 608i
=
= 259 + 271i.
fφ(P ) (−S)
115 + 533i
Then
ê(P, P ) = e173 (P, φ(P )) =

432 + 271i
= 242 + 92i ∈ F6912 .
259 + 271i

We check that (242 + 92i)173 = 1.
5.41. Continuing with the curve E, prime p = 691, and point P = (301, 14)
from Exercise 5.40, let
Q = (143, 27) ∈ E(F691 ).
Use the MOV method to solve the ECDLP for P and Q, i.e., compute ê173 (P, Q)
and express it as the nth power of ê173 (P, P ). Check your answer by verifying
that nP is equal to Q.
Solution to Exercise 5.41.
The distortion map gives φ(Q) = (548, 278i), and we use the randomly
chosen point S = (379 + 605i, 205 + 534i) ∈ E(F6912 ) to compute
ê173 (P, Q) = e173 (P, φ(Q)) =

139+432i
506+550i
239+375i
142+299i

= 500 + 603i ∈ F6912 .

From the previous exercise we have ê173 (P, P ) = 242+92i, so we need to solve
the DLP
(242 + 92i)n = 500 + 603i in F6912 .
The solution to this DLP is n = 122, and we can check that Q = P , so n = 122
is also a solution to the ECDLP.
Section. Applications of the Weil pairing

Exercises for Chapter 5

165

5.42. Alice, Bob, and Carl use tripartite Diffie–Hellman with the curve
E : y 2 = x3 + x

over the field F1723 .

They use the point
P = (668, 995) of order 431.
(a) Alice chooses the secret value nA = 278. What is Alice’s public point QA ?
(b) Bob’s public point is QB = (1275, 1550) and Carl’s public point is QC =
(897, 1323). What is the value of ê431 (QB , QC )?
(c) What is their shared value?
(d) Bob’s secret value is nB = 224. Verify that ê431 (QA , QC )nB is the same
as the value that you got in (c).
(e) Figure out Carl’s secret value nC . (Since P has order 431, you can do
this on a computer by trying all possible values.)
Solution to Exercise 5.42.
(a) Alice’s public point is QA = nA P = (726, 1127).
(b) ê431 (QB , QC ) = 1444 + 1288i.
(c) The shared value is ê431 (QB , QC )278 = (1444 + 1288i)278 = 68 + 428i.
(d) ê431 (QA , QC )224 = (1264 + 1083i)224 = 68 + 428i.
(e) Carl’s secret value is nC = 145. We check that he gets the same shared
value, ê431 (QA , QB )145 = (977 + 1163i)145 = 68 + 428i.
5.43. Show that Eve can break tripartite Diffie–Hellman key exchange as
described in Table 5.10.1 if she knows how to solve the Diffie–Hellman problem
(page 67) for the field Fq .
Solution to Exercise 5.43.
Eve can compute
ê` (P, P )

and

ê` (QA , P ) = ê` (nA P, P ) = ê` (P, P )nA .

But she can also compute
ê` (QB , QC ) = ê` (P, P )nB nC .
Thus Eve knows the quantities
g nA

and

g nB nC

for a certain primitive `th root of unity g in F∗q . If she can solve the Diffie–
Hellman problem in F∗q , then she can use these known values to compute Alice,
Bob, and Carl’s shared value g nA nB nC .

Chapter 6

Lattices and Cryptography
Exercises for Chapter 6
Section. A congruential public key cryptosystem
6.1. Alice uses the congruential cryptosystem with q = 918293817 and private
key (f, g) = (19928, 18643).
(a) What is Alice’s public key h?
(b) Alice receives the ciphertext e = 619168806 from Bob. What is the plaintext?
(c) Bob sends Alice a second message by encrypting the plaintext m = 10220
using the ephemeral key r = 19564. What is the ciphertext that Bob sends
to Alice?
Solution to Exercise 6.1.
(a) h = 767748560.
(b) First compute
a ≡ f e ≡ 600240756 (mod q)
Then

m = f −1 a = 9764 · 600240756 ≡ 11818 (mod g).

(The ephemeral key was 19564.)
(c)
e ≡ rh + m ≡ 619167208 (mod q).
Section. Subset-sum problems and knapsack cryptosystems
6.2. Use the algorithm described in Proposition 6.5 to solve each of the following subset-sum problems. If the “solution” that you get is not correct,
explain what went wrong.
(a) M = (3, 7, 19, 43, 89, 195), S = 260.
167

168

Exercises for Chapter 6

(b) M = (5, 11, 25, 61, 125, 261), S = 408.
(c) M = (2, 5, 12, 28, 60, 131, 257), S = 334.
(d) M = (4, 12, 15, 36, 75, 162), S = 214.
Solution to Exercise 6.2.
(a) Output from algorithm is x = (1, 0, 1, 1, 0, 1). Sum is correct.
(b) Output from algorithm is x = (1, 1, 0, 0, 1, 1). Sum is 402 instead of 408.
Incorrect. Superincreasing, but this S has no solution.
(c) Output from algorithm is x = (0, 1, 1, 0, 1, 0, 1). Sum is correct.
(d) Output from algorithm is x = (0, 0, 1, 1, 0, 1). Sum is 213 instead of 214.
Incorrect. M is not superincreasing, this problem has a solution (1, 1, 0, 1, 0, 1),
but it is not found by the algorithm.
6.3. Alice’s public key for a knapsack cryptosystem is
M = (5186, 2779, 5955, 2307, 6599, 6771, 6296, 7306, 4115, 7039).
Eve intercepts the encrypted message S = 26560. She also breaks into Alice’s
computer and steals Alice’s secret multiplier A = 4392 and secret modulus
B = 8387. Use this information to find Alice’s superincreasing private sequence r and then decrypt the message.
Solution to Exercise 6.3.
A solution for this exercise is not currently available.
6.4. Proposition 6.3 gives an algorithm that solves an n-dimensional knapsack problem in O(2n/2 ) steps, but it requires O(2n/2 ) storage. Devise an
algorithm, similar to Pollard’s ρ algorithm (Section 4.5), that takes O(2n/2 )
steps, but requires only O(1) storage.
Solution to Exercise 6.4.
A solution for this exercise is not currently available.
Section. A brief review of vector spaces
6.5. (a) Let
B = {(1, 3, 2), (2, −1, 3), (1, 0, 2)},

B0 = {(−1, 0, 2), (3, 1, −1), (1, 0, 1)}.

Each of the sets B and B 0 is a basis for R3 . Find the change of basis
matrix that transforms B0 into B.
(b) Let v = (2, 3, 1) and w = (−1, 4, −2). Compute the lengths kvk and kwk
and the dot product v · w. Compute the angle between v and w.
Solution to Exercise 6.5.
(a) Let


1 3 2
B = 2 −1 3
1 0 2



−1 0 2
and C =  3 1 −1
1 0 1

Exercises for Chapter 6
Then

 −1
C −1 = 

(b) kvk =

√

3
4
3
1
3

0
1
0

169


2
3
−5 
3
1
3

 13

and

A = BC −1


3 −11
3
= −1 −1 4 
1
0 43
3
3

√
14 ≈ 3.7417. kwk = 21 ≈ 4.5826. v · w = 8.
√
√
cos(θ) = 8/ 14 · 21 ≈ 0.4666, so

θ ≈ cos−1 (0.4666) ≈ 1.0854 radians ≈ 62.188 degrees.

6.6. Use the Gram–Schmidt algorithm (Theorem 6.13) to find an orthogonal
basis from the given basis.
(a) v1 = (1, 3, 2), v2 = (4, 1, −2), v3 = (−2, 1, 3).
(b) v1 = (4, 1, 3, −1), v2 = (2, 1, −3, 4), v3 = (1, 0, −2, 7).
Solution to Exercise 6.6.
(a)
v1∗ = (1, 3, 2),

v2∗ = (53/14, 5/14, −17/7),

v3∗ = (56/285, −14/57, 77/285).

(b)
v1∗ = (4, 1, 3, −1), v2∗ = (70/27, 31/27, −23/9, 104/27),
v3∗ = (−287/397, −405/397, 799/397, 844/397).
Section. Lattices: Basic definitions and properties
6.7. Let L be the lattice generated by {(1, 3, −2), (2, 1, 0), (−1, 2, 5)}. Draw a
picture of a fundamental domain for L and find its volume.
Solution to Exercise 6.7.
The volume is
¯

¯
¯
1 3 −2 ¯¯
¯
¯det  2 1 0 ¯ = 35.
¯
¯
¯
−1 2 5 ¯
6.8. Let L ⊂ Rm be an additive subgroup with the property that there is a
positive constant ² > 0 such that
©
ª
L ∩ w ∈ Rm : kwk < ² = {0}.
Prove that L is discrete, and hence is a lattice. (In other words, show that in
the defintion of discrete subgroup, it suffices to check that (6.8) is true for the
single vector v = 0.)

170

Exercises for Chapter 6

Solution to Exercise 6.8.
A solution for this exercise is not currently available.
6.9. Prove that a subset of Rm is a lattice if and only if it is a discrete additive
subgroup.
Solution to Exercise 6.9.
A solution for this exercise is not currently available.
6.10. This exercise describes a result that you may have seen in your linear
algebra course.
Let A be an n-by-n matrix with entries aij , and for each pair of indices i
and j, let Aij denote the (n − 1)-by-(n − 1) matrix obtained by deleting the ith
row of A and the j th column of A. Define a new matrix B whose ij th entry bij
is given by the formula
bij = (−1)i+j det(Aji ).
(Note that bij is the determinant of the submatrix Aji , i.e., the indices are
reversed.) The matrix B is called the adjoint of A.
(a) Prove that
AB = BA = det(A)In ,
where In is the n-by-n identity matrix.
(b) Deduce that if det(A) 6= 0, then
A−1 =

1
B.
det(A)

(c) Suppose that A has integer entries. Prove that A−1 exists and has integer
entries if and only if det(A) = ±1.
(d) For those who know ring theory from Section 2.10 or from some other
source, suppose that A has entries in a ring R. Prove that A−1 exists and
has entries in R if and only if det(A) is a unit in R.
Solution to Exercise 6.10.
A solution for this exercise is not currently available.
6.11. Recall from Remark 6.16 that the general linear group GLn (Z) is the
group of n-by-n matrices with integer coefficients and determinant ±1. Let A
and B be matrices in GLn (Z).
(a) Prove that AB ∈ GLn (Z).
(b) Prove that A−1 ∈ GLn (Z).
(c) Prove that the n-by-n identity matrix is in GLn (Z).
(d) Prove that GLn (Z) is a group. (Hint. You have already done most of the
work in proving (a), (b), and (c). For the associative law, either prove it
directly or use the fact that you know that it is true for matrices with
real coefficients.)

Exercises for Chapter 6

171

(e) Is GLn (Z) a commutative group?
Solution to Exercise 6.11.
(a) By assumption, A−1 and B −1 have integer entries. Hence (AB)−1 =
−1 −1
B A also has integer entries, so AB ∈ GLn (Z).
(b) (A−1 )−1 is equal to A, so it has integer entries. Hence A−1 ∈ GLn (Z).
(c) Let I be the identity matrix. Then I has integer entries, and I −1 = I
also has integer entries, so I ∈ GLn (Z).
(d) As the hint says, (a), (b) and (c) show that the product and inverse of
matrices in GLn (Z) are again in GLn (Z) and the identity matrix is in GLn (Z),
so really just need to check the associative law (AB)C = A(BC). But you
proved the associative law for matrix multiplication in linear algebra when
the entries are real numbers (or maybe even more generally), so it is certainly
true when the entries are integers.
(e) No, GLn (Z) is not commutative for n ≥ 2. For example, ( 11 01 ) and ( 10 11 )
do not commute, and similar examples exist for any n ≥ 2. Of course, for n =
1, GL1 (Z) = {±1} is commutative.
6.12. Which of the following matrices are in GLn (Z)? Find the inverses of
those matrices that are in GLn (Z).
µ ¶
31
22


3 22
(c) A3 =  2 1 2
−1 3 1

(a)

A1 =

Solution to Exercise 6.12.
(a) No, since det = 4.
(b) Yes, since det = 1. A−1
2 =

µ
¶
−1 2
.
−2 3

(c) No, since det = −9.



(d) Yes, since det = 1. A−2
4

µ
¶
3 −2
2 −1


−3 −1 2
(d) A4 =  1 −3 −1
3 0 −2

(b) A2 =


6 −2 7
= −1 0 −1.
9 −3 10

6.13. Let L be the lattice given by the basis
©
ª
B = (3, 1, −2), (1, −3, 5), (4, 2, 1) .
Which of the following sets of vectors are also bases for L? For those that are,
express the new basis in terms of the basis B, i.e., find the change of basis
matrix.
(a) B1 = {(5, 13, −13), (0, −4, 2), (−7, −13, 18)}.
(b) B2 = {(4, −2, 3), (6, 6, −6), (−2, −4, 7)}.

172

Exercises for Chapter 6

Solution to Exercise 6.13.
(a) Yes. The change of basis matrix is





5 13 −13
0 −3 2
3 1 −2
 0 −4 2  . =  1 1 −1 1 −3 5 
−7 −13 18
−2 3 −1
4 2 1
The inverse matrix is

−1 
231
0 −3 2
 1 1 −1 = 3 4 2
563
−2 3 −1


which shows that B and B1 generate the same lattice.
(b) No, since det(B) = −48 and det(B2 ) = 96.
6.14. Let L ⊂ Rm be a lattice of dimension n and let v1 , . . . , vn be a basis
for L. (Note that we are allowing n to be smaller than m.) The Gram matrix
of v1 , . . . , vn is the matrix
¡
¢
Gram(v1 , . . . , vn ) = vi · vj 1≤i,j≤n .
(a) Let F (v1 , . . . , vn ) be the matrix (6.11) described in Proposition (6.20),
except that now F (v1 , . . . , vn ) is an n-by-m matrix, so it need not be
square. Prove that
Gram(v1 , . . . , vn ) = F (v1 , . . . , vn )F (v1 , . . . , vn )t ,
where F (v1 , . . . , vn )t is the transpose matrix, i.e., the matrix with rows
and columns interchanged.
(b) If m = n, prove that
¡
¢
det Gram(v1 , . . . , vn ) = det(L)2 .
(6.1)
(c) In general, prove that det Gram(v1 , . . . , vn ) is the square of the volume
of a fundamental domain for L, so we can use (6.61) to compute det(L).
(d) Let L ⊂ R4 be the 3-dimensional lattice with basis
v1 = (1, 0, 1, −1),

v2 = (1, 2, 0, 4),

v3 = (1, −1, 2, 1).

Compute the Gram matrix of this basis and use it to compute det(L).
(e) Let v1∗ , . . . , vn∗ be the Gram–Schmidt orthogonalized vectors (Theorem 6.13) associated to v1 , . . . , vn . Prove that
Gram(v1 , . . . , vn ) = kv1∗ k2 kv2∗ k2 · · · kvn∗ k2 .

Exercises for Chapter 6

173

Solution to Exercise 6.14.
(a–c) A solution for this exercise is not currently available.
(d)


 1 1 1



1 0 1 −1 
3 −3 2

0
2
−1
 

Gram(v1 , . . . , vn ) = 1 2 0 4  
 1 0 2  = −3 21 3
1 −1 2 1
2 3 7
−1 4 1
Then
det(L) =

p
√
det Gram(v1 , . . . , vn ) = 231.

Section. The shortest and closest vector problems
6.15. Let L be a lattice and let F be a fundamental domain for L. This
exercise sketches a proof that
¡
¢
# BR (0) ∩ L
1
¡
¢ =
lim
.
(6.2)
R→∞ Vol BR (0)
Vol(F)
(a) Consider the translations of F that are entirely contained within BR (0),
and also those that have nontrivial intersection with BR (0). Prove the
inclusion of sets
[
[
(F + v) ⊂ BR (0) ⊂
(F + v).
v∈L
F +v⊂BR (0)

v∈L
(F+v)∩BR (0)6=∅

(b) Take volumes in (a) and prove that
©
ª
# v ∈ L : F + v ⊂ BR (0) · Vol(F)
¡
¢
©
ª
≤ Vol BR (0) ≤ # v ∈ L : (F + v) ∩ BR (0) 6= ∅ · Vol(F).
(Hint. Proposition 6.18 says that the different translates of F are disjoint.)
(c) Prove that the number of translates F + v that intersect BR (0) without
being entirely contained within BR (0) is comparatively small compared
to the number of translates Fv that are entirely contained within BR (0).
(This is the hardest part of the proof.)
(d) Use (b) and (c) to prove that
¡
¢
¡
¢
Vol BR (0) = # BR (0) ∩ L · Vol(F) + (smaller term).
¡
¢
Divide by Vol BR (0) and let R → ∞ to complete the proof of (6.62).
Solution to Exercise 6.15.
A solution for this exercise is not currently available.

174

Exercises for Chapter 6

6.16. A lattice L of dimension n = 251 has determinant det(L) ≈ 22251.58 .
With no further information, approximately how large would you expect the
shortest nonzero vector to be?
Solution to Exercise 6.16.
The Gaussian heuristic (6.21) predicts that the shortest nonzero vector in
L has length approximately
r
n
σ(L) =
(det L)1/n ≈ 1922.96.
2πe
Section. Babai’s algorithm and solving CVP with a “good” basis
6.17. Let L ⊂ R2 be the lattice given by the basis v1 = (213, −437) and
v2 = (312, 105), and let w = (43127, 11349).
(a) Use Babai’s algorithm to find a vector v ∈ L that is close to w. Compute
the distance kv − wk.
(b) What is the value of the Hadamard ratio det(L)/kv1 kkv2 k? Is the basis {v1 , v2 } a “good” basis?
(c) Show that the vectors v10 = (2937, −1555) and v20 = (11223, −5888) are
also a basis for L by expressing them as linear combinations of v1 and v2
and checking that the change-of-basis matrix has integer coefficients and
determinant ±1.
(d) Use Babai’s algorithm with the basis {v10 , v20 } to find a vector v0 ∈ L.
Compute the distance kv0 − wk and compare it to your answer from (a).
(e) Compute the Hadamard ratio using v10 and v20 . Is {v10 , v20 } a good basis?
Solution to Exercise 6.17.
(a) (t1 , t2 ) = (6.22, 133.98), so v = 6v1 + 134v2 = (43086, 11448). Then
kv − wk = 107.15.
(b) det(L)/kv1 kkv2 k = 158709/(486.15)(329.19) = 0.9917. The ratio is close
to 1, so {v10 , v20 } a good basis.
5 6 ) = 1.
(c) v10 = 5v1 + 6v2 and v20 = 19v1 + 23v2 . We have det ( 19
23
0
0
(d) (t1 , t2 ) = (−2402.52, 632.57), so v = −2403v1 + 633v20 = (46548, 9561).
Then kv − wk = 3860.08.
(e) det(L)/kv10 kkv20 k = 158709/(3323.25)(12673.76) = 0.00377. The ratio is
very small, so {v10 , v20 } a bad basis.
Section. The GGH public key cryptosystem
6.18. Alice uses the GGH cryptosystem with private basis
v1 = (4, 13),
and public basis
w1 = (25453, 9091),

v2 = (−57, −45),
w2 = (−16096, −5749).

Exercises for Chapter 6

175

(a) Compute the determinant of Alice’s lattice and the Hadamard ratio of
the private and public bases.
(b) Bob sends Alice the encrypted message e = (155340, 55483). Use Alice’s
private basis to decrypt the message and recover the plaintext. Also determine Bob’s random perturbation r.
(c) Try to decrypt Bob’s message using Babai’s algorithm with the public
basis {w1 , w2 }. Is the output equal to the plaintext?
Solution to Exercise 6.18.
(a) det(L) = 561, The Hadamard ratio of the private key is 0.75362. and
the Hadamard ratio of the public key is 0.0011.
(b)
e ≈ −6823.12v1 − 3204.08v2 .
v = −6823v1 − 3204v2
= (155336, 55481)
= 8w1 + 3w2 .
So the plaintext is m = (8, 3). Also r = w − v = (4, 2).
(c)
e ≈ −8.39w1 − 22.92w2 .
This yields the incorrect plaintext (−8, −23).
6.19. Alice uses the GGH cryptosystem with private basis
v1 = (58, 53, −68),

v2 = (−110, −112, 35),

v3 = (−10, −119, 123)

and public basis
w1 = (324850, −1625176, 2734951),
w2 = (165782, −829409, 1395775),
w3 = (485054, −2426708, 4083804).
(a) Compute the determinant of Alice’s lattice and the Hadamard ratio of
the private and public bases.
(b) Bob sends Alice the encrypted message e = (8930810, −44681748, 75192665).
Use Alice’s private basis to decrypt the message and recover the plaintext.
Also determine Bob’s random perturbation r.
(c) Try to decrypt Bob’s message using Babai’s algorithm with the public
basis {w1 , w2 , w3 }. Is the output equal to the plaintext?
Solution to Exercise 6.19.
(a) det(L) = −672858, The Hadamard ratio of the private key is 0.61697
and the Hadamard ratio of the public key is 0.00003.
(b)

176

Exercises for Chapter 6
e ≈ −334865.23v1 − 304373.02v2 + 512803.95v3 .
v = −334865v1 − 304373v2 + 512804v3
= (8930820, −44681745, 75192657)
= −50w1 − 91w2 + 83w3 .

So the plaintext is m = (−50, −91, 83). Also r = w − v = (−10, −3, 8).
(c)
e ≈ 51.59w1 + 416.67w2 − 158.55w3 .
This yields the incorrect plaintext (52, 417, −159).
6.20. Bob uses the GGH cryptosystem to send some messages to Alice.
(a) Suppose that Bob sends the same message m twice, using different random perturbations r and r0 . Explain what sort of information Eve can
deduce from the ciphertexts e = mW + r and e0 = mW + r0 .
(b) For example, suppose that n = 5 and that random permutations are
chosen with coordinates in the set {−2, −1, 0, 1, 2}. This means that there
are 55 = 3125 possibilities for r. Suppose further that Eve intercepts two
ciphertexts
e = (−9, −29, −48, 18, 48)

and e0 = (−6, −26, −51, 20, 47)

having the same plaintext. With this information, how many possibilities
are there for r?
(c) Suppose that Bob is lazy and uses the same perturbation to send two
different messages. Explain what sort of information Eve can deduce from
the ciphertexts e = mW + r and e0 = m0 W + r.
Solution to Exercise 6.20.
(a) Eve can compute e0 − e = r0 − r and use this information to narrow
down the possibilities for r and r0 .
(b) Eve computes
e − e0 = r − r0 = (−3, −3, 3, −2, 1).
Thus
r1 = r10 − 3,

r2 = r20 − 3,

r3 = r30 + 3,

r4 = r40 − 2,

r5 = r50 + 1.

Further, Eve knows that all of the ri and all of the ri0 are between −2 and 2.
Thus each equation puts some restrictions on the coordinates of r. For example
r1 = r10 − 3 ≤ 2 − 3 = −1,
and similarly

so

r1 ∈ {−2, −1},

Exercises for Chapter 6
r2
r3
r4
r5

177

= r20 − 3 ≤ −2 + 3 = 1,
= r30 + 3 ≥ −2 + 3 = 1,
= r40 − 2 ≤ 2 − 2 = 0,
= r50 + 1 ≥ −2 + 1 = −1,

so
so
so
so

r2
r3
r4
r1

∈ {−2, −1},
∈ {1, 2},
∈ {−2, −1, 0},
∈ {−1, 0, 1, 2}.

Hence the number of possibilities for r has been reduced to 2 · 2 · 2 · 3 · 3 = 72,
which is far less than 3125.
(c) This time Eve can compute (e − e0 )W −1 = m − m0 , and then the fact
that m and m0 are small again allows Eve to narrow down the possibilities.
Section. Convolution polynomial rings
6.21. Compute (by hand!) the polynomial convolution product c = a ? b
using the given value of N .
a(x) = −1 + 4x + 5x2 ,

b(x) = −1 − 3x − 2x2 ;

(b) N = 5,

a(x) = 2 − x + 3x3 − 3x4 ,

b(x) = 1 − 3x2 − 3x3 − x4 ;

(c)

a(x) = x + x2 + x3 ,

b(x) = 1 + x + x5 ;

(a)

N = 3,
N = 6,

(d) N = 10,

a(x) = x + x2 + x3 + x4 + x6 + x7 + x9 ,
b(x) = x2 + x3 + x6 + x8 .

Solution to Exercise 6.21.
(a) c = a ? b = −22 − 11x − 15x2 .
(b) c = a ? b = −6 − x + 3x3 − 2x4 .
(c) c = a ? b = 1 + 2x + 3x2 + 2x3 + x4 .
(d) c = a ? b = 3 + 2x + 3x2 + 2x3 + 3x4 + 4x5 + 2x6 + 3x7 + 2x8 + 4x9 .
6.22. Compute the polynomial convolution product c = a ? b modulo q using
the given values of q and N .
(a)

b(x) = −5 + 4x + 2x2 ;

N = 3,

q = 7,

a(x) = 1 + x,

(b) N = 5,

q = 4,

a(x) = 2 + 2x − 2x2 + x3 − 2x4 ,
b(x) = −1 + 3x − 3x2 − 3x3 − 3x4 ;

(c)

N = 7,

(d) N = 10,

q = 3,

a(x) = x + x3 ,

b(x) = x + x2 + x4 + x6 ;

q = 2,

a(x) = x2 + x5 + x7 + x8 + x9 ,
b(x) = 1 + x + x3 + x4 + x5 + x7 + x8 + x9 .

Solution to Exercise 6.22.
(a) c ≡ a ? b ≡ 4 + 6x + 6x2 (mod 7).
(b) c ≡ a ? b ≡ 1 + x + x2 + 3x3 + 3x4 (mod 4).
(c) c ≡ a ? b ≡ 2 + 2x2 + x3 + x4 + 2x5 (mod 3).
(d) c ≡ a ? b ≡ x + x2 + x4 + x6 (mod 2).
6.23. Let a(x) ∈ (Z/qZ)[x], where q is a prime.

178

Exercises for Chapter 6

(a) Prove that
a(1) ≡ 0 (mod q) if and only if

(x − 1) | a(x) in (Z/qZ)[x].

(b) Suppose that a(1) ≡ 0 (mod q). Prove that a(x) is not invertible in Rq .
Solution to Exercise 6.23.
(a) Working in (Z/qZ)[x], we use division with remainder to divide a(x)
by x − 1. The result is
a(x) = (x − 1)b(x) + r(x) with deg r < deg(x − 1) = 1.
Thus either r(x) = 0 or deg r(x) = 0, so in any case, r(x) is a constant. Thus
a(x) = (x − 1)b(x) + c for some c ∈ Z/qZ.
We can determine c by substituting x = 1, which gives c = a(1). Thus
a(x) = (x − 1)b(x) + a(1).
Now (a) is obvious, since this equation shows that a(x) is a multiple of x − 1
if and only if a(1) = 0.
(b) Suppose that a(x) is invertible in Rq , say a(x)b(x) = 1 in Rq . We have
a well-defined map Rq → Z/qZ defined by evaluating a polynomial at x = 1.
This map is well-defined because the extra relation xN = 1 is true when we
set x = 1. Further, the map respects addition and multiplication. Hence the
relation a(x)b(x) = 1 in Rq leads to the relation a(1)b(1) = 1 in Z/qZ. In
particular, we certainly can’t have a(1) = 0. This proves
a(x) invertible

=⇒

a(1) 6= 0,

which is equivalent to the statement
a(1) = 0

=⇒

a(x) is not invertible.

6.24. Let N = 5 and q = 3 and consider the two polynomials
a(x) = 1 + x2 + x3 ∈ R3

and

b(x) = 1 + x2 − x3 ∈ R3 .

One of these polynomials has an inverse in R3 and the other does not. Compute the inverse that exists, and explain why the other doesn’t exist.
Solution to Exercise 6.24.
a(x) does not have an inverse, because a(1) ≡ 0 (mod 3). The previous
exercise then implies that a(x) does not have an inverse. Alternatively, using
the Euclidean algorithm, one finds that

Exercises for Chapter 6

179

¡
¢
gcd a(x), x5 − 1 = 1 − x in (Z/3Z)[x],
so a(x) does not have an inverse from Proposition 6.45.
Similarly, gcd(b(x), x5 − 1) = 1 in (Z/3Z)[x], and using the extended
Euclidean algorithm, we find that
b(x)−1 = 1 − x − x2 − x3

in (Z/3Z)[x].

6.25. For each of the following values of N , q, and a(x), either find a(x)−1
in Rq or show that the inverse does not exist.
(a) N = 5, q = 11, and a(x) = x4 + 8x + 3;
(b) N = 5, q = 13, and a(x) = x3 + 2x − 3.
(c) N = 7, q = 23, and a(x) = 20x6 + 8x5 + 4x4 + 15x3 + 19x2 + x + 8.
Solution to Exercise 6.25.
(a) a(x)−1 = 7x4 + 8x3 + 3x2 + 2x + 3 in F11 [x].
(b) gcd(a(x), x5 − 1) = x + 12 in F13 [x], so no inverse.
(c) a(x)−1 = 17x6 + 4x5 + 12x4 + 18x2 + 12x + 10 in F23 [x].
6.26. This exercise illustrates how to find inverses in
Rm =

(Z/mZ)[x]
(xN − 1)

when m is a prime power pe .
(a) Let f (x) ∈ Z[x]/(X N − 1) be a polynomial, and suppose that we have
already found a polynomial F (x) such that
f (x) ? F (x) ≡ 1

(mod pi )

for some i ≥ 1. Prove that the polynomial
¡
¢
G(x) = F (x) ? 2 − f (x) ? F (x)
satisfies
f (x) ? G(x) ≡ 1 (mod p2i ).
(b) Suppose that we know an inverse of f (x) modulo p. Using (a) repeatedly,
how many convolution multiplications does it take to compute the inverse
of f (x) modulo pe ?
(c) Use the method in (a) to compute the following inverses modulo m = pe ,
where to ease your task, we have given you the inverse modulo p.

180

Exercises for Chapter 6
(i)

N = 5, m = 24 ,

f (x) = 7 + 3x + x2 ,
f (x)−1 ≡ 1 + x2 + x3 (mod 2).

(ii) N = 5, m = 27 ,

f (x) = 22 + 11x + 5x2 + 7x3 ,
f (x)−1 ≡ 1 + x2 + x3 (mod 2).

(iii)

N = 7, m = 55 ,

f (x) = 112 + 34x + 239x2 + 234x3 + 105x4
+ 180x5 + 137x6 ,
f (x)−1 ≡ 1 + 3x2 + 2x4 (mod 5).

Solution to Exercise 6.26.
(a) We have
f G − 1 = f (F (2 − f F )) − 1
= 2f F − (f F )2 − 1
= −(f F − 1)2 .
We are assuming that f F ≡ 1 (mod pi ), say f F = 1 + pi H. Then
f G − 1 = −(f F − 1)2 = p2i H,
so f G ≡ 1 (mod p2i ).
(b) Each iteration of (a) takes two convolution multiplications, and each doubles the exponent of p. So after k iterations, we’ve done 2k convolution multik
plications and we have an inverse of f modulo p2 . So we need 2k ≥ e, which
means that k = dlog2 ee (or one less, if e is a power of 2). Then the number
of convolution multiplications is 2dlog2 ee.
(c) (i) f (x)−1 mod 24 = 13 + 5X 2 + 7X 3 + 10X 4 .
(ii) f (x)−1 mod 27 = 101 + 12X + X 2 + 17X 3 + 34X 4 .
(iii) f (x)−1 mod 55 = 840 + 711X + 710X 2 + 268X 3 + 1710X 4 + 1142X 5 +
2430X 6 .
Section. The NTRU public key cryptosystem
6.27. Alice and Bob agree to communicate using the NTRU cryptosystem
with
(N, p, q) = (7, 2, 37).
Alice’s private key is
f (x) = x + x3 + x6

and

F2 (x) = 1 + x + x4 + x5 + x6 .

(You can check that f ? F2 ≡ 1 (mod 2).) Alice receives the ciphertext
e(x) = 1 + 3x + 3x2 + 4x3 + 4x4 + x5 + 35x6
from Bob. Decipher the message and find the plaintext.

Exercises for Chapter 6

181

Solution to Exercise 6.27.
Alice first computes
a ≡ f ? e ≡ 5 + 5x + 5x2 + 8x3 + 8x4 + 5x5 + 6x6

(mod 37).

Then she computes
F2 ? a = 29 + 31x + 31x2 + 32x3 + 32x4 + 29x5 + 26x6
≡ 1 + x + x2 + x5

(mod 2).

The plaintext is m = 1 + x + x2 + x4 + x6 .
6.28. Alice and Bob decide to communicate using the NTRU cryptosystem
with parameters (N, p, q) = (7, 2, 29). Alice’s public key is
h(x) = 23 + 23x + 23x2 + 24x3 + 23x4 + 24x5 + 23x6 .
Bob sends Alice the plaintext message m(x) = 1 + x5 using the ephemeral
key r(x) = 1 + x + x3 + x6 .
(a) What ciphertext does Bob send to Alice?
(b) Alice’s secret key is f (x) = 1 + x + x2 + x4 + x5 and F2 (x) = 1 + x5 + x6 .
Check your answer in (a) by using f and F2 to decrypt the message.
Solution to Exercise 6.28.
(a) c ≡ 2r?h+m ≡ 11+12x+12x2 +12x3 +14x4 +13x5 +14x6 (mod 29).
(b) First compute
b ≡ f ? c ≡ 4 + 5x + 4x2 + 5x3 + 5x4 + 4x5 + 7x6

(mod 37).

Then compute
F ? b = 13 + 14x + 14x2 + 14x3 + 16x4 + 15x5 + 16x6
≡ 1 + x5

(mod 2).

This agrees with the plaintext.
6.29. What is the message expansion of NTRU in terms of N , p, and q?
Solution to Exercise 6.29.
The plaintext is N numbers modulo p, so consists of N log2 (p) bits. The
ciphertext is N numbers modulo q, so consists of N log2 (q) bits. Hence the
message expansion of NTRU is log2 (q)/ log2 (p).
6.30. The guidelines for choosing NTRU public parameters (N, p, q, d) require
that gcd(p, q) = 1. Prove that if p | q, then it is very easy for Eve to decrypt
the message without knowing the private key. (Hint. First do the case that
p = q.)

182

Exercises for Chapter 6

Solution to Exercise 6.30.
We always have
e(x) ≡ pr(x)h(x) + m(x) ≡ m(x) (mod q).
If p = q, then this reduces to e(x) = m(x), so the ciphertext is equal to
the plaintext. In general, if p | q, then reducing e(x) modulo p gives the
plaintext m(x).
6.31. Alice uses the NTRU cryptosystem with p = 3 to send messages to
Bob.
(a) Suppose that Alice uses the same ephemeral key r(x) to encrypt two different plaintexts m1 (x) and m2 (x). Explain how Eve can use the two
ciphertexts e1 (x) and e2 (x) to determine approximately 29 of the coefficients of m1 (x). (See Exercise 6.34 for a way to exploit this information.)
(b) For example, suppose that N = 8, so there are 38 possibilities for m1 (x).
Suppsoe that Eve intercepts two ciphertexts
e1 (x) = 32 + 21x − 9x2 − 20x3 − 29x4 − 29x5 − 19x6 + 38x7 ,
e2 (x) = 33 + 21x − 7x2 − 19x3 − 31x4 − 27x5 − 19x6 + 38x7 ,
that were encrypted using the same ephemeral key r(x). How many coefficients of m1 (x) can she determine exactly? How many possibilities are
there for m1 (x)?
(c) Formulate a similar attack if Alice uses two different ephemeral keys r1 (x)
and r2 (x) to encrypt the same plaintext m(x). (Hint. Do it first assuming that h(x) has an inverse in Rq . The problem is harder without this
assumption.)
Solution to Exercise 6.31.
(a) Eve computes
¡
¢ ¡
¢
e1 (x) − e2 (x) ≡ r(x) ? h(x) + m1 (x) − r(x) ? h(x) + m2 (x) (mod q)
≡ m1 (x) − m2 (x) (mod q).
The coefficients of m1 (x)−m2 (x) are in the set {−2, −1, 0, 1, 2}, so since q > 5,
Eve recovers m1 (x) − m2 (x) exactly. Any coefficient that is nonzero limits the
possibilities for that coefficient of m1 (x). (This is the same as the analogous
GGH exercise.)
More precisely, Eve can recover the ith coefficient of m1 (x) if the ith coefficient of both m1 (x) and m2 (x) are both +1 or both −1. Assuming that the
coefficients are random, the probability of this happening is 2 · 31 · 13 = 92 . So
Eve recovers approximately 29 of the coefficients of m1 (x).
(b) Eve finds that
m1 (x) − m2 (x) = e1 (x) − e2 (x) = −1 − 2x2 − x3 + 2x4 − 2x5

Exercises for Chapter 6

183

The coefficients of x2 , x4 and x5 for m1 are determined, they are −x2 +x4 −x5 .
So Eve knows three of the coefficients of m1 .
More generally, m1 (x) looks like
A + Bx − x2 + Cx3 + x4 − x5 + Dx6 + Ex7 .
Further, Eve knows that A ∈ {0, 1} and C ∈ {0, 1}. So there are 2 · 3 · 2 · 3 · 3 =
108 possibilities for m1 (x), which is much smaller than 38 = 6561.
(c) If h(x) is invertible in Rq , then Eve can compute
¡
¢
¡
¢
h(x)−1 e1 (x) − e2 (x) = h(x)−1 pr1 (x)h(x) − pr2 (x)h(x) (mod q)
= r1 (x) − r2 (x) (mod q).
Then the analysis is the same as in (a), since r1 and r2 have ternary coefficients.
In general, however, h(x) is not invertible, since g(x) is not invertible,
since g(1) = 0. One way around this problem is to develop a theory of “almost
inverses”
on the fact that the ring Z[x]/(xN − 1) is isomorphic to Z ×
¡ based
¢
Z[x]/ Φ(x) , where Φ(x) = xN −1 + xN −2 + · · · + x + 1. The image of g(x) in
the product ring is (0, g(x)), so one inverts the second factor.
Section. NTRU as a lattice cryptosystem
6.32. This exercise explains how to formulate NTRU message recovery as a
closest vector problem. Let h(x) be an NTRU public key and let
e(x) ≡ pr(x) ? h(x) + m(x)

(mod q)

be a message encrypted using h(x).
(a) Prove that the vector (pr, e − m) is in LNTRU
.
h
(b) Prove that the lattice vector in (a) is almost certainly the closest lattice
vector to the known vector (0, e). Hence solving CVP reveals the plaintext m. (For simplicity, you may assume that d ≈ N/3 and q ≈ 2N , as
we did in Proposition 6.61.)
(c) Show how one can reduce the lattice-to-target distance, without affecting
the determinant, by using instead a modified NTRU lattice of the form
µ
¶
1 ph
.
0 q
Solution to Exercise 6.32.
(a) By the definition of e, we can find a polynomial v(x) satisfying
e = pr ? h + m + qv(x).
Thus

184

Exercises for Chapter 6
(pr, v)MhNTRU

µ ¶
1h
= (pr, v)
0q
= (pr, pr ? h + qv)
= (pr, e − m).

This shows that (pr, e − m) is in the NTRU lattice LNTRU
spanned by the
h
rows of MhNTRU . Also notice that
Eve knows this vector

(pr, e − m) =

z }| {
(0, e)

a short vector

z }| {
+ (pr, −m) .

(b) We have
°
° °
°
°(pr, e − m) − (0, e)° = °(pr, −m)°
p
= p2 · 2d + 2d
p
≈ (p2 + 1)2N/3.
√
√
Since p is small, typically 2 or 3, this is between 1.83 N and 2.58 N . But as
in the Proposition, the Gaussian heuristic predicts that a random CVP has a
solution of size approximately σ(LNTRU
) ≈ 0.484N .
h
(c) Let
µ
¶
1 ph
NTRU
=
.
Ah
0 q
Then with notation as in (a), we have
µ

(r, v)ANTRU
h

1 ph
= (r, v)
0 q

¶

= (pr, pr ? h + qv)
= (pr, e − m).
This shows that (r, e − m) is in the lattice spanned by the rows of ANTRU
.
h
N
NTRU
And det(ANTRU
)
=
q
,
which
is
the
same
as
det(M
).
h
h
Now reworking (b), we see that
°
° °
°
°(r, e − m) − (0, e)° = °(r, −m)°
p
√
√
= 4d ≈ 4N/3 ≈ 1.15 N .
So the distance to the closest vector using this new lattice is less than when
using the old lattice.
6.33. The guidelines for choosing NTRU public parameters (N, p, q, d) include
the assumption that N is prime. To see why, suppose (say) that N is even.
Explain how Eve can recover the private key by solving a lattice problem in
dimension N , rather than in dimension 2N . Hint. Use the natural map
Z[x]/(xN − 1) → Z[x]/(xN/2 − 1).

Exercises for Chapter 6

185

Solution to Exercise 6.33.
This method of breaking NTRU when N is composite is due to Craig
Gentry, Key recovery and message attacks on NTRU-composite, Advances in
cryptology—EUROCRYPT 2001 (Innsbruck), Lecture Notes in Comput. Sci.
2045, 182–194, Springer, Berlin, 2001.
6.34. Suppose that Bob and Alice are using NTRU to exchange messages
and that Eve intercepts a ciphertext e(x) for which she already knows part of
the plaintext m(x). (This is not a ludicrous assumption; see Exercise 6.31, for
example.) More precisely, suppose that Eve knows t of the coefficients of m(x).
Explain how to set up a CVP to find m(x) using a lattice of dimension 2N −2t.
Solution to Exercise 6.34.
A solution for this exercise is not currently available.
Section. Lattice reduction algorithms
6.35. Let b1 and b2 be vectors, and set
t = b1 · b2 /kb1 k2

and

b∗2 = b2 − tb1 .

Prove that b∗2 · b1 = 0 and that b∗2 is the projection of b2 onto the orthogonal
complement of b1 .
Solution to Exercise 6.35.
A solution for this exercise is not currently available.
6.36. Let a and b be nonzero vectors in Rn .
(a) What value of t ∈ R minimizes the distance ka − tbk? (Hint. It’s easier
to minimize the value of ka − tbk2 .)
(b) What is the minimum distance in (a)?
(c) If t is chosen as in (a), show that a − tb is the projection of a onto the
orthogonal complement of b.
(d) If the angle between a and b is θ, use your answer in (b) to show that
the minimum distance is kak sin θ. Draw a picture illustrating this result.
Solution to Exercise 6.36.
(a) We have
F (t) = ka − tbk2
= (a − tb) · (a − tb)
= kak2 − 2ta · b + t2 kbk2 .
One can then use calculus (i.e., set F 0 (t) = 0) or complete the square to
minimize the value of the quadratic polynomial. The minimizing value of t is
a·b
t = kbk
2.
(b) Substituting this value of t and simplifying gives the minimum distance

186

Exercises for Chapter 6
s
kak2 kbk2 − (a · b)2
.
kbk2

(c) A solution for this exercise is not currently available.
(d) Substitute a · b = kakkbk cos θ into (b) and use
¡
¢2
kak2 kbk2 − (a · b)2 = kak2 kbk2 − kakkbk cos θ
= kak2 kbk2 (1 − cos2 θ)
= kak2 kbk2 sin2 θ.

6.37. Apply Gauss’s lattice reduction algorithm (Proposition 6.63) to solve
SVP for the following two dimensional lattices having the indicated basis vectors. How many steps does the algorithm take?
(a) v1 = (120670, 110521) and v2 = (323572, 296358).
(b) v1 = (174748650, 45604569) and v2 = (35462559, 9254748).
(c) v1 = (725734520, 613807887) and v2 = (3433061338, 2903596381).
Solution to Exercise 6.37.
(a)
Step
1
2
3
4
5
6
7

v1
(120670, 110521)
(−38438, −35205)
(5356, 4906)
(−946, −863)
(−320, −272)
(14, −47)
(14, −47)

v2
(323572, 296358)
(120670, 110521)
(−38438, −35205)
(5356, 4906)
(−946, −863)
(−320, −272)
(−362, −131)

m
3
−3
−7
−6
3
3
0

The solution to SVP is v = (14, −47).
(b)
Step
1
2
3
4
5
6
7
8

v1
(35462559, 9254748)
(−2564145, −669171)
(−435471, −113646)
(48681, 12705)
(2658, 699)
(837, 123)
(147, 330)
(147, 330)

The solution to SVP is v = (147, 330).
(c)

v2
(174748650, 45604569)
(35462559, 9254748)
(−2564145, −669171)
(−435471, −113646)
(48681, 12705)
(2658, 699)
(837, 123)
(690, −207)

m
5
−14
6
−9
18
3
1
0

Exercises for Chapter 6
Step
1
2
3
4
5
6
7
8
9
10
11
12

187

v1
(725734520, 613807887)
(−195611262, −165443054)
(−56710528, −47964329)
(−25479678, −21550067)
(−5751172, −4864195)
(−2474990, −2093287)
(−801192, −677621)
(−71414, −60424)
(−15638, −12957)
(6776, 4361)
(4690, 126)
(4690, 126)

v2
(3433061338, 2903596381)
(725734520, 613807887)
(−195611262, −165443054)
(−56710528, −47964329)
(−25479678, −21550067)
(−5751172, −4864195)
(−2474990, −2093287)
(−801192, −677621)
(−71414, −60424)
(−15638, −12957)
(6776, 4361)
(2086, 4235)

m
5
−4
3
2
4
2
3
11
5
−3
1
0

The solution to SVP is v = (4690, 126).
6.38. Let V be a vector space, let W ⊂ V be a vector subspace of V , and
let W ⊥ be the orthogonal complement of W in V .
(a) Prove that W ⊥ is also a vector subspace of V .
(b) Prove that every vector v ∈ V can be written as a sum v = w + w0 for
unique vectors w ∈ W and w0 ∈ W ⊥ . (One says that V is the direct sum
of the subspaces W and W ⊥ .)
(c) Let w ∈ W and w0 ∈ W ⊥ and let v = aw + bw0 . Prove that
kvk2 = a2 kwk2 + b2 kw0 k2 .
Solution to Exercise 6.38.
A solution for this exercise is not currently available.
6.39. Let L be a lattice with basis vectors v1 = (161, 120) and v2 =
(104, 77).
(a) Is (0, 1) in the lattice?
(b) Find an LLL reduced basis.
¡
¢
(c) Use the reduced basis to find the closest lattice vector to − 29 , 11 .
Solution to Exercise 6.39.
A solution for this exercise is not currently available.
6.40. Use the LLL algorithm to reduce the lattice with basis
v1 = (20, 16, 3),

v2 = (15, 0, 10),

v3 = (0, 18, 9).

You should do this exercise by hand, writing out each step.
Solution to Exercise 6.40.
Compute
µ2,1 =

300 + 30
330
1
=
< .
400 + (14)2 + 64
665
2

188

Exercises for Chapter 6

Checking the Lovász condition for v2 amounts to checking that kv2 k2 ≥
3
2
2
2
4 kv1 k and kv2 k = 225 + 100 = 325, kv1 k = 665, so swap. Now v1 =
356
and subtract one mul(15, 0, 10) and v2 = (20, 16, 3). Recompute µ2,1 = 325
tiple of v1 from v2 . New v2 = (5, 16, −7). Note that the (new) µ2,1 is now
(75 − 70)/325 = −5/325.
Move on to v3 computing
−75 + 160
85
1
=
< ,
325
325
2
225
1
=
> .
330
2

µ3,1 =
µ3,2

Subtract one multiple of v2 from v3 obtaining the new v3 = (−5, 2, 16).
On to the Lovász condition, computing
kv3∗ +µ3,2 v2∗ k2 = kv3 −µ3,1 v1 k2 = k(−5, 2, 16)−
Next compute
kv2∗ k2 = k

85
1
(15, 0, 10)k2 = k (−14, 26, 174)k2 .
325
13

1
(62, 208, 3)k2
13

and we find the condition (2) is not satisfied, so we swap. At this point,
v1 = (15, 0, 10),

v2 = (−5, 2, 16),

v3 = (5, 16, −7).

Checking condition (2) for (the new) v2 : kv2 k2 = 285, which is larger than
3/4 times kv1 k2 = 325. (If, instead of 3/4, we had chosen a constant closer
to 1, like .99, then we would perform the swap step again. This makes sense
since the length of v2 is smaller than the length of v1 .) Now check the value
of
−105
µ3,2 =
285
and the Lovász condition for v3 , which is satisfied. So we now have an LLL
reduced basis.
6.41. Let L be the lattice generated by the rows of the matrix


20 51 35 59 73 73
 14 48 33 61 47 83 


 95 41 48 84 30 45 

.
M =

 0 42 74 79 20 21 
 6 41 49 11 70 67 
23 36 6 1 46 4
Implement the LLL algorithm (Figure 6.7) on a computer and use your program to answer the following questions.
(a) Compute det(L) and H(M ). What is the shortest basis vector?

Exercises for Chapter 6

189

(b) Apply LLL to M . How many swaps (Step [11]) are required? What is the
value of H(M LLL )? What is the shortest basis vector in the LLL reduced
basis? How does it compare with the Gaussian expected shortest length?
(c) Reverse the order of the rows of M and apply LLL to the new matrix.
How many swaps are required? What is the value of H(M LLL ) and what
is the shortest basis vector?
(d) Apply LLL to the original matrix M , but in the Lovász condition
(Step [8]), use 0.99 instead of 34 . How many swaps are required? What is
the value of H(M LLL ) and what is the shortest basis vector?
Solution to Exercise 6.41.
(a) det(L) = 21242880806, H(M ) = 0.45726, smallest basis vector is
kv6 k = 63.198,
(b) The output is


−6 −3 −2 2 −26 10
 11 30 2
5 −6 24 


−14 −10 14 −48 −3 −6 


 −3 24 43 23 −33 −38


 64 −44 −16 −46 −13 4 
−28 −25 41 5 30 39
There are 11 swap steps. We have H(M LLL ) = 0.91981 and the shortest
vector is kv1 k = 28.792. Gaussian expected shortest is σ(L) = 40.0239. This
suggests that v1 is probably the shortest vector in L.
(c) With the rows in reverse order, the LLL output is


6
3
2 −2 26 −10
 11 30 2 5 −6 24 


 14 10 −14 48 3
6 


−28 −25 41 5 30 39 


 −3 24 43 23 −33 −38
47 −35 54 30 −13 11
There are 8 swap steps. We have H(M LLL ) = 0.94427 and the shortest vector
is kv1 k = 28.792.
(d) With Lovász condition 0.99,


−6 −3 −2 2 −26 10
 11 30 2 5 −6 24 


−14 −10 14 −48 −3 −6 


 −3 24 43 23 −33 −38


−28 −25 41 5 30 39 
47 −35 54 30 −13 11
There are 12 swap steps. We have H(M LLL ) = 0.944270 and the shortest
vector is kv1 k = 28.792. This is the same basis as in (c), in a different order.

190

Exercises for Chapter 6

6.42. A more efficient way to implement the LLL algorithm is described in
Figure 6.8, with Reduce and Swap subroutines given in Figure 6.9. (This
implementation of LLL follows [26, Algorithm 2.6.3]. We thank Henri Cohen
for his permission to include it here.)
(a) Prove that the algorithm described in Figures 6.8 and 6.9 returns an LLL
reduced basis.
(b) For any given N and q, let LN,q be the N -dimensional lattice with basis v1 , . . . , vN described by the formulas
vi = (ri1 , ri2 , . . . , riN ),

rij ≡ (i + N )j (mod q),

0 ≤ rij < q.

Implement the LLL algorithm and use it to LLL reduce LN,q for each of
the following values of N and q:
(i) (N, q) = (10, 541)
(iii) (N, q) = (30, 1223)

(ii) (N, q) = (20, 863)
(iv) (N, q) = (40, 3571)

In each case, compare the Hadamard ratio of the original basis to the
Hadamard ratio of the LLL reduced basis, and compare the length of the
shortest vector found by LLL to the Gaussian expected shortest length.
Solution to Exercise 6.42.
(b) We write L for the original basis and L0 for the LLL reduced basis,
and we write v for the shortest vector in the original basis and v0 for the
shortest vector in the LLL reduced basis. Here are the shortest vectors in the
LLL reduced basis (N.B. the shortest vector was not always the first vector):
(i) v0 = (−98, 166, −131, −18, 100, 28, 81, 50, −39, −39).
(ii) v0 = (−122, −33, −59, 166, 9, −394, −46, 227, −148, −86, −46, 108, −214,
173, −107, 171, 34, −86, −153, −117).
0
(iii) v = (98, −148, −263, −370, 76, 53, 258, −128, 221, −435, −119, −59, 142,
− 336, 311, 290, 89, −538, 16, 437, 108, 361, 322, −374, 56, −117,
− 208, −131, 645, 42).
0
(iv) v = (192, −1426, 552, −292, 52, 482, 1046, −1344, −414, −226, −1413,
− 1466, −447, 653, −484, −553, −284, 232, 1975, 1944, 27, 1203,
− 1363, 707, 91, −549, −831, 974, 768, 1074, 57, −966, 1997,
2099, 828, −1295, −972, −842, 185, −2271).
The lengths, Hadamard ratios, and Gaussian expected shortest lengths are
given in the following table:
(i)
(ii)
(iii)
(iv)

kvk
632.369
1846.49
3133.91
10711.4

kv0 k
278.446
679.056
1505.95
6706.75

H(L)
0.309773
0.253273
0.304603
0.281214

H(L0 )
0.853005
0.694868
0.579003
0.470440

σ(L)
241.775
659.505
1613.89
5775.49

Exercises for Chapter 6
[1]
[2]
[3]
[4]
[5]
[6]
[7]
[8]
[9]
[10]
[11]
[12]
[13]
[14]
[15]
[16]
[17]
[18]
[19]
[20]

191

Input a basis {v1 , . . . , vn } for a lattice L
Set k = 2, kmax = 1, v1∗ = v1 , and B1 = kv1 k2
If k ≤ kmax go to Step [9]
Set kmax = k and vk∗ = vk
Loop j = 1, 2, . . . , k − 1
Set µk,j = vk · vj∗ /Bj and vk∗ = vk∗ − µk,j vj∗
End j Loop
Set Bk = kvk∗ k2
Execute ³Subroutine RED(k,
k − 1)
´
If Bk < 34 − µ2k,k−1 Bk−1
Execute Subroutine SWAP(k)
Set k = max(2, k − 1) and go to Step [9]
Else
Loop ` = k − 2, k − 3, . . . , 2, 1
Execute Subroutine RED(k, `)
End ` Loop
Set k = k + 1
End If
If k ≤ n go to Step [3]
Return LLL reduced basis {v1 , . . . , vn }
Figure 6.1: The LLL algorithm—Main routine

6.43. Let 41 < α < 1 and suppose that we replace the Lovász condition with
the condition
¡
¢ ∗ 2
k for all 1 < i ≤ n.
kvi∗ k2 ≥ α − µ2i,i−1 kvi−1
(a) Prove a more version of Theorem 6.66. What quantity, depending on α,
replaces the 2 that appears in the estimates (6.53), (6.54), and (6.55)?
(b) Prove a version of Theorem 6.68. In particular, how does the upper bound
for the number of swap steps depend on α? What happens as α → 1?
Solution to Exercise 6.43.
A solution for this exercise is not currently available.
6.44. Let v1 , . . . , vn be an LLL reduced basis for a lattice L.
(a) Prove that there are constants C1 > 1 > C2 > 0 such that for
all y1 , . . . , yn ∈ R we have
°X
°2
n
n
X
X
° n
°
n
2
2
n
°
C1
yi kvi k ≥ °
yi vi °
≥
C
yi2 kvi k2 .
(6.3)
2
°
i=1

i=1

i=1

(This is a hard exercise.) We observe that the inequality (6.63) is another
way of saying that the basis v1 , . . . , vn is quasi-orthogonal, since if it

192

Exercises for Chapter 6

[1]
[2]
[3]
[4]
[5]
[6]
[7]

—— Subroutine RED(k, `) ——
If |µk,` | ≤ 12 , return to Main Routine
Set m = bµk,` e
Set vk = vk − mv` and µk,` = µk,` − m
Loop i = 1, 2, . . . , ` − 1
Set µk,i = µk,i − mµ`,i
End i Loop
Return to Main Routine

[1]
[2]
[3]
[4]
[5]
[6]
[7]
[8]
[9]
[10]

—— Subroutine SWAP(k) ——
Exchange vk−1 and vk
Loop j = 1, 2, . . . , k − 2
Exchange µk−1,j and µk,j
End j Loop
Set µ = µk,k−1 and B = Bk − µ2 Bk−1
Set µk,k−1 = µBk−1 /B and Bk = Bk−1 Bk /B and Bk−1 = B
Loop i = k + 1, k + 2, . . . , kmax
Set m = µi,k and µi,k = µi,k−1 − µm and µi,k−1 = m + µk,k−1 µi,k
End i Loop
Return to Main Routine
Figure 6.2: The LLL algorithm—RED and SWAP subroutines

P
2
were
P 2 truly2 orthogonal, then we would have an equality k yi vi k =
yi kvi k .
(b) Prove that there is a constant C such that for any target vector w ∈ Rn ,
Babai’s algorithm (Theorem 6.34) finds a lattice vector v ∈ L satisfying
kw − vk ≤ C n min kw − uk.
u∈L

Thus Babai’s algorithm applied with an LLL reduced basis solves apprCVP to within a factor of C n . This is Theorem 6.73.
(c) Find explicit values for the constants C1 , C2 , and C in (a) and (b).
Solution to Exercise 6.44.
(a) This is a hard exercise. We follow the proof given in [?, §5.7].
Fix a basis v1 , . . . , vn , let v1∗ , . . . , vn∗ be the associated Gram–Schmidt
orthogonalized basis, and let
ei = vi∗ /kvi∗ k,

1 ≤ i ≤ n,

be the associated orthonormal basis. Let µi,j be as usual (settting µi,i = 1
and µi,j = 0 for i > j), so the change of basis matrix M = (µi,j ) satisfies
V = V ∗ M.

Exercises for Chapter 6

193

(The rows of V are v1 , . . . , vn and the rows of V ∗ are v1∗ , . . . , vn∗ .)
In general, for any linear transformation A, we write A = (ai,j ) as a matrix
relative to the orthonormal basis e∗1 , . . . , e∗n and define
kAk =

sup
06=v∈Rn

kvAk
kvk

and

kAke = sup |ai,j |.
i,j

P
P 2
We observe that if v = xi e∗i , then kvk2 =
xi and we have
°X
°
n
X
° n
°2
°
kvAk2 = °
x
a
e
i
i,j
j
°
°
i=1

=
≤
≤

j=1

n µX
n
X
j=1
n
X
j=1
n
X

¶2

xi ai,j

i=1

kvk2 ka·,j k2

(Cauchy–Schwartz),

kvk2 nkA|2e

j=1

= n2 kAk2e kvk2 .
Taking square roots, dividing by kvk, and taking the sup over nonzero v yields
kAk ≤ nkAke .
To ease notation, we let
ci = kvi∗ k,

so vi∗ = ci ei .

Now we compute
ei M =

∗
c−1
i vi M

=

c−1
i vi

=

c−1
i

¶
µ
µ
¶
i−1
i−1
X
X
−1
∗
∗
µi,j vj = ci
vi +
ci ei +
µi,j cj ej
j=1

j=1

= ei +

i−1
X

µi,j c−1
i cj ej .

j=1

So relative to the e basis, the linear transformation M has a matrix that is
lower triangular with 1’s on the diagonal and with ij th entry satisfying
|µi,j c−1
i cj | ≤

1 kvj∗ k
·
≤ 2(i−j)/2−1 ,
2 kvi∗ k

(6.4)

where for the last inequality we use (6.57) (N.B. This is where we use the
fact that the basis is reduced, since the size condition gives |µi,j | ≤ 21 and the
Lovász condition implies the estimate (6.57).) Therefore,

194

Exercises for Chapter 6
kM ke ≤

max 2(i−j)/2−1 = 2(n−3)/2 .

1≤j≤i≤n

(If n ≤ 2, we need to replace this upper bound by 1.)
This allows us to get an upper bound
°X
°2
° n
°
2
∗
∗
∗
°
yi vi °
°
° = kyV k = kyV M k ≤ kyV k kM k ≤ kyV k · nkM ke
i=1

≤ kyV ∗ kn2(n−3)/2 = n2(n−3)/2

n
X

yi2 kvi∗ k2 .

(6.5)

i=1

To obtain a lower bound, we observe that
n
X
i=1

°
°2
yi2 kvi∗ k2 = kyV ∗ k2 = °yV M −1 °
° n
°2
°
° −1 °2 ° −1 °2 °X
°
°
°
°
°
yi vi °
≤ kyV k M
= M
°
° .

(6.6)

i=1

°
°
So we need an upper bound for °M −1 °.
Note that
M = I − N,
where N is lower triangular with 0’s on the diagonal, so N is nilpotent, and
indeed it satisfies N n = 0. Hence
M −1 = I + N + N 2 + · · · + N n−1 .
The following lemma provides the necessary estimate. We refer the reader
to [?, Lemma 7.10] for the proof.
Lemma 6.1. Let B = (bi,j ) be the matrix of a linear transformation relative
to an orthonormal basis {ei }. Suppose that there are positive constants β and γ
so that
bi,j = 0 for i ≤ j and |bi,j | ≤ γδ j−i for i > j.
Then

°
°
°B + B 2 + B 3 + · · · + B n−1 ° ≤ γ(γ + 1)n−2 δ n−1 .
e

Note that (??) tells us that the coefficients of N satisfy
(i−j)/2−1

|coef. of N | ≤ 2

1
= ·
2

µ

1
√
2

¶j−i
.

√
So we can apply the lemma to N with γ = 1/2 and δ = 1/ 2, which gives
°
°
°N + N 2 + · · · + N n−1 ° ≤ 1 ·
e
2

µ ¶n−2 µ
¶n−1
µ ¶(n−1)/2
3
1
1 9
· √
=
.
2
3 8
2

Exercises for Chapter 6

195

Hence
° −1 °
°
°
°
° ¢
¡
°M ° ≤ n °M −1 ° ≤ n 1 + °N + N 2 + · · · + N n−1 °
e
e
µ ¶(n−1)/2
µ ¶(n−1)/2
n 9
9
≤n+
≤n
.
3 8
8
(The last inequality is valid for n ≥ 8. For smaller n, one can put in a small
correction factor.)
Substituting into (??) gives
n
X

yi2 kvi∗ k2

i=1

°2
µ ¶n−1 °X
° n
°
9
°
yi vi °
≤n
°
° .
8
i=1
2

(6.7)

We now apply (6.54) from Theorem 6.66, which says that kvi∗ k2 ≥ 2−(i−1) kvi k2 .
This yields
°X
°2
µ ¶n−1 X
µ ¶n−1 X
n
n
° n
°
8
4
−2
2
−(i−1)
2
−2
°
°
yi vi ° ≥ n
yi · 2
kvi k ≥ n
yi2 kvi k2 .
°
9
9
i=1

i=1

i=1

P

(b) Let a =
ai vi ∈ L be any lattice vector, for example, it could be the
lattice vector that is closest to w. Write
X
w=
βi vi with βi ∈ R,
and let
b=

X

bi vi

with

bi = bβi e

be the vector returned by Babai’s algorithm. Also write
βi = bi + δi

with

|δi | ≤

1
.
2

Then
°X
°2
°
°
kw − ak2 = ° (βi − ai )vi °
X
(βi − ai )2 kvi k2
≥ C2n
X
= C2n
(bi − ai + δi )2 kvi k2 .

from (6.63),

If ai 6= bi , then |bi − ai | ≥ 1, so
(bi − ai + δi )2 ≥

1
(bi − ai )2 ,
4

and clearly this is also valid if ai = bi . Hence using the other inequality
in (6.63),

196

Exercises for Chapter 6

Input a basis v1 , . . . , vn of a lattice L.
Input a target vector t.
Compute Gram–Schmidt orthogonalized vectors v1∗ , . . . , vn∗ (Theorem 6.13).
Set w = t.
Loop i = n, n − 1, . . . , 2, 1
¥
¨
Set w = w − w · vi∗ /kvi∗ k2 vi .
End i Loop
Return the lattice vector t − w.
Figure 6.3: Babai’s closest plane algorithm

kw − ak2 ≥ C2n

X1

(bi − ai )2 kvi k2
°X
°2
°
°
1
°
(b
−
a
)v
≥ C2n C1−n °
i
i i°
°
4
=

4

1 n −n
C C kb − ak2 .
4 2 1

Using the triangle inequality, we find that
kw − ak ≥

¡
¢
1
1
(C2 /C1 )n/2 kb − ak ≥ (C2 /C1 )n/2 kb − wk − kw − ak ,
2
2

and now a little bit of algebra yields
à µ ¶
!
n/2
C1
kw − bk ≤ 2
+ 1 kw − ak.
C2
This shows that the Babai vector b is the closest vector to w up to a factor
of 2(C1 /C2 )n/2 + 1.
6.45. Babai’s Closest Plane Algorithm, which is described in Figure 6.10, is
an alternative rounding method that uses a given basis to solve apprCVP. As
usual, the more orthogonal the basis, the better the solution, so generally
people first use LLL to create a quasi-orthogonal basis and then apply one of
Babai’s methods. In both theory and practice, Babai’s closest plane algorithm
seems to yield better results than Babai’s closest vertex algorithm.
Implement both of Babai’s algorithms (Theorem 6.34 and Figure 6.10)
and use them to solve apprCVP for each of the following lattices and target
vectors. Which one gives the better result?
(a) L is the lattice generated by the rows of the matrix

Exercises for Chapter 6

197


−5
 26

 15
ML = 
 32

 15
5


16 25 25 13
8
−3 −11 14
5 −26 

−28 16 −7 −21 −4 

−3
7 −30 −6 26 

−32 −17 32 −3 11 
24
0 −13 −46 15

and the target vector is t = (−178, 117, −407, 419, −4, 252). (Notice that
the matrix ML is LLL reduced.)
(b) L is the lattice generated by the rows of the matrix


−33 −15 22 −34 −32 41
 10
9 45 10 −6 −3 


 −32 −17 43 37 29 −30 


ML = 

 26 13 −35 −41 42 −15 
 −50 32 18 35 48 45 
2 −5 −2 −38 38 41
and the target vector is t = (−126, −377, −196, 455, −200, −234). (Notice
that the matrix ML is not LLL reduced.)
(c) Apply LLL reduction to the basis in (b), and then use both of Babai’s
methods to solve apprCVP. Do you get better solutions?
Solution to Exercise 6.45.
(a) The Closest Plane Algorithm gives the vector
w = (−185, 105, −414, 419, −8, 277) = (−1, −4, −13, −3, 12, 5)ML ∈ L.
It satisfies kt − wk = 29.7153. The Closest Vertex Algorithm gives the vector
w = (−159, 102, −425, 433, −3, 251) = (−1, −3, −13, −3, 12, 5)ML ∈ L.
It satisfies kt − wk = 33.2866. So the Closest Plane Algorithm gives a slightly
better result than the Closest Vertex Algorithm.
(b) The Closest Plane Algorithm gives the vector
w = (−166, −394, −203, 460, −196, −204) = (−6, −13, 4, −12, −4, 3)ML ∈ L.
It satisfies kt − wk = 53.6563. The Closest Vertex Algorithm gives the vector
w = (−156, −385, −158, 470, −202, −207) = (−6, −12, 4, −12, −4, 3)ML ∈ L.
It satisfies kt − wk = 58.0172. So the Closest Plane Algorithm gives a slightly
better result than the Closest Vertex Algorithm.
(c) The LLL reduced basis is


10 9 45 10 −6 −3
 9 −3 11 37 28 15 


−24 −18 33 3 −4 56 


 2 −5 −2 −38 38 41 


−41 −14 32 0 1 −45
−35 53 −26 −5 24 −26

198

Exercises for Chapter 6

The Closest Plane Algorithm gives the vector
w = (−132, −367, −191, 467, −198, −263) = (−12, 6, 1, −9, 4, −4)ML ∈ L.
It satisfies kt − wk = 33.9116. The Closest Vertex Algorithm gives the exact
same result. So starting with an LLL reduced basis yields a significantly better
solution to apprCVP.
Section. Applications of LLL to cryptanalysis
6.46. You have been spying on George for some time and overhear him receiving a ciphertext e = 83493429501 that has been encrypted using the congruential cryptosystem described in Section 6.1. You also know that George’s
public key is h = 24201896593 and the public modulus is q = 148059109201.
Use Gaussian lattice reduction to recover George’s private key (f, g) and the
message m.
Solution to Exercise 6.46.
Gaussian lattice reduction on the lattice generated by
(1, 24201896593) and

(0, 148059109201)

gives the short basis
(233444, 255333)

and

(330721, −272507),

so the private key is
f = 233444

and g = 255333.

We check that
f −1 g ≡ 133037176740 · 255333 ≡ 24201896593 ≡ h (mod q).
In order to decrypt the message, we first compute
a ≡ f e ≡ 94843884201 (mod q).
Then we do a computation modulo g to recover the plaintext m,
m = f −1 a = 94649 · 94843884201 ≡ 186000 (mod g).

6.47. Let
M = (81946, 80956, 58407, 51650, 38136, 17032, 39658, 67468, 49203, 9546)
and let S = 168296. Use the LLL algorithm to solve the subset-sum problem
for M and S, i.e., find a subset of the elements of M whose sum is S.

Exercises for Chapter 6

199

Solution to Exercise 6.47.
We apply LLL to the matrix

2
0

0
0

0

0

0

0

0
0
1

0
2
0
0
0
0
0
0
0
0
1

0
0
2
0
0
0
0
0
0
0
1

0
0
0
2
0
0
0
0
0
0
1

0
0
0
0
2
0
0
0
0
0
1

0
0
0
0
0
2
0
0
0
0
1

0
0
0
0
0
0
2
0
0
0
1

0
0
0
0
0
0
0
2
0
0
1

0
0
0
0
0
0
0
0
2
0
1

0
0
0
0
0
0
0
0
0
2
1



81946
80956 

58407 
51650 

38136 

17032 
.
39658 

67468 

49203 
9546 
168296

It takes LLL 102 swaps to find the reduced matrix

1
1

3
3

−2

−2

−1

−2

−2
−4
2

−1
−1
1
1
0
−2
−3
2
4
−2
4

1
1
1
1
2
4
−1
0
4
0
2

1
1
3
−1
−2
−4
1
4
0
2
0

−1
−1
1
3
4
−2
1
−2
−2
4
−2

1
1
1
1
0
0
−3
−4
2
0
−2

−1
1
1
−3
2
2
−3
−2
0
0
−2

1
1
−3
1
2
0
−1
0
−2
2
2

1
−1
1
1
2
0
1
0
2
−4
−4

−1
1
−1
1
0
−2
1
2
0
−2
−4



0
1 

0 
2 

2 

−2
.
3 

−2

1 
0 
−1

The top row gives the solution
(0, −1, 0, 0, −1, 0, −1, 0, 0, −1, 1),
i.e., we have
80956 + 38136 + 39658 + 9546 = 168296.
This problem was created using the superincreasing sequence
r = (73, 160, 323, 657, 1325, 2660, 5348, 10698, 21396, 42807)
and the multiplier and modulus A = 79809 and B = 85733.
6.48. Alice and Bob communicate using the GGH cryptosystem. Alice’s public key is the lattice generated by the rows of the matrix


10305608 −597165
45361210
39600006 12036060
 −71672908 4156981 −315467761 −275401230 −83709146 


 −46304904 2685749 −203811282 −177925680 −54081387  .


 −68449642 3969419 −301282167 −263017213 −79944525 
−46169690 2677840 −203215644 −177405867 −53923216

200

Exercises for Chapter 6

Bob sends her the encrypted message
e = (388120266, −22516188, 1708295783, 1491331246, 453299858).
Use LLL to find a reduced basis for Alice’s lattice, and then use Babai’s
algorithm to decrypt Bob’s message.
Solution to Exercise 6.48.
LLL takes 52 swaps to produce the following matrix whose Hadamard
ratio is H = 0.963, so it is quite orthogonal:


72
 180

−158
 114
462

−116
−218
−301
172
164

172
−53
−230
−148
−258

−290
298
−185
−311
91



−51
161 

−25  .
297 
−491

Babai’s closest vertex method gives the lattice vector
(388120256, −22516180, 1708295793, 1491331242, 453299848)
that is close to the target vector e. If we let v1 , . . . , v5 be the LLL-reduced
basis vectors and w1 , . . . , w5 be the original basis vectors, then
(388120256, − 22516180, 1708295793, 1491331242, 453299848)
= 1622959v1 + 2403687v2 − 4093270v3 − 1942134v4 − 1269978v5
= −3w1 − 9w2 + 0w3 + 6w4 − 4w5 .
So Bob’s plaintext is the vector (−3, −9, 0, 6, −4).
6.49. Alice and Bob communicate using the NTRU cryptosystem with public
parameters (N, p, q, d) = (11, 3, 97, 3). Alice’s public key is
h = 39 + 9x + 33x2 + 52x3 + 58x4 + 11x5 + 38x6 + 6x7 + x8 + 48x9 + 41x10 .
Apply the LLL algorithm to the associated NTRU lattice to find an NTRU
private key (f , g) for h. Check your answer by verifying that g ≡ f ?h (mod q).
Use the private key to decrypt the ciphertext
e = 52 + 50x + 50x2 + 61x3 + 61x4 + 7x5 + 53x6 + 46x7 + 24x8 + 17x9 + 50x10 .
Solution to Exercise 6.49.
associated to h.
We apply LLL to the 22 dimensional NTRU lattice LNTRU
h
It requires 322 swaps and returns the LLL reduced the matrix

Exercises for Chapter 6
 −1


















1
0
0
0
0
−1
1
−1
0
1
−4
9
−6
−3
−9
7
−2
13
−1
−6
−3

−1
−1
1
1
1
0
0
−1
0
0
1
8
2
−2
7
7
1
−3
3
−5
11
−3

−1
−1
0
0
1
2
0
1
2
0
1
0
3
−5
2
7
0
9
−4
11
0
−3

0
0
−1
0
1
0
1
1
1
1
−1
0
0
−1
0
1
0
1
−1
1
0
0
1
0
1
−1
0
−2
0
−1
0
0
0
−2
1
−1
1
1
0
1
−1 −1 −1
0
0
−1
0
1
1
0
1
0
−1
0
0
1
1
−1
1
0
0
−1
0
−1
1
0
2
0
1
0
1
0
1
0
0
0
0
−2
0
−1
1
0
1
−1
1
0
0
−1
0
0
0
−1
0
0
−1 −1
0
−2
0
−2
−1
1
1
0
0
−2
0
0
0
−1
2
−7
0
1
−8
13
−4 −2
4
−9
−7 −1
0
−9
12
−4 −4 −3 −10
0
3
13
2
0
1
5
−11 −1 16
−3
0
8
−12
3
3
4
−8 −1
0
−7
−8 −14
2
−10 −8 −2
1
−1
4
−3
9 −12
4
4
3
−9 −2 −3 −7
6
2
3
−6 −1
0
−8
13 −4 −11
4
1
3
4
−12 −2
9
−14 −2 14
−2
1
6
2
5
−3 −13 −2
0
5
14
6
2
5
−3 −11 −3
1
−2 15
4
9
2
1
−8 −1
0
−7 13
18 −10

201
−1
0
0
−1 −1
0
1
0
−1
1
−1
0
0
−1 −1
0
1
0
0
0
−1
1
1
0
−1
1
0
0
1
0
0
0
0
−1
1
−1
−1
1
1
−1
0
0
−1 −1
0
1
0
0
0
1
1
0
−1
0
0
1
−1 −1
0
1
−1
0
0
1
−1
0
1
0
1
0
0
0
0
0
0
0
1
2
−1
0
0
0
0
−1
0
0
0
−1 −1
0
0
1
1
0
1
0
−1
1
0
−1
0
7
−7 −6
1
2
18 −10
0
7
−6 −5
1
1
17 −11
4
9
5
3
2
−8 −5 −14 −4 −2
7
6
−1 −2 −17 12 −5
9
1
1
−6
2
0
3
−1 21
12
−1
5
−1 −1 −17 11 −4 10
0
0
−8
0
0
8
−6 −5
1
2
18
−7
2
−10 −12
3
3
8
4
−4
4
2
−16
3
−9 −5 −3 −2
8
1 −16
2
−9 −5 −3 −2
9
4
5
−9
0
0
7
−7 −5
1
0

The top row is
(−1, −1, −1, 0, 0, −1, 0, 1, 0, 1, 1, 1, 1, −1, 0, 0, −1, −1, 0, 1, 0, −1),
which gives the private key polynomials
f (x) = −1 − x − x2 − x5 + x7 + x9 + x10
g(x) = 1 + x − x2 − x5 − x6 + x8 − x10 .
To decipher the message, we compute
a ≡ f ?e ≡ −11−13x−1x2 +3x3 −4x4 +2x5 +16x6 +4x7 +4x9 −2x10

(mod q).

Then we use
f −1 ≡ −1 + x − x3 + x4 − x6 + x7 − x8 − x9 + x10

(mod 3)

to compute the plaintext
m ≡ a ? f −1 1 − x − x2 − x3 − x4 + x7 + x10

(mod 3).

In vector form, m = (1, −1, −1, −1, −1, 0, 0, 1, 0, 0, 1).
√
6.50. (a) Suppose that k is a 10 digit integer, and suppose that when k is
computed, the first 15 digits after the decimal place are 418400286617716.
Find the number k. (Hint. Reformulate it as a lattice problem.)
(b) More generally,
suppose that you know the first d-digits after the decimal
√
place of K. Explain how to set up a lattice problem to find K.
See Exercise 1.47 for a cryptosystem associated to this problem.
Solution to Exercise 6.50.
We do (b) first, then illustrate the general idea by doing (a). Let α be√the ddigit number consisting of the first d digits after the decimal place of K. If
we let β = α/10d , then we can write
√
K ≈J +β
for some J ∈ Z.










.









202

Exercises for Chapter 6

There are two unknowns here, K and J, and all that we know is that they
are both integers. Squaring both sides gives
K ≈ J 2 + 2Jβ + β 2 .
Thus there are integers A and B satisfying
β 2 + Aβ + B ≈ 0,
namely A = 2J and B = J 2 − K. Of course, we don’t know A or B, so we now
describe a lattice reduction problem that finds a (quadratic) polynomial with
small integer coefficients that has a given decimal number as an (approximate)
root. Once we find A and B, it is easy to recover K as K = 41 A2 − B.
Let L be the lattice generated by the rows of the matrix


100 c
M = 0 1 0 cβ  ,
0 0 t cβ 2
where we will choose t and c later. Notice that


¡
¢ 100 c
¡
¢
B A 1 0 1 0 cβ  = B A t c(B + Aβ + β 2 ) .
0 0 t cβ 2
So if B + Aβ + β 2 is small, then we have found a small vector in the lattice. In
other words, if we find a small vector in the lattice having the form (B, A, ∗),
then we have probably found the values of A and B that we want. (We choose t
reasonably large to force the relation to look like (B, A, 1), instead of (B, A, C)
for some larger C.)
(b) A solution for this exercise is not currently available.

Chapter 7

Digital Signatures
Exercises for Chapter 7
Section. RSA digital signatures
7.1. Samantha uses the RSA signature scheme with primes p = 541 and
q = 1223 and public verification exponent v = 159853.
(a) What is Samantha’s public modulus? What is her private signing key?
(b) Samantha signs the digital document D = 630579. What is the signature?
Solution to Exercise 7.1.
(a) Samantha’s public modulus is N = p · q = 541 · 1223 = 661643. Samantha knows that (p − 1)(q − 1) = 540 · 1222 = 659880, so she can solve
vs ≡ 1 (mod (p − 1)(q − 1)),

159853 · s ≡ 1 (mod 659880),

for the private signing key s = 561517.
(b) Samantha takes the document D = 630579 and computes
S = Ds

(mod N ),

630579561517 ≡ 206484 (mod 661643).

So the signature is S = 206484.
She can check that this is correct by computing
C ≡ Sv

(mod N ),

C ≡ 206484159853 ≡ 630579 (mod 661643)

and noting that this value agrees with D = 630579.
7.2. Samantha uses the RSA signature scheme with public modulus N =
1562501 and public verification exponent v = 87953. Adam claims that
Samantha has signed each of the documents
D = 119812,

D0 = 161153,
203

D00 = 586036,

204

Exercises for Chapter 7

and that the associated signatures are
S 0 = 870099,

S = 876453,

S 00 = 602754.

Which of these are valid signatures?
Solution to Exercise 7.2.
Victor uses Samantha’s public key (N, v) = (1562501, 87953) to compute:
C ≡ Sv
0

0v

00

00 v

C ≡S
C ≡S

(mod N ),
(mod N ),
(mod N ),

C ≡ 87645387953 ≡ 772481

(mod 1562501),

0

87953

≡ 161153

(mod 1562501),

00

87953

≡ 586036

(mod 1562501).

C ≡ 870099
C ≡ 602754

Comparing the values of C, C 0 , C 00 with the document values D, D0 , D00 , we see
that S 0 and S 00 are valid signatures, but S is not. We remark that Samantha’s
private factorization is
N = p · q = 1301 · 1201 = 1562501
and her signing key is s = 261617.
7.3. Samantha uses the RSA signature scheme with public modulus and public verification exponent
N = 27212325191

and v = 22824469379.

Use whatever method you want to factor N , and then forge Samantha’s signature on the document D = 12910258780.
Solution to Exercise 7.3.
The factorization of Samantha’s public modulus is
N = p · q = 128311 · 212081 = 27212325191.
Then (p − 1)(q − 1) = 128310 · 212080 = 27211984800, so we can solve
vs ≡ 1

(mod (p − 1)(q − 1)),

22824469379 · s ≡ 1

(mod 27211984800)

for Samantha’s private signing exponent s = 18408628619. We can then sign
the document D = 12910258780 by computing
S ≡ Ds

(mod N ),

1291025878018408628619 ≡ 22054770669 (mod 27212325191).
To check that this signature is correct, we compute
C ≡ Sv

(mod N ),

C ≡ 2205477066922824469379 ≡ 12910258780 (mod 27212325191)
and note that it agrees with D = 12910258780.

Exercises for Chapter 7

205

Section. Discrete logarithm digital signatures
7.4. Samantha uses the ElGamal signature scheme with prime p = 6961 and
primitive root g = 437.
(a) Samantha’s private signing key is s = 6104. What is her public verification
key?
(b) Samantha signs the digital document D = 5584 using the ephemeral key
e = 4451. What is the signature?
Solution to Exercise 7.4.
(a)
v ≡ 4376104 ≡ 2065 (mod 6961).
(b)
S1 ≡ 4374451 ≡ 3534 (mod 6961),
S2 ≡ (5584 − 6104 · 3534)4451−1 ≡ 5888 (mod 6960).
So the signature on D is (3534, 5888).
7.5. Samantha uses the ElGamal signature scheme with prime p = 6961 and
primitive root g = 437. Her public verification key is v = 4250. Adam claims
that Samantha has signed each of the documents
D = 1521,

D0 = 1837,

D00 = 1614,

and that the associated signatures are
(S1 , S2 ) = (4129, 5575),

(S10 , S20 ) = (3145, 1871),

(S100 , S200 ) = (2709, 2994).

Which of these are valid signatures?
Solution to Exercise 7.5.
(a)
v S1 · S1S2 ≡ (42504129 ) · 41295575 ≡ 231 (mod 6961).
g D ≡ 4371521 ≡ 231 (mod 6961).
So the signature is valid. (The ephemeral key was e = 5627.)
(b)
v S1 · S1S2 ≡ (42503145 ) · 31451871 ≡ 6208 (mod 6961).
g D ≡ 4371837 ≡ 2081 (mod 6961).
So the signature is not valid.
(c)
v S1 · S1S2 ≡ (42502709 ) · 27092994 ≡ 2243 (mod 6961).
g D ≡ 4371614 ≡ 2243 (mod 6961).
So the signature is valid. (The ephemeral key was e = 3997.)
(Samantha’s private signing key is s = 4804.)

206

Exercises for Chapter 7

7.6. Let p be a prime and let i and j be integers with gcd(j, p − 1) = 1. Set
S1 ≡ g i v j (mod p),

S2 ≡ −S1 j −1 (mod p − 1),

D ≡ −S1 ij −1 (mod p − 1).

Prove that (S1 , S2 ) is a valid ElGamal signature on the document D for the
verification key v. Thus Eve can produce signatures on random documents.
Solution to Exercise 7.6.
We compute
v S1 S1S2 ≡ v S1 (g i v j )−S1 j
≡ v S1 g −ij
≡g

D

−1

−1

S1 −S1

v

(mod p)
(mod p)

(mod p).

7.7. Suppose that Samantha is using the ElGamal signature scheme and that
she is careless and uses the same ephemeral key e to sign two documents D
and D0 .
(a) Explain how Eve can tell at a glance whether Samantha has made this
mistake.
(b) If the signature on D is (S1 , S2 ) and the signature on D0 is (S10 , S20 ),
explain how Eve can recover s, Samantha’s private signing key.
(c) Apply your method from (b) to the following example and recover Samantha’s signing key s, where Samantha is using the prime p = 348149,
base g = 113459, and verification key v = 185149.
D = 153405,
D0 = 127561,

S1 = 208913,
S10 = 208913,

S2 = 209176,
S20 = 217800.

Solution to Exercise 7.7.
0
(a) Since S1 ≡ g e and S10 = g e , Eve can check if the two signatures used
the same ephemeral key by checking if S1 = S10 .
(b) Using discrete logarithms to the base g, the verification conditions are
S1 log(v) + S2 log(S1 ) ≡ D (mod p − 1),
S10 log(v) + S20 log(S10 ) ≡ D0 (mod p − 1).
Since S1 = S10 from (a), this becomes
S1 s + S2 log(S1 ) ≡ D (mod p − 1),
S1 s + S20 log(S1 ) ≡ D0 (mod p − 1),
where s = log(v) is Samantha’s secret signing key. Taking S20 times the first
congruence and subtracting S2 times the second congruence, we obtain
S1 (S20 − S2 )s ≡ S20 D − S2 D0 (mod p − 1).

Exercises for Chapter 7

207

For notational convenience we write this congruence as
As ≡ B (mod p − 1),
where we know the values of A and B. If gcd(A, p − 1) = 1, we can solve
uniquely for s. In general, if gcd(A, p − 1) > 1 (it’s unlikely to be too large),
then there are gcd(A, p − 1) solutions for s, and after computing them, we can
decide which one is correct by checking which one yields g s ≡ v (mod p).
(c) From (b) we begin by computing
A ≡ S1 (S20 − S2 ) ≡ 347960 (mod p − 1),
B ≡ S20 D − S2 D0 ≡ 252868 (mod p − 1).
We need to solve As ≡ B (mod p − 1), so we need to solve
347960s ≡ 252868 (mod 348148).
This congruence has several solutions. More precisely, since gcd(347960, 348148) =
4 and 4 | 252868, we divide through by 4 to get
86990s ≡ 63217 (mod 87037).
Then gcd(86990, 87037) = 1, so we can solve this congruence. The solution is
s ≡ 72729 (mod 87037).
Adding on multiples of (p − 1)/4 = 87037 yields the four solutions
s ≡ 72729, 159766, 246803, 333840 (mod 348148)
to the original congruence. We can pick out which solution is correct from the
relation g s ≡ v (mod p), i.e., the correct value of s should satisfy
113459s ≡ 185149

(mod 348149).

We compute
11345972729 ≡ 185149

(mod 348149),

113459

159766

≡ 137653

(mod 348149),

113459

246803

≡ 163000

(mod 348149),

113459

333840

≡ 210496

(mod 348149).

Hence Samantha’s secret signing key is
s = 72729.

208

Exercises for Chapter 7

7.8. Samantha uses DSA with public parameters (p, q, g) = (22531, 751, 4488).
She chooses the secret signing key s = 674.
(a) What is Samantha’s public verification key?
(b) Samantha signs the document D = 244 using the ephemeral key e = 574.
What is the signature?
Solution to Exercise 7.8.
(a) Samantha’s public verification key is
v ≡ 4488674 ≡ 4940 (mod 22531).
(b) The signature is
S1 = (4488574 mod 22531) mod 751 = 444,
S2 ≡ (244 + 674 · 444)574−1 ≡ 56 (mod 751).

7.9. Samantha uses DSA with public parameters (p, q, g) = (22531, 751, 4488).
Her public verification key is v = 22476.
(a) Is (S1 , S2 ) = (183, 260) a valid signature on the document D = 329?
(b) Is (S1 , S2 ) = (211, 97) a valid signature on the document D = 432?
Solution to Exercise 7.9.
(a) Victor computes
V1 ≡ 329 · 260−1 ≡ 293 (mod 751)

and V2 ≡ 183 · 260−1 ≡ 252 (mod 751).

He then computes
g V1 v V2 ≡ 4488293 · 22476252 ≡ 6191 (mod 22531)
and verifies that 6191 mod 751 = 183 is equal to S1 . So the signature is valid.
(Samantha’s secret signing key happens to be s = 38.)
(b) Victor computes
V1 ≡ 432 · 97−1 ≡ 709 (mod 751)

and V2 ≡ 211 · 97−1 ≡ 428 (mod 751).

He then computes
g V1 v V2 ≡ 4488709 · 22476428 ≡ 3979 (mod 22531).
Then he observes that
(g V1 v V2 mod p) mod q = 3979 mod 751 = 224
is not equal to S1 = 211. So the signature is not valid.

Exercises for Chapter 7

209

7.10. Samantha’s DSA public parameters are (p, q, g) = (103687, 1571, 21947),
and her public verification key is v = 31377. Use whatever method you prefer
(brute-force, collision, index calculus, . . . ) to solve the DLP and find Samantha’s private signing key. Use her key to sign the document D = 510 using
the ephemeral key e = 1105.
Solution to Exercise 7.10.
Solving 31377 ≡ 21947s (mod 103687) gives s = 602. Then the signature
on D = 510 using the ephemeral key e = 1105 is
S1 = (219471105 mod 103687) mod 1571 = 439
S2 ≡ (510 + 602 · 439)1105−1 ≡ 1259 (mod 1571).

7.11. The Elliptic Curve Digital Signature Algorithm (ECDSA) is described
in Table 7.7. Prove that ECDSA works, i.e., prove that the verification step
succeeds in verifying a valid signature.
Solution to Exercise 7.11.
We compute
−1
v1 G + v2 V = ds−1
2 G + s1 s2 (sG)

= (d + ss1 )s−1
2 G
= (es2 )s2−1 G
= eG ∈ E(Fp ).
Hence
x(v1 G + v2 V ) mod q = x(eG) mod q = s1 .
7.12. This exercise asks you to compute some numerical instances of the
elliptic curve digital signature algorithm described in Table 7.7 for the public
parameters
E : y 2 = x3 +231x+473,

p = 17389,

q = 1321,

G = (11259, 11278) ∈ E(Fp ).

You should begin by verifying that G is a point of order q in E(Fp ).
(a) Samantha’s private signing key is s = 542. What is her public verification
key? What is her digital signature on the document d = 644 using the
ephemeral key e = 847?
(b) Tabitha’s public verification key is V = (11017, 14637). Is (s1 , s2 ) =
(907, 296) a valid signature on the document d = 993?
(c) Umberto’s public verification key is V = (14594, 308). Use any method
that you want to find Umberto’s private signing key, and then use the
private key to forge his signature on the document d = 516 using the
ephemeral key e = 365.

210

Exercises for Chapter 7

Public Parameter Creation
A trusted party chooses a finite field Fp , an elliptic curve E/Fp ,
and a point G ∈ E(Fp ) of large prime order q.
Samantha
Victor
Key Creation
Choose secret signing key
1 < s < q − 1.
Compute V = sG ∈ E(Fp ).
Publish the verification key V .
Signing
Choose document d mod q.
Choose ephemeral key e mod q.
Compute eG ∈ E(Fp ) and then,
s1 = x(eG) mod q and
s2 ≡ (d + ss1 )e−1 (mod q).
Publish the signature (s1 , s2 ).
Verification
Compute v1 ≡ ds−1
(mod q) and
2
v2 ≡ s1 s−1
(mod
q).
2
Compute v1 G+v2 V ∈ E(Fp ) and verify that
x(v1 G + v2 V ) mod q = s1 .

Table 7.1: The elliptic curve digital signature algorithm (ECDSA)
Solution to Exercise 7.12.
(a) Samantha’s public verification key is
V = 542(11259, 11278) = (8689, 1726) ∈ E(Fp ).
Her signature on d = 644 using e = 847 is obtained by first computing eG =
(8417, 8276) ∈ E(Fp ) and then
s1 = x(eG) mod q = 491

and

s2 ≡ (d + ss1 )e−1 ≡ 290 (mod q).

(b) Victor computes
v1 ≡ ds−1
2 ≡ 106 (mod q) and

v2 ≡ s1 s−1
2 ≡ 311 (mod q).

Then v1 G + v2 V = (8833, 4526) ∈ E(Fp ), and
x(v1 G + v2 V ) mod q = 8833 mod 1321 = 907
is equal to s1 , so the signature is valid.
(c) After some work, one finds that Umberto’s private signing key is s = 1294,
since

Exercises for Chapter 7

211

1294G = 1294(11259, 11278) = (14594, 308) ∈ E(Fp ).
We can then forge a signature on the document d = 516 using the ephemeral
key e = 365 by first computing eG = (3923, 12121) ∈ E(Fp ) and then
s1 = x(eG) mod q = 1281

and

s2 ≡ (d + ss1 )e−1 ≡ 236 (mod q).

To check that the signature is valid, we compute v1 G + v2 V = (3923, 12121) ∈
E(Fp ), and
x(v1 G + v2 V ) mod q = 3923 mod 1321 = 1281,
which is equal to s1 .
Section. Lattice-based digital signatures
7.13. Samantha uses the GGH digital signature scheme with private and
public bases
v1 = (−20, −8, 1),
v2 = (14, 11, 23),
v3 = (−18, 1, −12),

w1 = (−248100, 220074, 332172),
w2 = (−112192, 99518, 150209),
w3 = (−216150, 191737, 289401).

What is her signature on the document
d = (834928, 123894, 7812738)?
Solution to Exercise 7.13.
Samantha uses Babai’s algorithm with the good basis to find the vector
s = 283411v1 + 233700v2 − 179519v3 = (834922, 123893, 7812739)
that is close to d,
ks − dk ≈ 6.16.
She then expresses the signature in terms of bad basis,
s = 785152901w1 − 1383699316w2 − 183004589w3
and publishes the signature (785152901, −1383699316, −183004589).
7.14. Samantha uses the GGH digital signature scheme with public basis
w1 = (3712318934, −14591032252, 11433651072),
w2 = (−1586446650, 6235427140, −4886131219),
w3 = (305711854, −1201580900, 941568527).
She publishes the signature
(6987814629, 14496863295, −9625064603)
on the document
d = (5269775, 7294466, 1875937).
If the maximum allowed distance from the signature to the document is 60,
verify that Samantha’s signature is valid.

212

Exercises for Chapter 7

Solution to Exercise 7.14.
We first compute
s = 6987814629w1 + 14496863295w2 − 9625064603w3
= (5269774, 7294492, 1875902) ∈ L.
Then we compute the distance
ks − dk ≈ 43.61
and verify that it is smaller than the cutoff value of 60, so the signature is
valid.
7.15. Samantha uses the GGH digital signature scheme with public basis
w1 = (−1612927239, 1853012542, 1451467045),
w2 = (−2137446623, 2455606985, 1923480029),
w3 = (2762180674, −3173333120, −2485675809).
Use LLL or some other lattice reduction algorithm to find a good basis for
Samantha’s lattice, and then use the good basis to help Eve forge a signature
on the document
d = (87398273893, 763829184, 118237397273).
What is the distance from your forged signature lattice vector to the target
vector? (You should be able to get a distance smaller than 100.)
Solution to Exercise 7.15.
Eve’s implementation of LLL gives the basis
v1 = (−9, −147, −136),

v2 = (73, 169, −41),

v3 = (109, −132, −110).

Using this LLL reduced basis, she computes
s = −1542740188v1 − 532211991v2 + 1030872363v3
= (87398273916, 763829241, 118237397269) ∈ L.
It satisfies
ks − dk ≈ 61.60,
so is quite a good solution. To find the signature, Eve expresses s in terms of
the original bad basis,
s = 203927306009123w1 + 225365519245447w2 + 293473443761381w3 .
The signature is
(203927306009123, 225365519245447, 293473443761381).

Exercises for Chapter 7

213

Section. NTRU digital signatures
7.16. Samantha uses an NTRU digital signature with (N, q, d) = (11, 23, 3).
(a) Samantha’s private key is
f = (1, −1, 1, 0, 1, 0, −1, 1, 0, −1, 0),
g = (0, −1, 0, 1, 1, 0, 0, 1, −1, 1, −1),
F = (0, −1, −1, 1, −3, −1, 0, −3, −3, −2, 2),
G = (−3, −1, 2, 4, 3, −4, −1, 3, 5, 5, −1).
She uses her private key to sign the digital document D = (D1 , D2 ) given
by
D1 = (0, 8, −6, −6, −5, −1, 9, −2, −6, −4, −6),
D2 = (9, 9, −10, 2, −3, 2, 6, 6, 5, 0, 8).
Compute the signature s.
(b) Samantha’s public verification key is
h = (5, 8, −5, −11, 8, 8, 8, 5, 3, −10, 5).
Compute the other part of the signature t ≡ h ? s (mod q) and find the
distance between the lattice vector (s, t) and the target vector D.
(c) Suppose that Eve attempts to sign D using Samantha’s public vectors (1, h) and (0, q). What signature (s0 , t0 ) does she get and how far is
it from the target vector D?
Solution to Exercise 7.16.
(a) Samantha first computes
¥¡
¢¨
−64 32 143 146 1 −178 −1,120 108 25
v1 = −164
23 , 23 , 23 , 23 , 23 , 23 , 23 ,
23 , 23 , 23
= (−7, −3, 1, 6, 6, 0, −8, −1, 5, 5, 1),
¥¡
¢¨
13 0,35 −27 4
2 16 11 −24 −4
v2 = 27
23 , 23 , 23 , 23 , 23 , 23 , 23 , 23 , 23 , 23
= (1, 1, 0, 2, −1, 0, 0, 1, 0, −1, 0),
and then
s = (−2, 8, −4, −7, −5, −2, 9, −4, −9, −4, −8).
(b)
t ≡ h ? s ≡ (11, 9, −9, 2, −4, 5, 4, 8, 4, 0, 11)
Then
k(s, t) − Dk = 7.74597.
(c) The signature using the h-basis is

(mod 23).

214

Exercises for Chapter 7
s0 = (0, 8, −6, −6, −5, −1, 9, −2, −6, −4, −6),
t0 = (5, 17, −3, 9, 3, 2, 14, 13, 2, 10, 1).

It satisfies
k(s0 , t0 ) − Dk = 22.0227.

7.17. Samantha uses an NTRU digital signature with (N, q, d) = (11, 23, 3).
(a) She creates a private key using the ternary vectors
f = (1, 1, 1, 1, 0, −1, −1, 0, 0, 0, −1),
g = (−1, 0, 1, 1, −1, 0, 0, 1, −1, 0, 1).
Use the algorithm described in Table 7.6 to find short vectors F and G
satisfying f ? G − g ? F = q.
(b) Samantha uses the private signing key (f , g, F, G) to sign the digital
document D = (D1 , D2 ) given by
D1 = (5, 5, −5, −10, 3, −7, −3, 2, 0, −5, −11),
D2 = (8, 9, −10, −7, 6, −3, 1, 4, 4, 4, −7).
What the signature s?
(c) What is Samantha’s public verification key h?
(d) Compute t ≡ h ? s (mod q) and determine the distance from the lattice
vector (s, t) to the target vector D.
Solution to Exercise 7.17.
(a) First she computes
f1
f2
g1
g2

= (1363, −944, 786, 358, 482, 80, −133, 368, 955, 126, 476),
= (−2554, 419, 1205, 1563, 682, 343, 368, 955, 126, 476, 0),
= (−63, 121, −173, −232, 208, −251, 96, 172, 620, 799, 574),
= (−1808, −121, 110, 290, −197, −275, 53, −46, −799, −574, 0).

The resultants are Rf = 3917 and Rg = 1871, and then Sf = −866 and
Sg = 1813 satisfies Sf Rf + Sg Rg = 1. Then we find that
A = (2627037, −5045579, 7213927, 9674168, −8673392, 10466449,
−4003104, −7172228, −25853380, −33317501, −23935226),
B = (−27148234, 18802592, −15655548, −7130644, −9600476, −1593440,
2649094, −7329824, −19021690, −2509668, −9480968).
Next we use

Exercises for Chapter 7

215

f −1 = (0.35, −0.24, 0.20, 0.09, 0.12, 0.02, −0.03, 0.09, 0.24, 0.03, 0.12),
g−1 = (−0.03, 0.06, −0.09, −0.12, 0.11, −0.13, 0.05, 0.09, 0.33, 0.43, 0.31),
to compute
C = (−2522997, −13372262, 753797, −4259685, −6431876, −1572245,
−16151212, −8516655, −12881295, −9762142, −3302247).
Using this yields
F = (−1, −1, 1, 2, 0, 2, 0, 0, −3, −5, −5),
G = (4, −4, 3, 2, 1, 0, 0, 2, 3, 0, 2).
These are reasonably short, kF k = 8.36660 and kGk = 7.93725.
(b)
¥¡
¢¨
−117 −116 −37 51 −40 7
7
−22 −2
v1 = −2
23 , 23 , 23 , 23 , 23 , 23 , 23 , 23 , 1, 23 , 23
= (0, −5, −5, −2, 2, −2, 0, 0, 1, −1, 0)
¢¨
¥¡
55 1 −53 21 5 −21 −15 37 31 −22
v2 = −4
23 , 23 , 23 , 23 , 23 , 23 , 23 , 23 , 23 , 23 , 23
= (0, 2, 0, −2, 1, 0, −1, −1, 2, 1, −1)
Then
s = (7, 3, −4, −10, 4, −6, −3, 1, 1, −5, −10).
(c) We have
f −1 mod 23 = (−9, −10, −6, −8, −10, −5, 4, 0, 5, −5, −1),
h = (−3, −2, 2, −1, 3, −5, 11, −10, −2, −6, −9).
(d)
t = (9, 6, −9, −7, 4, −4, 0, 4, 5, −2, −5).
k(s, t) − Dk = 8.48528.
7.18. Let a ∈ RN be a fixed vector.
(a) Suppose that b is an N -dimensional vector whose coefficients are chosen
randomly from the set {−1, 0, 1}. Prove that the expected values of kbk2
and ka ? bk2 are given by
¡
¢ 2
E kbk2 = N
3

¡
¢
¡
¢
and E ka ? bk2 = kak2 E kbk2 .

(b) More generally, suppose that the coefficients of b are chosen at random
from the set of integers {−T, −T +1, . . . , T −1, T }. Compute the expected
values of kbk2 and ka ? bk2 as in (a).

216

Exercises for Chapter 7

(c) Suppose now that the coefficients of b are real numbers that are chosen
uniformly and independently in the interval from −R to R. Prove that
¡
¢ R2 N
E kbk2 =
3

and

¡
¢
¡
¢
E ka ? bk2 = kak2 E kbk2 .

(Hint. The most direct way to do (c) is to use continuous probability
theory. As an alternative, let the coefficients of b be chosen uniformly and
independently from the set {jR/T : −T ≤ j ≤ T }, redo the computation
from (b), and then let T → ∞.)
Solution to Exercise 7.18.
Let c = a ? b. Then
X
kck2 =
c2k
k mod N

X µ

=

k mod N

X

=

¶2

X

ai bj

i+j≡k (mod N )

X

k mod N i+j≡k (mod N )

X

=

X

ai bj

au bv

u+v≡k (mod N )

ai au bj bv .

i+j≡u+v (mod N )

Note that this last sum is over all 4-tuples (i, j, u, v) mod N satisfying i + j ≡
u + v (mod N ). We suppose now that the coefficients of b are independent
random variables whose average value is 0, i.e., we assume that E(bi ) = 0.
This is a valid assumption in (a), (b), and (c). Since the coefficients of a are
fixed, we can compute
X
¡
¢
E(ai au bj bv )
E ka ? bk2 =
i+j≡u+v (mod N )

X

=

ai au E(bj bv )

i+j≡u+v (mod N )

X

=
X

a2i

i mod N
2

X

E(b2j )

j mod N

+ · · · + b2N −1 )
¢
= kak2 E kbk2 .
= kak

X

i mod N j mod N

i+j≡u+v (mod N )
j6=v

=

X

ai au E(bj )E(bv ) +

E(b20
¡

Hence in all cases we have
¡
¢
¡
¢
E ka ? bk2 = kak2 E kbk2 .

a2i E(b2j )

Exercises for Chapter 7

217

¡
¢
It remains to compute E kbk2 under the various scenarios.
(a) The coefficients of b are independent random variables taking values
in {−1, 0, 1} with equal probabilities, so
1
1
1
· (−1) + · 0 + · 1 = 0,
3
3
3
1
1 2 1 2
2
2
2
E(bi ) = · (−1) + · 0 + · 1 = ,
3
3
3
3
¢
¡
2
2
2
2
2
E kbk = E(b0 + · · · + bN −1 ) = E(b0 ) + · · · + E(b2N −1 ) = N.
3
E(bi ) =

(b) Similar to (a), but now the values are integers between −T and T . So
E(b2i ) =

T
T
X
X
1
T2 + T
2
2 T (T + 1)(2T + 1)
j2 =
j2 =
=
.
2T + 1
2T + 1 j=1
2T + 1
6
3
j=−T

Hence

−1
¡
¢ NX
T2 + T
E kbk2 =
E(b2j ) =
N.
3
i=0

(c) The computation using continuous probability is
Z R
1
1 b3 ¯¯R
R2
E(b2i ) =
b2 db =
·
=
.
−R
2R −R
2R 3
3
¡
¢
Hence E kbk2 = R2 N/3.
The alternative computation using the hint gives, for any particular value
of T ,
E(b2i )

µ ¶2
T
T
X
X
1
2R2
jR
=
=
j2
2T + 1
T
(2T + 1)T 2 j=1
j=−T

2R2
T (T + 1)(2T + 1)
R2 (T + 1)
=
=
.
2
(2T + 1)T
6
3T
Letting T → ∞ yields E(b2i ) = R2 /3, and then
¡
¢
E kbk2 = N R2 /3.

7.19. Let (f , g, F, G) be an NTRU digital signature private key and let
h ≡ f −1 ? g (mod q)
be the associated public key. Suppose that (s, t) is the signature on the document D = (D1 , D2 ), so in particular, the vector (s, t) is in the NTRU lattice LNTRU
.
h

218

Exercises for Chapter 7

(a) Prove that for every vector w ∈ ZN , the vector
(s + w ? f , t + w ? g)
is in the NTRU lattice LNTRU
.
h
(b) Let f −1 be the inverse of f in the ring R[x]/(xN − 1) (cf. Table 7.6). Prove
that the vector
s0 = s + b−f −1 ? D1 e ? f
is a signature on a document of the form D0 = (0, D2 + D3 ) for some D3
that depends on D1 .
(c) Conclude that anyone who can sign documents of the form (0, D0 ) is also
able to sign documents of the form (D1 , D2 ). Hence in the NTRU digital
signature scheme (Table 7.5), we might as well assume that the document
being signed is of the form (0, D2 ). This has several benefits, including
speeding the computation of v1 and v2 .
Solution to Exercise 7.19.
(a) The NTRU lattice is characterized as the set of vectors
©
ª
LNTRU
= (u, v) : v ≡ u ? h (mod q) .
h
We are given that (s, t) ∈ LNTRU
, so
h
(s + w ? f ) ? h ≡ s ? h + w ? f ? h ≡ t + w ? g (mod q),
where we have used the fact that h ≡ f −1 ? g (mod q).
(b) For notational convenience, we let w = b−f −1 ? D1 e. Then
w ? f = −D1 + ² ? f ,
where the coefficients of ² are between − 12 and 12 . Hence
s0 = s + w ? f = s − D1 + ² ? f .
We know that ks − D1 k is small, so we find that
ks0 k ≤ ks − D1 k + k² ? f k ≈ ks − D1 k + k²k kf k
is also small.
Next we compute
t0 ≡ s0 ? h ≡ s ? h + w ? f ? h ≡ t + w ? g (mod q).
Since t is close to D2 , we find that t0 is close to D2 + D3 , where
D3 = w ? g = b−f −1 ? D1 e ? g.
(c) Suppose that Eve knows how to sign documents of the form (0, D0 ) and
that she wants to sign (D1 , D2 ). She starts by finding a signature s0 to the
document

Exercises for Chapter 7
¡

219

¢
0, D2 + b−f −1 ? D1 e ? g .

Then from the calculations in (b), she sees that
s = s0 − b−f −1 ? D1 e ? g
is a signature on (D1 , D2 ).
7.20. Verify the identity
µ
¶µ
¶
¶ µ
f g
f ?f +F?F g?f +G?F
f F
,
=
FG
gG
f ?g+F?G g?f +G?F
where bar indicates reversal of a vector as in Remark 7.13. Prove that the
corresponding
¡ 2N
¢-by-2N matrix is the Gram matrix associated to the 2N -byg
2N matrix Ff G
. (See Exercise 6.14 for the definition of the Gram matrix.)

Chapter 8

Additional Topics in
Cryptography

221

http://www.springer.com/978-0-387-77993-5



Source Exif Data:
File Type                       : PDF
File Type Extension             : pdf
MIME Type                       : application/pdf
PDF Version                     : 1.4
Linearized                      : No
Page Count                      : 223
Create Date                     : 2011:04:15 17:54:08+02:00
Producer                        : iText 5.0.4 (c) 1T3XT BVBA
Modify Date                     : 2011:04:15 17:54:08+02:00
EXIF Metadata provided by EXIF.tools

Navigation menu